0% found this document useful (0 votes)
149 views244 pages

Topics in Calculus

Uploaded by

SSA
Copyright
© © All Rights Reserved
We take content rights seriously. If you suspect this is your content, claim it here.
Available Formats
Download as PDF, TXT or read online on Scribd
0% found this document useful (0 votes)
149 views244 pages

Topics in Calculus

Uploaded by

SSA
Copyright
© © All Rights Reserved
We take content rights seriously. If you suspect this is your content, claim it here.
Available Formats
Download as PDF, TXT or read online on Scribd
You are on page 1/ 244

B.A.

(Programme) Minor Paper


Paper-II Discipline A1/Paper-III Discipline B1
Semester-I
Course Credit - 4

(Department of Mathematics)
As per the UGCF - 2022 and National Education Policy 2020
Topics in Calculus

Editorial Board
Prof. S.K.Verma
Dr. K.A.Sharma

Content Writers
Mr. Sandeep Bhatt
Ms. Setu Rani
Ms. Parvinder Kaur
Ms. Mridu Sharma

Academic Coordinator
Deekshant Awasthi

© Department of Distance and Continuing Education


ISBN : 978-93-95774-36-9
1st edition: 2022
E-mail: [email protected]
[email protected]

Published by:
Department of Distance and Continuing Education under
the aegis of Campus of Open Learning/School of Open Learning,
University of Delhi, Delhi-110 007

Printed by:
School of Open Learning, University of Delhi

© Department of Distance & Continuing Education, Campus of Open Learning,


School of Open Learning, University of Delhi
Topics in Calculus

 Corrections/Modifications/Suggestions proposed by Statutory Body, DU/Stakeholder/s in the Self


Learning Material (SLM) will be incorporated in the next edition. However, these
corrections/modifications/suggestions will be uploaded on the website https://sol.du.ac.in. Any
feedback or suggestions can be sent to the email- [email protected]

© Department of Distance & Continuing Education, Campus of Open Learning,


School of Open Learning, University of Delhi
Contents

1 Limits 6
1.1 Learning Objectives . . . . . . . . . . . . . . . . . . . . . . . . . . . . . . 6
1.2 Introduction . . . . . . . . . . . . . . . . . . . . . . . . . . . . . . . . . . 7
1.3 Limit (An Informal Approach) . . . . . . . . . . . . . . . . . . . . . . . . 7
1.4 Limit (Formal Approach) . . . . . . . . . . . . . . . . . . . . . . . . . . . 12
1.5 Algebraic Properties of Limits . . . . . . . . . . . . . . . . . . . . . . . . 15
1.6 Infinite Limits and Limits at Infinity . . . . . . . . . . . . . . . . . . . . . 20
1.7 Summary . . . . . . . . . . . . . . . . . . . . . . . . . . . . . . . . . . . 26
1.8 Self-Assessment Exercises . . . . . . . . . . . . . . . . . . . . . . . . . . 28
1.9 Solutions to In-text Exercises . . . . . . . . . . . . . . . . . . . . . . . . . 29
1.10 Suggested Readings . . . . . . . . . . . . . . . . . . . . . . . . . . . . . . 30

2 Continuity and Differentiability 31


2.1 Learning Objectives . . . . . . . . . . . . . . . . . . . . . . . . . . . . . . 31
2.2 Introduction . . . . . . . . . . . . . . . . . . . . . . . . . . . . . . . . . . 32
2.3 Continuity . . . . . . . . . . . . . . . . . . . . . . . . . . . . . . . . . . . 32
2.4 Properties of Continuous Functions . . . . . . . . . . . . . . . . . . . . . . 38
2.5 Differentiability . . . . . . . . . . . . . . . . . . . . . . . . . . . . . . . . 40
2.6 Some Theorems on Derivatives . . . . . . . . . . . . . . . . . . . . . . . . 46
2.7 Summary . . . . . . . . . . . . . . . . . . . . . . . . . . . . . . . . . . . 51
2.8 Self-Assessment Exercises . . . . . . . . . . . . . . . . . . . . . . . . . . 54
2.9 Solutions to In-text Exercises . . . . . . . . . . . . . . . . . . . . . . . . . 55
2.10 Suggested Readings . . . . . . . . . . . . . . . . . . . . . . . . . . . . . . 56

3 Successive Differentiation 57
3.1 Learning Objectives . . . . . . . . . . . . . . . . . . . . . . . . . . . . . . 57
3.2 Introduction . . . . . . . . . . . . . . . . . . . . . . . . . . . . . . . . . . 58
3.3 nth Derivatives of Some Standard Functions . . . . . . . . . . . . . . . . . 61
3.4 nth Derivative of Rational Functions . . . . . . . . . . . . . . . . . . . . . 67
3.5 Leibnitz’s Theorem . . . . . . . . . . . . . . . . . . . . . . . . . . . . . . 70
3.6 Summary . . . . . . . . . . . . . . . . . . . . . . . . . . . . . . . . . . . 73
3.7 Self-Assessment Exercises . . . . . . . . . . . . . . . . . . . . . . . . . . 75
3.8 Solutions to In-text Exercises . . . . . . . . . . . . . . . . . . . . . . . . . 76
3.9 Suggested Readings . . . . . . . . . . . . . . . . . . . . . . . . . . . . . . 76

1
2 CONTENTS

4 Partial Differentiation 77
4.1 Learning Objectives . . . . . . . . . . . . . . . . . . . . . . . . . . . . . . 77
4.2 Introduction . . . . . . . . . . . . . . . . . . . . . . . . . . . . . . . . . . 78
4.3 Function of Two Variables . . . . . . . . . . . . . . . . . . . . . . . . . . 78
4.4 Partial Derivatives . . . . . . . . . . . . . . . . . . . . . . . . . . . . . . . 79
4.5 Homogeneous Functions . . . . . . . . . . . . . . . . . . . . . . . . . . . 85
4.6 Summary . . . . . . . . . . . . . . . . . . . . . . . . . . . . . . . . . . . 90
4.7 Self-Assessment Exercises . . . . . . . . . . . . . . . . . . . . . . . . . . 92
4.8 Solutions to In-text Exercises . . . . . . . . . . . . . . . . . . . . . . . . . 93
4.9 Suggested Readings . . . . . . . . . . . . . . . . . . . . . . . . . . . . . . 94

5 Mean Value theorem 96


5.1 Learning Objectives . . . . . . . . . . . . . . . . . . . . . . . . . . . . . . 96
5.2 Introduction . . . . . . . . . . . . . . . . . . . . . . . . . . . . . . . . . . 97
5.3 Rolle’s Theorem . . . . . . . . . . . . . . . . . . . . . . . . . . . . . . . . 97
5.4 Lagrange’s Mean Value Theorem . . . . . . . . . . . . . . . . . . . . . . . 103
5.5 Applications of Mean Value Theorem to monotonic functions and inequalities110
5.6 Summary . . . . . . . . . . . . . . . . . . . . . . . . . . . . . . . . . . . 117
5.7 Self-Assessment Exercises . . . . . . . . . . . . . . . . . . . . . . . . . . 118
5.8 Solutions to In-text Exercises . . . . . . . . . . . . . . . . . . . . . . . . . 119
5.9 Suggested Readings . . . . . . . . . . . . . . . . . . . . . . . . . . . . . . 120

6 Extremum and Convergence of Series 121


6.1 Learning Objectives . . . . . . . . . . . . . . . . . . . . . . . . . . . . . . 121
6.2 Introduction . . . . . . . . . . . . . . . . . . . . . . . . . . . . . . . . . . 122
6.3 Extremum of a Function . . . . . . . . . . . . . . . . . . . . . . . . . . . 122
6.4 Sequences . . . . . . . . . . . . . . . . . . . . . . . . . . . . . . . . . . . 132
6.5 Infinite Series . . . . . . . . . . . . . . . . . . . . . . . . . . . . . . . . . 139
6.6 Convergence and Divergence of an Infinite Series . . . . . . . . . . . . . . 141
6.7 Summary . . . . . . . . . . . . . . . . . . . . . . . . . . . . . . . . . . . 145
6.8 Self-Assessment Exercises . . . . . . . . . . . . . . . . . . . . . . . . . . 146
6.9 Solutions to In-text Exercises . . . . . . . . . . . . . . . . . . . . . . . . . 147
6.10 Suggested Readings . . . . . . . . . . . . . . . . . . . . . . . . . . . . . . 149

7 Indeterminate Forms 150


7.1 Learning Objectives . . . . . . . . . . . . . . . . . . . . . . . . . . . . . . 150
7.2 Introduction . . . . . . . . . . . . . . . . . . . . . . . . . . . . . . . . . . 151
7.3 Indeterminate Forms of Limits . . . . . . . . . . . . . . . . . . . . . . . . 151
7.4 Indeterminate Form 00 . . . . . . . . . . . . . . . . . . . . . . . . . . . . . 152
7.5 Indeterminate Form ∞ ∞
. . . . . . . . . . . . . . . . . . . . . . . . . . . . 155
7.6 Indeterminate Form ∞ − ∞ . . . . . . . . . . . . . . . . . . . . . . . . . 157
7.7 Indeterminate Form 0 × ∞ . . . . . . . . . . . . . . . . . . . . . . . . . . 158
7.8 Indeterminate Forms 1∞ , ∞0 and 00 . . . . . . . . . . . . . . . . . . . . . 160
7.9 Summary . . . . . . . . . . . . . . . . . . . . . . . . . . . . . . . . . . . 164
7.10 Self-Assessment Exercises . . . . . . . . . . . . . . . . . . . . . . . . . . 164
CONTENTS 3

7.11 Suggested Readings . . . . . . . . . . . . . . . . . . . . . . . . . . . . . . 167

8 Taylor’s Series and Maclaurin’s Series 168


8.1 Learning Objectives . . . . . . . . . . . . . . . . . . . . . . . . . . . . . . 168
8.2 Introduction . . . . . . . . . . . . . . . . . . . . . . . . . . . . . . . . . . 169
8.3 Cauchy’s Mean Value Theorem . . . . . . . . . . . . . . . . . . . . . . . . 169
8.4 Taylor’s Theorem . . . . . . . . . . . . . . . . . . . . . . . . . . . . . . . 171
8.5 Maclaurin’s Series Expansion of Some Standard Functions . . . . . . . . . 179
8.6 Summary . . . . . . . . . . . . . . . . . . . . . . . . . . . . . . . . . . . 187
8.7 Self-Assessment Exercises . . . . . . . . . . . . . . . . . . . . . . . . . . 188
8.8 Solutions to In-text Exercises . . . . . . . . . . . . . . . . . . . . . . . . . 188
8.9 Suggested Readings . . . . . . . . . . . . . . . . . . . . . . . . . . . . . . 189

9 Reduction Formulae for Integrals 190


9.1 Learning Objectives . . . . . . . . . . . . . . . . . . . . . . . . . . . . . . 190
9.2 Introduction . . . . . . . . . . . . . . . . . . . . . . . . . . . . . . . . . . 191
9.3 Reduction formulae for and . . . . . . . . . . . . . . . . . . . . . . . . . 193
9.4 Reduction formulae for . . . . . . . . . . . . . . . . . . . . . . . . . . . . 197
9.5 Reduction formulae for . . . . . . . . . . . . . . . . . . . . . . . . . . . . 200
9.6 Application of the reduction formulae . . . . . . . . . . . . . . . . . . . . 203
9.7 Summary . . . . . . . . . . . . . . . . . . . . . . . . . . . . . . . . . . . 205
9.8 Self-Assessment Exercise . . . . . . . . . . . . . . . . . . . . . . . . . . . 206
9.9 Solution to In-text Exercises . . . . . . . . . . . . . . . . . . . . . . . . . 207
9.10 Suggested Readings . . . . . . . . . . . . . . . . . . . . . . . . . . . . . . 213

10 Asymptotes, Concavity and Point of Inflexion 215


10.1 Learning Objectives . . . . . . . . . . . . . . . . . . . . . . . . . . . . . . 215
10.2 Introduction . . . . . . . . . . . . . . . . . . . . . . . . . . . . . . . . . . 216
10.3 Asymptote . . . . . . . . . . . . . . . . . . . . . . . . . . . . . . . . . . . 216
10.4 Concavity of a Curve . . . . . . . . . . . . . . . . . . . . . . . . . . . . . 221
10.5 Point of Inflexion . . . . . . . . . . . . . . . . . . . . . . . . . . . . . . . 223
10.6 Summary . . . . . . . . . . . . . . . . . . . . . . . . . . . . . . . . . . . 225
10.7 Self Assessment Exercise . . . . . . . . . . . . . . . . . . . . . . . . . . . 225
10.8 Solutions to In-text Exercises . . . . . . . . . . . . . . . . . . . . . . . . . 226
10.9 Suggested Readings . . . . . . . . . . . . . . . . . . . . . . . . . . . . . . 228

11 Tracing of Curves 227


11.1 Learning Objectives . . . . . . . . . . . . . . . . . . . . . . . . . . . . . . 227
11.2 Introduction . . . . . . . . . . . . . . . . . . . . . . . . . . . . . . . . . . 227
11.3 Procedure to trace the curve y = f (x) . . . . . . . . . . . . . . . . . . . . 228
11.4 Tracing of a polynomial function y = f (x) . . . . . . . . . . . . . . . . . 228
11.5 Tracing of a Rational function y = f (x) . . . . . . . . . . . . . . . . . . . 230
11.6 Tracing of a function of the form y2 = f (x) . . . . . . . . . . . . . . . . . 232
11.7 Summary . . . . . . . . . . . . . . . . . . . . . . . . . . . . . . . . . . . 238
11.8 Self Assessment Exercise . . . . . . . . . . . . . . . . . . . . . . . . . . . 239
4 CONTENTS

11.9 Suggested Readings . . . . . . . . . . . . . . . . . . . . . . . . . . . . . . 240


Unit-1: Limits, Continuity and
Differentiability

Unit Overview
The independent work of two famous mathematicians, Isaac Newton and Gottfried Leibniz.
in the 17th century, laid foundation to calculus. Differential Calculus and Integral Calcu-
lus are two main parts of Calculus. In this unit, we will discuss the concepts of Limits,
Continuity and Differentiability. It is divided into four lessons.
In Lesson-1, we will discuss the concepts of the limit of a function. Both informal and
formal approach(ϵ − δ approach) are to be used. Limits of various kinds of functions, limits
at infinity and infinite limits will be discussed. Some theorems which help us to evaluate
limits are to be discussed in this lesson.
In Lesson-2, we will discuss two important concepts of Calculus, namely continuity and
differentiability. We will discuss important algebraic properties of these topics together
with their applications. Types of discontinuity and the geometrical interpretation of the
differentiability will also be discussed.
Continuing with the concepts of Lesson-1 and Lesson-2, in Lesson-3, we will discuss
the higher order derivatives of functions. Leibnitz’s Theorem and its uses will be discussed
in this lesson.
In Lesson-4, we will discuss the partial derivatives of the functions of two and three
variables. Homogeneous functions and the Euler’s Theorem for homogeneous functions
will be studied in this lesson.
Various examples, In-text Exercises and Self-Assessment Exercises will be included in
each lesson to boost the confidence of the students.

5
Lesson - 1

Limits Mr. Sandeep Bhatt


Ramlal Anand College
UUniversity of Delhii

Structure
1.1 Learning Objectives 6
1.2 Introduction 7
1.3 Limit (An Informal Approach) 7
1.4 Limit (Formal Approach) 12
1.5 Algebraic Properties of Limits 15
1.6 Infinite Limits and Limits at Infinity 20
1.6.1 Infinite Limits 20
1.6.2 Limits at Infinity 21
1.6.3 Infinite Limits at Infinity 22
1.7 Summary 26
1.8 Self-Assessment Exercises 28
1.9 Solutions to In-text Exercises 29
1.10 Suggested Readings 30

1.1 Learning Objectives


The learning objectives of this lesson are to:

• understand the concept of the limit of a function.

• study the algebraic properties of limits.

• study the concept of infinite limits and the limits at infinity.

6
1.2. INTRODUCTION 7

1.2 Introduction
The concept of a limit or limiting process is essential for the understanding of calculus. It
has been around for thousands of years. In fact, early mathematicians used the limiting
procedure to obtain better and accurate approximations of the areas enclosed by closed
plane curves. Yet, the formal definition of a limit as we know and understand it today did
not appear until the late 19th century. The idea of limits gives us a method for describing
how the outputs of a function behave as the inputs approaches to some specific value. Limits
are used as real-life approximations to calculate derivatives, which are helpful in finding
the slope of the tangent to a curve at a point, maxima-minima of a function etc. The limit
of a function is a fundamental concept in calculus and analysis concerning the behavior of
that function near a particular point.

1.3 Limit (An Informal Approach)


To understand what do we mean by the limit of a function, let us first see how the function
f (x) = x3 − 2 behaves as the value of the variable x approaches towards 1. We have the
following table:

Value of x Value of f (x) Value of x Value of f (x)


0 -2 2 6
0.5 -1.875 1.5 1.375
0.9 -1.271 1.1 -.669
0.99 -1.0297 1.01 -0.969699
0.999 -1.003 1.001 -0.996997

Table 1.1: Values of f (x) = x3 − 2.

We can see that the value of the function f (x) approaches to -1 as the value of x ap-
proaches to 1 from both sides (left and right). This leads to the following informal definition
of the limit of a function

Definition 1.1 (Informal Definition). A function f (x) is said to have a limit L as x ap-
proaches to a, written as

f (x) → L as x → a or lim f (x) = L


x→a

if the values of f (x) can be made arbitrary close (as close as we like) to L by choosing
values of x sufficiently close to a.

Note. It is to be noted that

1. the number L mentioned above is a finite real number.

2. while talking about the limit lim f (x), it is not necessary for the function f (x) to be
x→a
defined at the point x = a.
8 LESSON - 1. LIMITS

3. the limit defined above is also known as two sided limit because it requires the
value of the function f (x) to approach towards L as x approaches towards a from
left(x < a) and right (x > a). When we talk about the limit of a function at a point,
we talk about the two sided limit.
cos x − 1 cos x − 1
Example 1.1. Consider the limit lim . We note that the function is not
x→0 x x
cos x − 1
defined at x = 0, but the values of f (x) = approach to 0 when the value of x
x
approach to 0, as shown in Table 1.2 and Figure 1.1.

Value of x Value of f (x) Value of x Value of f (x)


-1 0.459698 1 -0.459698
-0.5 0.244835 0.5 -0.244835
-0.25 0.12435 0.25 - 0.12435
-0.1 0.049958 0.1 - 0.049958
-0.01 0.004999 0.01 -0.004999
-0.001 0.000500 0.001 -0.000500
cos x − 1
Table 1.2: Values of f (x) = .
x

cos x − 1
Figure 1.1: Graph of f (x) =
x
 
1
Example 1.2. Let us consider the behavior of f (x) = sin as x → 0. From the Table
 x
1
1.3 and Figure 1.2, we see that as x → 0, the value of sin oscillates rapidly between
x
-1 and 1,
 and
 does not approach to a fixed real number. Therefore, in this case, we say that
1
lim sin does not exist.
x→0 x
1.3. LIMIT (AN INFORMAL APPROACH) 9

Value of x Value of f (x) Value of x Value of f (x)


-1 -0.841471 1 0.841471
-0.1 0.544021 0.1 -0.544021
-0.05 -0.912945 0.05 0.912945
-0.001 -0.826880 0.001 0.826880
-0.0005 -0.930040 0.0005 0.930040
 
1
Table 1.3: Values of f (x) = sin .
x

Figure 1.2: Graph of f (x) = sin d x1




Definition 1.2 (One Sided Limits).

1. A function f (x) is said to have a limit L as x approaches a from the right, written
lim f (x) = L, if the value of f (x) can be made sufficiently close (as close as we
x→a+
like) to L by choosing the value of x sufficiently close to a (x > a). This limit is also
known as the right hand limit (R.H.L.).

2. A function f (x) is said to have a limit L as x approaches a from the left, written
lim f (x) = L, if the value of f (x) can be made sufficiently close (as close as we
x→a−
like) to L by choosing the value of x sufficiently close to a (x < a). This limit is also
known as the left hand limit (L.H.L.).
10 LESSON - 1. LIMITS

Example 1.3. Consider the function


(
x2 + 2, x<0
f (x) = . (1.1)
x − 2, x≥0

We have the following table for the values of f (x) near x = 2 :

Value of x Value of f (x) Value of x Value of f (x)


-1 3 1 -1
-0.5 2.25 0.5 -1.5
-0.25 2.0625 0.25 -1.75
-0.1 2.01 0.1 -1.9
-0.01 2.0001 0.01 -1.99
-0.001 2.000001 0.001 -1.999

Table 1.4: Values of f (x).

Figure 1.3: Graph of f (x).

From the Table 1.4, we observe that the value of the function f (x) approaches towards
2 as the value of x approaches towards 0 from the left (x < 0). On the other hand, if the
value of x approaches towards 0 from the right (x > 0), the value of the function f (x)
approaches towards -2. Thus we write

lim f (x) = −2 and lim f (x) = 2.


x→0+ x→0−
1.3. LIMIT (AN INFORMAL APPROACH) 11

Theorem 1.1 (Necessary and Sufficient Condition). The limit of a function f (x) exists at
a point x = a if and only if both the two sided limits of f (x) exist at x = a and they are
equal. That is;

lim f (x) = L if and only if lim f (x) = L = lim− f (x).


x→a x→a+ x→a

Theorem 1.2 (Non Existence of Limit). The limit of a function f (x) at x = a does not
exist if
1. Either lim+ f (x) or lim− f (x) or both do not exist, or
x→a x→a

2. Both lim+ f (x) and lim− f (x) exist, but they are not equal.
x→a x→a

Definition 1.3. (Absolute Value Function) Let x ∈ R. Then the absolute value of x is
defined by the function (
−x, x<0
f (x) = .
x, x≥0
and is denoted by |x|.
For example,
| − 10| = 10 and |10| = 10.
Note. We note that
1. |0| = 0.

2. |x| = | − x| > 0, for all non-zero x.

Figure 1.4: Graph of f (x) = |x|.


12 LESSON - 1. LIMITS

|x|
Example 1.4. Consider the function f (x) = lim , x ̸= 0. We have
x→0 x

|x| −x
L.H.L. = lim− = lim = lim (−1) = −1,
x→0 x x→0 x x→0
|x| x
and R.H.L. = lim+ = lim = lim (1) = 1.
x→0 x x→0 x x→0

|x|
Since, L.H.L. ̸= R.H.L. Therefore, lim does not exists.
x→0 x
Definition 1.4. (Greatest Integer Function) Let x ∈ R. Then the greatest integer function
[x] is defined as the largest integer less than or equal to x.
For example,

[2.4] = 2, [2] = 2, [1.9] = 1, [−1] = −1 and [−1.3] = −2.

Example 1.5. Consider the function f (x) = [x], −2 ≤ x ≤ 2. We have




 −2, −2 ≤ x < −1

−1, −1 ≤ x < 0



[x] = 0, 0≤x<1

1, 1≤x<2





2, x=2

Now,

L.H.L. = lim− [x] = lim (−1) = −1,


x→0 x→0

and R.H.L. = lim+ [x] = lim 0 = 0.


x→0 x→0

Since, L.H.L. ̸= R.H.L. Therefore, lim [x] does not exist. Similarly, we can show that the
x→0
limit of f (x) = [x] does not exist at x = −1 and x = 1.

In general, limit of f (x) = [x] defined on R does not exit at all integer values i.e. at
x = 0, ±1, ±2, ±3, . . ..

1.4 Limit (Formal Approach)


Definition 1.5 ( ϵ − δ Approach). Let f (x) be a real valued function defined in a set
containing a, except possibly at a. Then f (x) is said to approach to a real number L as x
approaches to a, if for every real number ϵ > 0, there exists a real number δ > 0, such that

|f (x) − L| < ϵ when 0 < |x − a| < δ.


1.4. LIMIT (FORMAL APPROACH) 13

Note. We note the following:


1. The value of δ depends on ϵ and a.
2. For a given ϵ > 0, the choice of δ is not unique. Once we find one value of δ, any
value that is less than δ also works.
Example 1.6. We will show that for a ∈ R, lim c = c, where c is a real constant.
x→a
Here f (x) = c, L = c.
Let ϵ > 0 be an arbitrary real number. Since
|f (x) − L| = |c − c| = 0.
Let us choose any real number δ > 0. So, if
0 < |x − a| < δ,
we have
|f (x) − c| = 0 < ϵ.
Since ϵ > 0 is an arbitrary number. Therefore, by ϵ − δ definition, we have
lim c = c.
x→a

Example 1.7. We will show that for a ∈ R, lim x = a. Here f (x) = x, L = a.


x→a
Let ϵ > 0 be an arbitrary real number. Since
|f (x) − L| = |x − a|.
Let us choose δ = ϵ. Then δ > 0. So, if
0 < |x − a| < δ,
we have
|f (x) − L| = |x − a| < δ = ϵ.
Since ϵ > 0 is an arbitrary number. Therefore, by ϵ − δ definition, we have
lim x = a.
x→a
14 LESSON - 1. LIMITS

Example 1.8. Show that for a ∈ R, lim (3x − 5) = 3a − 5.


x→a

Solution. Let f (x) = (3x − 5) and ϵ > 0 be given. Then

|f (x) − (3a − 5)| = |(3x − 5) − (3a − 5)|


= |3(x − a)|
= 3|x − a|
ϵ
< ϵ when |x − a| < .
3
ϵ
Therefore, by taking δ = , we get
3
|f (x) − (3a − 5)| < ϵ when |x − a| < δ.

Hence, by ϵ − δ definition, we have

lim f (x) = lim (3x − 5) = 3a − 5.


x→a x→a

Example 1.9. Show that


lim (x2 − 3) = 1.
x→2

Solution. Let f (x) = x2 − 3 and ϵ > 0 be given. Then

|f (x) − L| = |x2 − 3 − (1)|


= |x2 − 4|
= |x − 2||x + 2|
= |x − 2| (|x| + |2|) [Using Triangle Inequality of R] (1.2)

Since, we have to find limit of f (x) as x tends to 2, we take values of x near 2. Let us take
x such that |x − 2| < 1 or 1 < x < 3. Then

3 < |x| + |2| < 5. (1.3)

Therefore, from (1.2) and (1.3), we get

|f (x) − L| ≤ |x − 2|(|x| + |2|)


< 5|x − 2| when |x − 2| < 1
ϵ
< ϵ when |x − 2| < as well as |x − 1| < 1
5
nϵ o
Therefore, by taking δ = min , 1 , we get
5
|f (x) − 1| < ϵ when |x − 2| < δ

Hence, by ϵ − δ definition, we have

lim x2 − 3 = 1.
x→2
1.5. ALGEBRAIC PROPERTIES OF LIMITS 15

In-text Exercise 1.1. Using ϵ − δ definition, prove the following:

1. lim 2x + 3 = 9
x→3

2. lim 3x2 + 5 = 17
x→2

1 1
3. lim =
x→2 x 2

1.5 Algebraic Properties of Limits


Theorem 1.3. Let
lim f (x) = L and lim g(x) = M,
x→a x→a

then

1. lim (f + g)(x) = lim f (x) + lim g(x) = L + M.


x→a x→a x→a

2. lim (f − g)(x) = lim f (x) − lim g(x) = L − M.


x→a x→a x→a

3. lim (k · f )(x) = k · lim f (x) = k · L, where k is a real constant.


x→a x→a

4. lim (f · g)(x) = lim f (x) · lim g(x) = L · M.


x→a x→a x→a

 
f lim f (x) L
x→a
5. lim (x) = = provided M ̸= 0.
x→a g lim g(x) M
x→a

Some Useful Limits:

1. lim xn = an for n ∈ N and for all a ∈ R.


x→a

2. lim sin(x) = sin(a), lim cos(x) = cos(a) for all a ∈ R.


x→a x→a

3. lim ex = ea for all a ∈ R.


x→a

4. lim ln(x) = ln(a) for all a > 0, where ln(x) is the natural logarithm.
x→a

sin x
5. lim = 1.
x→0 x

6. lim (1 + x)1/x = e.
x→0

Example 1.10.
16 LESSON - 1. LIMITS

(i) Let p(x) = p0 + p1 x + · · · + pn xn be a polynomial of degree n where p0 , p1 , . . . , pn


are constants in R and pn ̸= 0. Then, by using Theorem 1.3, we get

lim p(x) = lim p0 + lim p1 x + · · · + lim pn xn


x→a x→a x→a x→a
= p0 + p1 lim x + · · · + pn lim xn
x→a x→a
n
= p0 + p1 a + · · · + pn a

That is,

lim p(x) = p(a)


x→a

(ii) lim (x sin x + 3 ln x) = lim (x sin x) + lim (3 ln x) [Using Theorem 1.3(1)]


x→1 x→1 x→1
= lim x · lim sin x + 3 · lim ln x [Using Theorem 1.3(3,4)]
x→1 x→1 x→1
= 1 · sin 1 + 3 · ln 1
= sin 1 + 3 · 0
= sin 1 + 0 = sin 1

x3 + 5 lim (x3 + 5)
x→2
(iii) lim = [Using Theorem 1.3(5)]
x→2 x2 − 6 lim (x2 − 6)
x→2
lim x3 + lim 5
x→2 x→2
= [Using Theorem 1.3(1)]
x2
lim − lim 6
x→2 x→2
 3
lim x + 5
= x→2 2 [Using Theorem 1.3(4)]
lim x − 6
x→2
3
2 +5
=
22 − 6
8+5 13 −13
= = =
4−6 −2 2
sin 4x sin 4x
(iv) lim = 4 · lim [x → 0 =⇒ 4x → 0]
x→0 x 4x→0 4x
=4·1
=4

tan x sin x 1
(v) lim = lim ·
x→0 x x→0 cos x x
sin x 1
= lim ·
x→0 x cos x
1
=1· =1
1
1.5. ALGEBRAIC PROPERTIES OF LIMITS 17
h i2
(vi) lim (1 + 2x)1/x = lim (1 + 2x)1/2x
x→0 2x→0
h i2
= lim (1 + y)1/y
y→0
 2
1/y
= lim (1 + y)
y→0

= e2

In-text Exercise 1.2. Discuss the existence of the following limits:


(
x4 − x − 1, x≤4
1. lim f (x), where f (x) =
x→4 3x − 5, x>4

2|x|
2. lim
x→0 |x| + 5

1 − cos x
3. lim
x→0 x2
2
4. lim
x→0 (1 + x)3/x

Example 1.11.

x + 2|x|
(i) Consider the function f (x) = . We have
−5x + |x|

x + 2|x| x − 2x
lim− = lim
x→0 −5x + |x| x→0 −5x − x
−x
= lim
x→0 −6x
1 1
= lim = .
x→0 6 6

Also,

x + 2|x| x + 2x
lim+ = lim
x→0 −5x + |x| x→0 −5x + x
3x
= lim
x→0 −4x
3 −3
= lim = .
x→0 −4 4

x + 2|x| x + 2|x| x + 2|x|


Since, lim− ̸= lim+ . Therefore, lim does not exist.
x→0 −5x + |x| x→0 −5x + |x| x→0 −5x + |x|
18 LESSON - 1. LIMITS

p(x) x2 + x − 2
(ii) Let = 2 . We note that p(1) = 0 = q(1). We can write
q(x) x + 2x − 3
p(x) x2 + x − 2
= lim 2
q(x) x→1 x + 2x − 3
(x − 1)(x + 2)
= lim
x→1 (x − 1)(x + 3)

(x + 2)
= lim
x→1 (x + 3)
1+2
= [Using Theorem 1.3(5)]
1+3
3
= .
4

x3 + 1 (x + 1)(x2 − x + 1)
(iii) lim = lim
x→−1 x + 1 x→−1 (x + 1)
2
= lim (x − x + 1)
x→1
=1+1+1 [Using Theorem 1.3(1,4)]
= 3.
Theorem 1.4 (Sandwich Theorem). Let X be a subset of R and a ∈ R. Let f (x), g(x)
and h(x) be functions defined on X, except possibly at x = a, such that
1. f (x) ≤ g(x) ≤ h(x) ∀x ∈ X

2. lim f (x) = L = lim h(x)


x→a x→a

Then,
lim g(x) = L.
x→a

Note. The Sandwich Theorem is also known as the Squeeze Theorem.


Example 1.12. Use the Sandwich Theorem to evaluate
 
1
lim x sin
x→0 x
 
1
Solution. Let g(x) = x sin , x ̸= 0. We have
x
 
1
− 1 ≤ sin ≤1
x
 
1
=⇒ − x ≤ x sin ≤ x for x > 0
x
 
1
and − x ≥ x sin ≥ x for x < 0.
x
1.5. ALGEBRAIC PROPERTIES OF LIMITS 19

In both the cases as lim x = lim (−x) = 0, therefore, by using the Sandwich Theorem, we
x→0 x→0
get  
1
lim x sin =0
x→0 x
Example 1.13. Using the inequality
 π
sin x < x < tan x, x ∈ 0,
2
and the Sandwich Theorem, prove that
sin x
lim = 1.
x→0 x
Solution. We have
 π
sin x < x < tan x, x ∈ 0,
2
sin x  π
=⇒ sin x < x < , x ∈ 0,
cos x 2
x 1  π
=⇒ 1 < < , x ∈ 0,
sin x cos x 2
sin x  π
=⇒ 1 > > cos x, x ∈ 0, (1.4)
x 2
 π   π
Also, if x ∈ − , 0 then −x ∈ 0, . Therefore, from (1.4)
2 2
sin(−x)  π 
1> > cos(−x), x ∈ − , 0
−x 2
sin x  π 
=⇒ 1 > > cos x, x ∈ − , 0 . (1.5)
x 2
Therefore in both cases (when x > 0 and when x < 0), we have
sin x
1> > cos x
x
and
lim cos x = 1 = lim (1)
x→0 x→0
Hence, by using the Sandwich Theorem, we get
sin x
lim = 1.
x→0 x

In-text Exercise 1.3. Find the following limits:


x2 − x
1. lim .
x→1 x2 − 1

x2 − 1
2. lim 2 .
x→−1 x − 4x − 5

3. Use Sandwich Theorem to prove that lim sin x = 0.


x→0
20 LESSON - 1. LIMITS

1.6 Infinite Limits and Limits at Infinity

1.6.1 Infinite Limits

1. If the values of a function f (x) gets larger and larger (larger than any given K > 0)
as the value of x approaches to a, then we say that the function f (x) tends to ∞ as x
tends to a and represent it by lim f (x) = ∞.
x→a
1
For example, lim 2 = ∞ (see Figure 1.5).
x→0 x

2. If the value of a function f (x) gets smaller and smaller (smaller than any given K <
0) as the value of x approaches to a, then we say that the function f (x) tends to −∞
as x tends to a and represent it by lim f (x) = −∞.
x→a
−1
For example, lim 2 = −∞ (see Figure 1.6).
x→0 x

1
Figure 1.5: Graph of f (x) = . −1
x2 Figure 1.6: Graph of f (x) = .
x2

1
3. One sided limits can be defined similarly as in section 1.3. For example, lim+ =∞
x→0 x
1
and lim− = −∞ (see Figure 1.7).
x→0 x
1.6. INFINITE LIMITS AND LIMITS AT INFINITY 21

1
Figure 1.7: Graph of f (x) = .
x

4. In the above cases 1-3, we say that lim f (x) does not exist, as −∞ and ∞ are not
x→a
fixed real numbers.

1.6.2 Limits at Infinity

1. If the values of a function f (x) gets very close (as close as we like) to L as the values
of x becomes larger and larger, then we say that the function f (x) tends to L as x
tends to ∞ and represent it by lim f (x) = L.
x→∞
1
For example, lim = 0 (see Figure 1.8).
x→∞ x

2. If the values of a function f (x) gets very close (as close as we like) to L as the values
of x becomes smaller and smaller, then we say that the function f (x) tends to L as x
tends to −∞ and represent it by lim f (x) = L.
x→−∞
−1
For example, lim = 0 (see Figure 1.9).
x→−∞ x
22 LESSON - 1. LIMITS

−1
Figure 1.8: Graph of f (x) = x1 . Figure 1.9: Graph of f (x) = .
x

1.6.3 Infinite Limits at Infinity


If the values of a function f (x) becomes infinitely large for infinitely large/small values of
x, then we say that
lim f (x) = ∞ or lim f (x) = ∞.
x→∞ x→−∞

Similarly, if the values of a function f (x) becomes infinitely small(negative value) for
infinitely small/large values of x, then we say that

lim f (x) = −∞ or lim f (x) = −∞.


x→−∞ x→∞

For example, lim x3 = ∞ and lim x3 = −∞ (see Figure 1.10).


x→∞ x→−∞

Figure 1.10: Graph of f (x) = x3


1.6. INFINITE LIMITS AND LIMITS AT INFINITY 23

Some Useful Limits:


1 1 1
1. lim n
= lim+ n
= lim− = ∞, when n is even.
x→a (x − a) x→a (x − a) x→a (x − a)n

1 1
2. lim+ = ∞ and lim = −∞, when n is odd.
x→a (x − a)n x→a− (x − a)n

3. lim xn = ∞ when n is even.


x→∞
(
∞, when n is even
4. lim xn = .
x→−∞ −∞, when n is odd
5. lim+ ln x = −∞
x→0

6. lim ln x = ∞
x→∞

7. lim ex = ∞
x→∞

8. lim ex = 0
x→−∞

1
9. If lim f (x) = ±∞ =⇒ lim = 0, f (x) ̸= 0.
x→a x→a f (x)
 x
1
10. lim 1+ =e
x→∞ x
sin x sin x
11. lim = lim = 0.
x→∞ x x→−∞ x

Note. Results similar to those in Theorem 1.3, can be established for the limits at infinity.
Example 1.14. Evaluate
3 − e2/x
lim .
x→0 5 + e2/x

Solution. We have
2
x → 0− =⇒ → −∞
x
=⇒ e2/x → 0.
Therefore,

3 − e2/x lim− (3 − e2/x )


L.H.L. = lim− = x→0
x→0 5 + e2/x lim− (5 + e2/x )
x→0
3 − lim− e2/x
x→0
=
5 + lim− e2/x
x→0
3−0 3
= = .
5+0 5
24 LESSON - 1. LIMITS

Also,

2
x → 0+ =⇒ →∞
x
=⇒ e2/x → ∞
1
=⇒ 2/x → 0
e

Therefore,

 
3
lim −1
3 − e2/x x→0+ e2/x
R.H.L. = lim+ =  
x→0 5 + e2/x 5
lim +1
x→0+ e2/x
 
3
lim −1
x→0+ e2/x
=  
5
lim +1
x→0+ e2/x
3·0−1
= = −1.
5·0+1

3 − e2/x 3 − e2/x 3 − e2/x


Since L.H.L.̸= R.H.L., i.e. lim− ̸= lim+ . Therefore, lim does
x→0 5 + e2/x x→0 5 + e2/x x→0 5 + e2/x
not exists.

Note. While computing the limits of rational functions as x → ±∞, it is beneficial to


divide the function by highest power of x that appears in the denominator. It is illustrated
in the following example.

Example 1.15. Evaluate following limits:

5x − 7
(i) lim
x→∞ x + 21

45x2 − 1
(ii) lim
x→∞ 7x4 − 11x

x3 + 5x2 − 1
(iii) lim
x→∞ 7x2 + 21
1.6. INFINITE LIMITS AND LIMITS AT INFINITY 25

7
5x − 7 5−
Solution. (i) lim = lim x
x→∞ x + 21 x→∞ 21
1+
 x 
7
lim 5 −
x→∞ x
=  
21
lim 1 +
x→∞ x
7
lim 5 − lim
x→∞ x
= x→∞
21
lim 1 + lim
x→∞ x→∞ x
 
5−0 1
= x → ∞ =⇒ → 0
1+0 x
= 5.

45 1
2
45x − 1 2

(ii) lim = lim x x4
x→∞ 7x4 − 11x x→∞ 11
7− 3
 x 
45 1
lim −
x→∞ x2 x4
=  
11
lim 7 − 3
x→∞ x
45 1
lim 2 − lim 4
= x→∞ x x→∞ x
11
lim 7 − lim 3
x→∞
x→∞ x 
0−0 1
= x → ∞ =⇒ n → 0 for n > 0
7−0 x
= 0.

 
3 5 1
x 1+ 2 − 3
x3 + 5x2 − 1 x x
(iii) lim 2
= lim  
x→∞ 7x + 21 x→∞ 21
x2 7 + 2
x
5 1
 
1+ 2 − 3
= lim x 
 x x 
x→∞ 21 
7+ 2
x
=∞
26 LESSON - 1. LIMITS

√ √ √ !
  x2 + 5 + x
Example 1.16. lim 2
x + 5 − x = lim 2
x +5−x √
x→∞ x→∞ x2 + 5 + x

( x2 + 5)2 − x2
= lim √
x→∞ x2 + 5 + x
x + 5 − x2
2
= lim √
x→∞ x2 + 5 + x
5
= lim √
x→∞ x2 + 5 + x
5
= lim r x
x→∞ 5
1+ 2 +1
x
5
lim
= r x→∞ x
5
1 + lim 2 + 1
x→∞ x
0
=√ = 0.
1+0+1
In-text Exercise 1.4. Find the following limits:
x+2
1. lim+ .
x→2 x2 − 4
√ 
2. lim x4 + 5x2 − x2 .
x→∞

√ 
3. lim x4 + 3x − x2 .
x→∞

1.7 Summary
In this lesson we have discussed the following points:

1. A function f (x) is said to have a limit L as x approaches a, written lim f (x) = L


x→a
if the values of f (x) can be made close (as close as we like) to L by choosing the
values of x sufficiently close to a. It is expressed by lim f (x) = L.
x→a

2. A function f (x) is said to have a limit L as x approaches a from the right, written
lim f (x) = L, if the value of f (x) can be made close (as close as we like) to L by
x→a+
choosing values of x sufficiently close to a (x > a). This limit is also known as the
right hand limit (R.H.L.).

3. A function f (x) is said to have a limit L as x approaches a from the left, written
lim f (x) = L if the value of f (x) can be made close (as close as we like) to L by
x→a−
1.7. SUMMARY 27

choosing values of x sufficiently close to a (x < a). This limit is also known as the
left hand limit (L.H.L.).

4. Necessary and sufficient condition: The limit of a function f (x) exists at a point
x = a if and only if both the two sided limits of f (x) exist at x = a and they are
equal. That is;

lim f (x) = L if and only if lim f (x) = L = lim− f (x).


x→a x→a+ x→a

5. Non existence of limit: The limit of a function f (x) at x = a does not exist if

(i) Either lim+ f (x) or lim+ f (x) or both do not exist, or


x→a x→a

(ii) Both lim+ f (x) and lim− f (x) exist, but they are not equal.
x→a x→a

6. Formal Definition of Limit (ϵ − δ Approach): Let f (x) be a real valued function


defined in a set containing a, except possibly at a. Then f (x) is said to approach to
a real number L as x approaches to a, if for every real number ϵ > 0, there exists a
real number δ > 0, such that

|f (x) − L| < ϵ when 0 < |x − a| < δ.

7. Algebra of limits:

(i) Limit of the sum/difference/product of functions f and g is equal to the sum/


difference/product of limits of f and g.
(ii) Limit of the quotient of functions f and g is equal to the quotient of limits of f
and g provided the limit of the divisor is not-zero.

8. Sandwich Theorem: Let X be a subset of R and a ∈ R. Let f (x), g(x) and h(x) be
functions defined on X, except possibly at x = a, such that

(i) f (x) ≤ g(x) ≤ h(x) ∀x ∈ X


(ii) lim f (x) = L = lim h(x)
x→a x→a

Then,
lim g(x) = L.
x→a

9. Infinite limits:

(i) If the values of a function f (x) gets larger and larger (larger than any given
K > 0) as the value of x approaches to a, then we say that the function f (x)
tends to ∞ as x tends to a and represent it by lim f (x) = ∞.
x→a

(ii) If the value of a function f (x) gets smaller and smaller (smaller than any given
K < 0) as the value of x approaches to a, then we say that the function f (x)
tends to −∞ as x tends to a and represent it by lim f (x) = −∞.
x→a
28 LESSON - 1. LIMITS

10. Limits at Infinity:


(i) If the values of a function f (x) gets very close (as close as we like) to L as the
values of x becomes larger and larger, then we say that the function f (x) tends
to L as x tends to ∞ and represent it by lim f (x) = L.
x→∞
(ii) If the values of a function f (x) gets very close (as close as we like) to L as
the value of x becomes smaller and smaller, then we say that the function f (x)
tends to L as x tends to −∞ and represent it by lim f (x) = L.
x→−∞

11. Infinite limits at Infinity:


(i) If the value of a function f (x) becomes infinitely large for infinitely large/small
values of x, then we say that
lim f (x) = ∞ or lim f (x) = ∞.
x→∞ x→−∞

(ii) If the value of a function f (x) becomes infinitely small(negative value) for in-
finitely small/large values of x, then we say that
lim f (x) = −∞ or lim f (x) = −∞.
x→−∞ x→∞

1.8 Self-Assessment Exercises


1. Using the ϵ − δ definition, prove the following:
(i) lim (7x − 3) = 1/2
x→1/2

(ii) lim (x2 + 5) = 9


x→−2

|x − 5|
2. Show that lim does not exists.
x→5 x − 5
3 . Discuss the existence of the limit of the function at x = 0
 e − e−3/x
 3/x
, x ̸= 0
f (x) = e3/x + e−3/x .
0, x=0

(x + 3)(x + a)
4. Find the values of a for which lim exists.
x→2 x2 − 4
5. Find the value of a for which lim f (x) exists, where
x→3
(
4x − 5, x≤3
f (x) = .
x + 2a, x>3

x2 + 2x
6. Discuss the existence of lim .
x→0 |x|
7. Find the following limits
1.9. SOLUTIONS TO IN-TEXT EXERCISES 29

x4 − 3x2 + 5
(i) lim ,
x→∞ 5x4 − 3x + 9

x2 + 2
(ii) lim 2 ,
x→∞ x − 9x + 1

(iii) lim ( x8 − 4x3 − x4 )
x→∞

(iv) lim ( x8 − 4x4 + 2x − x4 ).
x→∞
 
|x − 2|
8. Find the limit lim + [x] .
x→2 x−2
x(x + 2)
9. Show that lim does not exist.
x→0 |x|
10. By using Sandwich Theorem, prove that

lim cos x = 1.
x→0

11. Find the following limits:


sin 2x
(i) lim
x→∞ sin 5x
tan 4x
(ii) lim
x→∞ tan 3x
 2
sin 2x
(iii) lim
x→∞ tan 3x

1.9 Solutions to In-text Exercises


Exercise 1.2
1. Limit does not exist.

2. 0.
1
3.
2
2
4.
e3
Exercise 1.3
1
1. .
2
1
2. .
3
Exercise 1.4
30 LESSON - 1. LIMITS

1. Does not exist.


5
2. .
2
3. 0.

1.10 Suggested Readings


1. Narayan, S. & Mittal, P. K.(2019). Differential Calculus. S. Chand Publishing.
2. Anton, H., Bivens, I. C., & Davis, S. (2015). Calculus: Early Transcendentals. John
Wiley & Sons.
3. Singh, J.P. (2017). Calculus, 2nd Edition. Ane Books Pvt Ltd.
Lesson - 2

Continuity and Differentiability


Mr. Sandeep Bhatt
Ramlal Anand College
UUniversity of Delhi

Structure
2.1 Learning Objectives 31
2.2 Introduction 32
2.3 Continuity 32
2.3.1 Types of Discontinuity 35
2.4 Properties of Continuous Functions 38
2.5 Differentiability 40
2.5.1 Geometric Interpretation of a Derivative 44
2.5.2 Derivative as the Rate of Change 45
2.6 Some Theorems on Derivatives 46
2.6.1 Derivatives of the Inverse of an Invertible Function 48
2.6.2 Application of Derivative 50
2.7 Summary 51
2.8 Self-Assessment Exercises 54
2.9 Solutions to In-text Exercises 55
2.10 Suggested Readings 56

2.1 Learning Objectives


The learning objectives of this lesson are to:
• learn the concepts of continuity and discontinuity of functions and their property.
• differentiate between various types of discontinuity of a function.
• learn the differentiability/derivability of functions.
• understand the geometrical interpretation of differentiation.
• calculate the derivatives of the functions of various types.

31
32 LESSON - 2. CONTINUITY AND DIFFERENTIABILITY

2.2 Introduction
To understand the concept of continuity, let us first consider the scenario where two peo-
ple, A and B, are playing catch. When the ball leaves the hand of person A and reaches
person B, we see that the ball follows an unbroken curve. It can not suddenly disappear
from one point and reappear at some other point. In mathematics, these kinds of unbroken
curves are known as continuous curves, and this type of curve property is called continuity.
Therefore, in mathematics, a continuous function is a function such that a continuous vari-
ation (that is, a change without a jump) of the variable induces a continuous variation of
the value of the function. This means that there are no sudden changes in the value (known
as discontinuity).
In mathematics, the derivative of a real-valued function measures the rate of change of
the function value (output value) corresponding to a change in its argument (input value).
Differentiation is a fundamental tool of calculus. For example, the derivative of the dis-
placement vector of a moving object with respect to time is the object’s velocity: this
measures how instantly the object’s position changes when time advances. The deriva-
tive of a function of a single variable at a chosen input value, when it exists, represents
geometrically the slope of the tangent line to the function’s graph at that point.

2.3 Continuity
Definition 2.1. A function f (x) is said to be continuous at a point x = a if the following
conditions are satisfied:

(i) f (x) is defined at x = a.

(ii) lim f (x) exists.


x→a

(iii) lim f (x) = f (a).


x→a

Definition 2.2. A function f (x) is said to be continuous on X ⊂ R, if f (x) is continuous


at each x ∈ X.

Definition 2.3. A function f (x) is said to be discontinuous at x = a if it is not continuous


at x = a, i.e., if any one of the three conditions mentioned in Definition 2.1 are not satisfied.
The point x = a is called a point of discontinuity of f (x).

Example 2.1.

(i) Let p(x), x ∈ R, be a polynomial function. Then p(x) is a continuous function. For
example, let p(x) = 4x2 + x − 5. Since

lim (4x2 + x − 5) = 0 = p(1).


x→1

Therefore, p(x) is continuous at x = 1.


2.3. CONTINUITY 33

(ii) Exponential functions are continuous on R. Since,


lim ex = e2 , lim e−x = e−2 , lim 3x = 32 = 9.
x→2 x→2 x→2

Therefore, ex , e−x and 3x are continuous at x = 2.


(iii) Consider the logarithmic function f (x) = log x, x > 0, x ∈ R. For a > 0, since we
have
lim f (x) = lim log x
x→a x→a
= log a
= f (a)
Therefore, f (x) = log x is continuous on (0, ∞). Similarly, ln x, log2 x are also
continuous on (0, ∞).
(iv) Consider the trigonometric function f (x) = sin x, x ∈ R. For a ∈ R, since we have
lim f (x) = lim sin x
x→a x→a
= sin a
= f (a)
Therefore, f (x) = sin x is continuous on R. Similarly, cos x, tan x, cot x, sec x and
cosec x are also continuous in their respective domains.
Example 2.2. Let (
5x3 − x, x≤1
f (x) = .
3x + 1, x>1
We have f (1) = 4, i.e., f (x) is defined at x = 1. Now,
L.H.L. = lim− f (x) = lim 5x3 − x = 5 · 13 − 1 = 5 − 1 = 4,

x→1 x→1

and R.H.L. = lim+ f (x) = lim (3x + 1) = 3 · 1 + 1 = 3 + 1 = 4.


x→1 x→1

Therefore, L.H.L.=R.H.L. That is, lim f (x) exists and it is 4 = f (1). Hence, by Definition
x→1
2.1, f (x) is continuous at x = 1.
Example 2.3. Consider the function
(
x2 − x, x≤2
f (x) = .
2x, x>2
We have f (2) = 2, i.e., f (x) is defined at x = 2. Now,
L.H.L. = lim− f (x) = lim x2 − x) = 22 − 2 = 4 − 2 = 2,

x→2 x→2

and R.H.L. = lim+ f (x) = lim (2x) = 2 · 2 = 4.


x→2 x→2

Therefore, L.H.L.̸=R.H.L. That is, lim f (x) does not exists. Hence, by Definition 2.3, f (x)
x→2
is not continuous at x = 2.
34 LESSON - 2. CONTINUITY AND DIFFERENTIABILITY

In-text Exercise 2.1. Examine the continuity of the following functions at x = 1:

x2 − 3x + 2
1. f (x) = ,
x−1
 2
 x − 3x + 2
, x ̸= 1
2. f (x) = x−1 ,
1, x=1

 2
 x − 3x + 2
, x ̸= 1
3. f (x) = x−1 .
−1, x=1

Example 2.4. Consider the function f defined by




 2x + 5, x < −2

1, x = −2
f (x) =


 x + 3, −2 < x ≤ 0

sin x, 0<x

Let a be any real number. We will find the values of a at which f (x) is continuous. Consider
the following cases:
Case (i) a < −2 : In this case, f (x) = 2x − 5. Since 2x − 5 is a polynomial, therefore
f (x) is continuous at x = a.
Case (ii) a = −2 : In this case,

lim f (x) = lim − (2x + 5) = 2(−2) + 5 = 1 = f (−2)


x→−2− x→−2

lim f (x) = lim + (x + 3) = −2 + 3 = 1 = f (−2).


x→−2+ x→−2

Therefore, lim f (x) = 1 = f (−2). Hence, f (x) is continuous at a = −2.


x→−2
Case (iii) −2 < a < 0 : In this case, f (x) = x + 3. Since x + 3 is a polynomial, therefore
f (x) is continuous at x = a.
Case (iv) a = 0 : In this case,

lim f (x) = lim− (x + 3) = 0 + 3 = 3


x→0− x→0
lim+ f (x) = lim+ sin x = sin 0 = 0.
x→0 x→0

Since lim− f (x) ̸= lim+ f (x). Therefore, lim f (x) does not exists. So, f (x) is not contin-
x→0 x→0 x→0
uous at a = 0.
Case (v) a > 0 : In this case, f (x) = sin x which is continuous for all real numbers.
Therefore f (x) is continuous at x = a.
Hence f (x) is continuous everywhere except at x = 0.
2.3. CONTINUITY 35

Example 2.5. Consider the function f (x) = [x] (greatest integer function) defined on
[−2, 2] as


 −2, −2 ≤ x < −1

−1, −1 ≤ x < 0



[x] = 0, 0≤x<1

1, 1≤x<2





2, x=2

Since lim − [x] = −2 ̸= −1 = lim + [x], therefore f (x) = [x] is not continuous at x = −1.
x→−1 x→−1
Similarly, [x] is not continuous at x = 0, 1. In general, [x] over R is discontinuous at all
integer values, i.e., x = 0, ±1, ±2, ±3, . . .

2.3.1 Types of Discontinuity

1. Removal Discontinuity: A function f (x) is said to have a removable discontinuity


at x = a if f (x) is defined at x = a, lim f (x) exists but lim f (x) ̸= f (a). Such type
x→a x→a
of discontinuity can be removed by changing the value of the function f (x) at x = a
(see Figure 2.1). .

Figure 2.1: Removable discontinuity at x = 3.

2. Discontinuity of First Kind: A function f (x) is said to have a discontinuity of first


kind at x = a if both lim− f (x) and lim+ f (x) exists but lim+ f (x) ̸= lim− f (x).
x→a x→a x→a x→a
This is also known as jump discontinuity because we see a jump in the value of
f (x) as we cross x = a from left to right or vice versa (see Figure 2.2).
36 LESSON - 2. CONTINUITY AND DIFFERENTIABILITY

Figure 2.2: Discontinuity of first kind at x = 3.

(i) A function f (x) is said to have a discontinuity of first kind at x = a from left
only, if lim− f (x) exists but lim− f (x) ̸= f (a).
x→a x→a

(ii) A function f (x) is said to have a discontinuity of first kind at x = a from right
only, if lim+ f (x) exists but lim+ f (x) ̸= f (a).
x→a x→a

3. Discontinuity of Second Kind: A function f (x) is said to have a discontinuity of


second kind at x = a if neither lim+ f (x) nor lim− f (x) exists (see Figure 2.3).
x→a x→a

Figure 2.3: Discontinuity of second kind at x = 1.


2.3. CONTINUITY 37

(i) A function f (x) is said to have a discontinuity of second kind at x = a from


left only, if lim− f (x) does not exists but lim+ f (x) does.
x→a x→a

(ii) A function f (x) is said to have a discontinuity of first kind at x = a from right
only, if lim+ f (x) does not exists but lim− f (x) does.
x→a x→a

Example 2.6. Consider the following function


(
x + 2, x ̸= 1
f (x) = .
2, x=1

We note that
lim f (x) = lim (x + 2) = 3 ̸= 2 = f (1).
x→1 x→1

Hence f (x) has a removable discontinuity at x = 1. The discontinuity is removed if we


redefine f (x) at x = 1 as f (1) = 3.

Example 2.7. Consider the following function


(
x − 1, x≤1
f (x) = .
2x, x>1

We can see that


lim f (x) = lim− (x − 1) = 1 − 1 = 0
x→1− x→1

and
lim f (x) = lim+ 2x = 2 · 1 = 2.
x→1+ x→1

As lim− f (x) ̸= lim+ f (x). Therefore, f (x) has discontinuity of first kind at x = 1.
x→1 x→1

Example 2.8. Consider the following function



 1 , x ̸= 1
f (x) = x − 1 .
5x2 , x=1

We note that
1
lim− f (x) = lim− = −∞,
x→1 x→1 x−1
and
1
lim+ f (x) = lim+ = ∞.
x→1 x→1 x−1
Therefore, f (x) has discontinuity of second kind at x = 1.

In-text Exercise 2.2. Examine the type of discontinuity (if any) of following functions at
x=2
38 LESSON - 2. CONTINUITY AND DIFFERENTIABILITY

1.  2
 x − 3x + 2
, x ̸= 2
f (x) = x−2 .
1, x=2

2. (
3x + 5, x<2
f (x) = .
x3 − 7, x≥2

3. (
x2 − 1, x≤2
f (x) = .
ln(x − 2), x>2

Definition 2.4. A function f (x) is said to be continuous on an open interval (a, b), if f (x)
is continuous at each point x such that a < x < b.
Note: For defining continuity on the closed interval [a, b], we can not have lim− f (x) and
x→a
lim+ f (x), as f (x) is not defined for x < a and x > b. Therefore, keeping this in mind, we
x→b
give the following definition:
Definition 2.5. A function f (x) is said to be continuous on a closed interval [a, b], if
(i) f (x) is continuous on (a, b),

(ii) f (x) is continuous at a from the right, i.e., lim+ f (x) = f (a),
x→a

(iii) f (x) is continuous at b from the left, i.e., lim− (x) = f (b).
x→b

2.4 Properties of Continuous Functions


Theorem 2.1 (Algebraic Properties). Let f and g be two functions continuous at a point
x = a,
i.e., lim f (x) = f (a), and lim g(x) = g(a),
x→a x→a

then
1. f + g is continuous at x = a, as

lim (f + g)(x) = lim f (x) + lim g(x) = f (a) + g(a) = (f + g)(a).


x→a x→a x→a

2. f − g is continuous at x = a, as

lim (f − g)(x) = lim f (x) + lim g(x) = f (a) − g(a) = (f − g)(a).


x→a x→a x→a

3. k · f is continuous at x = a for k (a real constant), as

lim (k · f )(x) = k · lim f (x) = k · f (a) = (k · f )(a),


x→a x→a
2.4. PROPERTIES OF CONTINUOUS FUNCTIONS 39

4. f · g is continuous at x = a, as
lim (f · g)(x) = lim f (x) · lim g(x) = f (a) · g(a) = (f · g)(a),
x→a x→a x→a

f
5. is continuous at x = a, provided g(a) ̸= 0, as
g
 
f lim f (x) f (a)
 
f
x→a
lim (x) = = = (a).
x→a g lim g(x) g(a) g
x→a

Theorem 2.2 (Composition of Continuous Functions). Let f : A → B and g : C → D


are two continuous functions such that g(C) ⊆ A, then their composition f ◦ g : C → B
defined by
(f ◦ g)(x) = f (g(x)), ∀x ∈ C (2.1)
is also a continuous function. Moreover,
 
lim (f ◦ g)(x) = lim f (g(x)) = f lim g(x) = f (g(a)) = (f ◦ g)(a). (2.2)
x→a x→a x→a

Example 2.9.
sin x
(i) Using Theorem 2.1, tan x = is continuous at all points where cos x ̸= 0.
cos x
Therefore, tan x is continuous for all x ∈ R\ ± π2 , ± 3π 5π

2
, ± 2
, . . . . Similarly we
can also find the points of continuity of other trigonometric functions.
(ii) Consider the function f (x) = sin x1 . We write f = g ◦ h, i.e., f (x) = g(h(x)),


where
1
g(x) = sin x, x ∈ R and h(x) = , x ̸= 0.
x
Since we know that g(x) = sin x is continuous for all x ∈ R and h(x) = x1 is
continuous at all x ∈ R except at x = 0. Hence, by using the Theorem 2.2, f (x) =
g(h(x)) = sin x1 is continuous at all x ∈ R except at x = 0.
(iii) Consider the function f (x) = | cos x|. We write f = g ◦ h, i.e., f (x) = g(h(x)),
where
g(x) = |x|, x ∈ R and h(x) = cos x, x ∈ R.
Since we know that g(x) = |x| is continuous for all x ∈ R and h(x) = cos x is
continuous at all x ∈ R. Hence, by using the Theorem 2.2, f (x) = g(h(x)) = | cos x|
is continuous at all x ∈ R.
(iv) Consider the function

−1, x < 0

f (x) = sgn(x) = 0, x = 0

1, x > 0

Therefore, f (x) = sgn(x) is continuous at all x ∈ R except at x = 0.


40 LESSON - 2. CONTINUITY AND DIFFERENTIABILITY

(v) Consider the function f (x) = [sin x]. We write f = g ◦ h, i.e., f (x) = g(h(x)),
where
g(x) = [x], x ∈ R and h(x) = sin x, x ∈ R.
Since we know that g(x) = [x] is continuous for all x ∈ R except at integers, i.e.,
x = 0, ±1, ±2, . . . and h(x) = sin x is continuous at all x ∈ R. Hence, by using the
Theorem 2.2, f (x) = g(h(x)) = [sin x] is continuous at all x ∈ R except at x such
that
sin x = −1, 0, 1
π
i.e., x = nπ and x = nπ + where n = 0, ±1, ±2, . . .
2
In-text Exercise 2.3. Find the points of discontinuity of following functions:
1. f (x) = cosec x
1
2. f (x) =
|x + 1|
3. f (x) = [2x]

2.5 Differentiability
Definition 2.6. A function f (x) is said to be differentiable(derivable) at x = a if
f (a + h) − f (a)
lim
h→0 h
exists. This limit is known as the derivative of the function f (x) at x = a and is denoted
by f ′ (a). That is,
f (a + h) − f (a)
f ′ (a) = lim .
h→0 h
Note.
f (a + h) − f (a)
1. lim− is known as the left hand derivative of f (x) at x = a and it
h→0 h
is denoted by Lf ′ (a)
f (a + h) − f (a)
2. lim+ is known as the right hand derivative of f (x) at x = a and
h→0 h
it is denoted by Rf ′ (a).
3. f (x) is differentiable at the point x = a if an only if Lf ′ (a) = Rf ′ (a).
Example 2.10. Consider the function f (x) = x + 2. Let us check the differentiability of
f (x) at x = 1. We have,
f (1 + h) − f (1) (1 + h + 2) − (1 + 2) h
lim = lim = lim = 1.
h→0 h h→0 h h→0 h

Therefore, f is differentiable at x = 1 and f ′ (1) = 1.


2.5. DIFFERENTIABILITY 41

Example 2.11. Consider the function

(
x, x≥0
f (x) = |x| =
−x, x<0

. Let us check the differentiability of f (x) at x = 0. We have,

f (0 + h) − f (0) f (h) − f (0) −h − 0 −h


Lf ′ (0) = lim− = lim− = lim = lim = −1
h→0 h h→0 h h→0 h h→0 h

and

f (0 + h) − f (0) f (h) − f (0) h−0 h


Rf ′ (0) = lim+ = lim+ = lim = lim = 1
h→0 h h→0 h h→0 h h→0 h

Since, Lf ′ (0) ̸= Rf ′ (0). Therefore f (x) is not differentiable at x = 0.

Example 2.12. Let discuss the differentiability of f (x) = |x| + |x − 1| at x = 0 and x = 1.


First, we simplify the given expression of the function f (x).

If x < 0, then |x| = −x and |x − 1| = −(x − 1). Therefore f (x) = |x| + |x − 1| =


−x − (x − 1) = −2x + 1.

If 0 ≤ x < 1, then |x| = x and |x − 1| = −(x − 1). Therefore f (x) = |x| + |x − 1| =


x − (x − 1) = 1.

If x ≥ 1, then |x| = x and |x − 1| = x − 1. Therefore f (x) = |x| + |x − 1| = x + (x − 1) =


2x − 1.

Therefore,,

−2x + 1,
 x<0
f (x) = 1, 0≤x<1.

2x − 1, x≥1

42 LESSON - 2. CONTINUITY AND DIFFERENTIABILITY

Therefore, for the differentiability at x = 0, we have

f (0 + h) − f (0)
Lf ′ (0) = lim−
h→0 h
f (h) − f (0)
= lim−
h→0 h
−2h + 1 − 1
= lim
h→0 h
−2h
= lim = −2
h→0 h
f (0 + h) − f (0)
and Rf ′ (0) = lim+
h→0 h
f (h) − f (0)
= lim+
h→0 h
1−1
= lim
h→0 h
0
= lim = 0
h→0 h

Since, Lf ′ (0) ̸= Rf ′ (0). Therefore, f (x) is not differentiable at x = 0.


Similarly, for the differentiability at x = 1, we have

f (1 + h) − f (1)
Lf ′ (1) = lim−
h→0 h
1−1
= lim =0
h→0 h
f (1 + h) − f (1)
and Rf ′ (1) = lim+
h→0 h
2(1 + h) − 1 − 1
= lim
h→0 h
2h
= lim =2
h→0 h

Since, Lf ′ (1) ̸= Rf ′ (1). Therefore, f (x) is not differentiable at x = 1.

In-text Exercise 2.4. Check the differentiability of the function


(
x + 2, x<2
1. f (x) = 2
, at x = 2.
x, x≥2

2. f (x) = x|x|, at x = 0.

3. f (x) = |x + 1| + |x − 1|, at x = −1 and x = 1.



 sin x2
, x ̸= 0
4. f (x) = x , at x = 0.
0, x=0
2.5. DIFFERENTIABILITY 43

Definition 2.7 (Derivative Function). Let f (x) be a function defined on (a, b). If f (x) is

derivable at each x ∈ (a, b), then the function f (x) defined by

′ f (x + h) − f (x)
f (x) = lim
h→0 h

d
is known as the derivative of f (x) with respect to x. It is also represented by f (x) ≡
dx
df
(x).
dx

Example 2.13.

(i) Consider the constant function f (x) = c, where c is a real constant. The domain of
f (x) is R. Therefore,

f (x + h) − f (x) c−c
lim = lim =0
h→0 h h→0 h

Therefore, f ′ (x) = 0 for all x ∈ R.

(ii) Consider f (x) = xn , x ∈ R, n ∈ N. Therefore,

f (x + h) − f (x) (x + h)n − xn
lim = lim
h→0 h h→0
h h i
n n−1 n(n−1) n−2 2 n
x + nx h+ 2
x h + ··· + h − xn
= lim
h→0 h
(Using Binomial Theorem)
n(n−1) n−2 2
nxn−1 h + 2
x h + · · · + hn
= lim
h→0
 h 
n−1 n(n − 1) n−2 n−1
= lim nx + x h + ··· + h
h→0 2
= nxn−1

Hence, f ′ (x) = nxn−1 ∀ x ∈ R, n ∈ N.


44 LESSON - 2. CONTINUITY AND DIFFERENTIABILITY

(iii) Consider f (x) = x, x > 0. Therefore,
√ √
f (x + h) − f (x) x+h− x
lim = lim
h→0 h h→0 h
√ √  √ √ 
x+h− x x+h+ x
= lim √ √
h→0 h x+h+ x
x+h−x
= lim √ √ 
h→0 h x+h+ x
h
= lim √ √ 
h→0 h x+h+ x
1
= lim √ √
h→0 x+h+ x
1 1
=√ √ = √
x+ x 2 x
1
Hence f ′ (x) = √ for x > 0.
2 x

Note. We note that f (x)is either a constant or a function of x.
Some Useful Derivatives:
d x d x
1. e = ex and a = ax · ln a, where a > 0 is a constant.
dx dx
d 1 d 1
2. ln x = and loga x = , x, a > 0.
dx x dx x ln a
d d
3. sin x = cos x and cos x = − sin x.
dx dx

2.5.1 Geometric Interpretation of a Derivative

Figure 2.4: Slope of the tangent to y = f (x) at the point P (a, f (a)).
2.5. DIFFERENTIABILITY 45

Consider the graph of function y = f (x) given in Figure 2.4. The tangent line to the graph
of y = f (x) at the point P (a, f (a)) is P S. Consider the secant lint P Q joining the points
P (a, f (a)) and Q(a + h, f (a + h)). The slope of the secant line is given by

f (a + h) − f (a) f (a + h) − f (a)
mP Q = = .
(a + h) − a h
From Figure 2.4 we can see that as h → 0, the point Q approaches the point P along the
curve and the secant line P Q approaches towards the tangent line P S. Therefore, the slope
of the tangent line can be defined as the limiting case of slope of secant line. So we have
f (a + h) − f (a)
mP S = lim mP Q = lim = f ′ (a),
h→0 h→0 h
provided the limit exists.
Therefore, the derivative f ′ (a) is the slope of the tangent line to the curve y = f (x)
at the point (a, f (a)). The equation of the tangent line at the point (a, f (a)) of the curve
y = f (x) is given by

y − f (a) = f (a)(x − a).
Also, the equation of the normal line at the point (a, f (a)) of the curve y = f (x) is given
by
1 ′
y − f (a) = ′ (x − a), provided f (a) ̸= 0.
f (a)
Example 2.14. Consider the parabola f (x) = x2 + 2. We are interested in finding the

equation of tangent and normal line to the given curve at the point (2, 6). Since f (x) = 2x.

Therefore, slope of the tangent at (2, 6) is f (2) = 2 · 2 = 4. Therefore, the equation of the
tangent line at (2, 6) is

y − 6 = 4(x − 2)
=⇒ y − 6 = 4x − 8
=⇒ y = 4x − 2

Also, the equation of the normal line at (2, 6) is


1
y − 6 = (x − 2)
4
x 1
=⇒ y−6= −
4 2
x 11
=⇒ y= +
4 2

2.5.2 Derivative as the Rate of Change


In daily life, rate of change occurs in many places. For example:
• A driver is interested in the velocity of the car which comprises of the speed he is
driving with and the direction he is driving towards.
46 LESSON - 2. CONTINUITY AND DIFFERENTIABILITY

• In pandemic, the government was interested in the rate at which the Covid-19 virus
was spreading relative to time, so that they can take proper measures to contain the
spread.
• Scientists are interested in the rate at which the glaciers are melting with time due to
temperature increase.
If y = f (x) shows a relation between a variable y and x, then we define
1. The average rate of change of y with respect to x in the interval [a, b], as
f (b) − f (a)
Ravg = .
b−a
2. The instantaneous rate of change of y with respect to x at the point x = a, as
f (a + h) − f (a)
Rinst = lim = f ′ (a),
h→0 h
provided the limit exists.

2.6 Some Theorems on Derivatives


Theorem 2.3. Every differentiable function is continuous.
Proof. Let us assume that f (x) is differentiable at x = a. Therefore,
f (a + h) − f (a)
f ′ (a) = lim . (2.3)
h→0 h
Now, we will show that f (x) is continuous at x = a, that is
lim f (x) = f (a). (2.4)
x→a

If h = x − a, then x → a implies h → 0. Therefore from (2.4), we have


lim f (a + h) = f (a). (2.5)
h→0

Consider
lim f (a + h) = lim [f (a + h) − f (a) + f (a)]
h→0 h→0
= lim [f (a + h) − f (a)] + lim f (a)
h→0
 h→0
f (a + h) − f (a)
= lim h + f (a)
h→0 h
f (a + h) − f (a)
= lim lim h + f (a)
h→0 h h→0
= f ′ (a) · 0 + f (a) (Using (2.3))
= f (a)
Therefore, f (x) is continuous at x = a.
2.6. SOME THEOREMS ON DERIVATIVES 47

Note.

1. From Theorem 2.3, we note that continuity is a necessary condition for a function to
be differentiable. A function f , which is not continuous at the point x = a, can not
be differentiable at x = a.

2. The converse of above theorem is not true. That is, a function which is continuous at
a point may or may not be differentiable at that point. For example,

(i) The function f (x) = |x| is continuous at x = 0 but not differentiable at x = 0.


(ii) The function f (x) = x|x| is continuous as well as differentiable at x = 0.

Theorem 2.4. (Algebraic Properties) Let f and g be two differentiable functions. Then

d d d
1. [f (x) + g(x)] = f (x) + g(x).
dx dx dx
d d d
2. [f (x) − g(x)] = f (x) − g(x).
dx dx dx
d d
3. [k · f (x)] = k · f (x), where k is a real constant.
dx dx
d d d
4. [f (x) · g(x)] = f (x) · g(x) + f (x) · g(x).
dx dx dx
d d

d f (x)
 f (x) · g(x) − f (x) · g(x)
5. = dx dx , provided g(x) ̸= 0.
dx g(x) [g(x)]2
cos x
Example 2.15. Consider the function f (x) = cot x = . Therefore,
sin x
d
f ′ (x) = (cot x)
dx
d  cos x 
=
dx sin x
d d
sin x (cos x) − cos x (sin x)
= dx dx
sin2 x
sin x · (− sin x) − cos x · cos x
=
sin2 x 
− sin2 x + cos2 x
=
sin2 x
−1
∵ sin2 x + cos2 x = 1

= 2
sin x
= −cosec2 x
48 LESSON - 2. CONTINUITY AND DIFFERENTIABILITY

Similarly, we can prove that,


d
tan x = sec2 x,
dx
d
sec x = tan x sec x,
dx
d
and cosec x = −cosec x cot x.
dx
Theorem 2.5. (Chain Rule) Let f : A → B and g : C → D are two differentiable
functions such that g(C) ⊆ A, then their composition f ◦ g : C → B is also differentiable.
Moreover,
′ ′ ′
(f ◦ g) (x) = f (g(x)) · g (x). (2.6)

Note. Here f (g(x)) represents the derivative of function f with respect to taking g(x) as
a single variable.
Example 2.16. Consider the function h(x) = sin(x2 ). If f (x) = sin x and g(x) = x2 , then
h(x) = f (g(x)). So we have
′ ′ ′
f (x) = cos x =⇒ f (g(x)) = f (x2 ) = cos x2 ,
and g ′ (x) = 2x.

Therefore,
′ ′
h′ (x) = f (g(x)) · g (x) = cos x2 · 2x = 2x cos x2 .
In-text Exercise 2.5. Find the derivative of the following functions:
1. f (x) = x2 sin x.

2. f (x) = ln(x2 + sin x).

2.6.1 Derivatives of the Inverse of an Invertible Function


Let y = f (x) be an invertible differentiable function in the domain (a, b) and let x = g(y)
be the inverse of y = f (x), i.e.,

(f ◦ g)(y) = y and (g ◦ f )(x) = x.

Therefore, we have
d
1= (x)
dx
d
= (g ◦ f )(x)
dx

= g (f (x)) · f ′ (x) (Using chain rule)
′ ′
= g (y)f (x)
′ 1 ′ 1
=⇒ g (y) = ′ or f (x) = ′ (2.7)
f (x) g (y)
2.6. SOME THEOREMS ON DERIVATIVES 49

Example 2.17. Consider the function y = f (x) = x1/n , where x > 0 and n ∈ N. The
inverse function of f (x) is given by g(y) = y n . Therefore, by using (2.7), we get
′ 1
f (x) = ′
g (y)
1
=
ny n−1
1 1−n
= y
n
1 1/n 1−n
= (x )
n
1 1/n−1
= x
n
Hence,
d 1/n 1
x = x1/n−1 , x > 0, n ∈ N.
dx n
−1
Example
  2.18. Consider the function y = f (x) = sin (x) where x ∈ [−1, 1], y ∈
−π π
, . The inverse function of f (x) is given by g(y) = sin y. Therefore, by using
2 2
(2.7), we get
′ 1
f (x) = ′
g (y)
1 π
= , provided cos y ̸= 0 i.e., y ̸= ±
cos y 2
1 −π π
=p (∵ cos y > 0 for < y < )
1 − sin2 y 2 2
1
=√
1 − x2
Therefore,
d 1
sin−1 (x) = √ , x ∈ (−1, 1)
dx 1 − x2
Similarly, we can also prove the following:
d −1
1. cos−1 x = √ , ∀x ∈ (−1, 1)
dx 1 − x2
d 1
2. tan−1 x = , ∀x ∈ R
dx 1 + x2
d −1
3. cot−1 x = , ∀x ∈ R
dx 1 + x2
d 1
4. sec−1 x = √ , ∀x ∈ R\(−1, 1)
dx |x| x2 − 1
d −1
5. cosec −1 x = √ , ∀x ∈ R\(−1, 1)
dx |x| x2 − 1
50 LESSON - 2. CONTINUITY AND DIFFERENTIABILITY

2.6.2 Application of Derivative


Derivatives have numerous applications. We consider the following few applications of
derivatives:
1. Let x(t) denote the displacement an object at time t relative to the origin. Then
dx dv d2 x
v(t) = gives the speed of the object at time t and a(t) = = 2 gives the
dt dt dt
acceleration of the object at time t.
2. Let C = C(x) and R = R(x) denote the cost function and the revenue function of
dC dR
a product for x units of the product. Then, and represent the marginal cost
dx dx
and the marginal revenue of the product.
dy
3. For the differentiable function y = f (x), is the slope of the tangent line at the
dx
dy
point P (x, y) on the curve y = f (x). That is, is the slope of the curve at P (x, y).
dx
1 dx
Similarly, slope of the normal to the curve at P (x, y) is dy or .
dx
dy
dy
4. For the function y = f (x), the sign of at the point P (a, f (a)) determine the
dx
increasing and decreasing nature of the function at the point P .
Example 2.19. Solve the following questions:
(i) If x(t) = t4 − 3t2 represent the position of an object at time t, where t is the time in
seconds and x is the displacement in meters. Find the speed and acceleration of the
object at time t = 3 seconds.
(ii) If the total cost C(x) of a commodity for x units is
C(x) = 5x3 − 2x2 − 30x + 5000
and the total revenue of the commodity for x units is
x3
R(x) = 2 + .
5
Find the marginal cost and the marginal revenue of the commodity.
(iii) Find the equation of the tangent and the normal to the curve y = f (x) = x2 at the
point P (1, 1).
Solution. (i) We have,
x(t) = t4 − 3t2
dx
=⇒ v(t) = = 4t3 − 6t
dt
dv
and a(t) = = 12t2 − 6
dt
2.7. SUMMARY 51

Hence, the speed of the object at t = 3 seconds is

v(3) = 4 · 33 − 6 · 3 = 90 m/sec

and acceleration is
a(3) = 12 · 32 − 6 = 102 m/sec2 .

(ii) We have

C(x) = 5x3 − 2x2 − 30x + 5000


dC
=⇒ Marginal cost = = 15x2 − 4x − 30
dx
Also,

x3
R(x) = 200 +
5
dR 3x2
=⇒ Marginal revenue = =
dx 5

(iii) We have

y = f (x) = x2

=⇒ f (x) = 2x

=⇒ f (1) = 2

Therefore, the equation of the tangent line at the point P (1, 1) is



y − 1 = f (1)(x − 1)
=⇒ y − 1 = 2(x − 1)
=⇒ y = 2x − 1

Similarly, the equation of the normal line to the curve at the point P (1, 1) is

1
y−1= ′ (x − 1)
f (1)
1
=⇒ y − 1 = (x − 1)
2
x 1
=⇒ y= +
2 2

2.7 Summary
In this lesson we have discussed the following points:

1. A function f is said to be continuous at a point x = a if


52 LESSON - 2. CONTINUITY AND DIFFERENTIABILITY

(i) f (x) is defined at x = a.


(ii) lim f (x) exists.
x→a
(iii) lim f (x) = f (a).
x→a

2. If any one of the three conditions mentioned above are not satisfied, then we say that
the function f is not continuous at x = a or f has discontinuity at x = a or x = a
is a point of discontinuity of f.

3. Type of discontinuity:

(i) Removal discontinuity: A function f (x) is said to have a removable disconti-


nuity at x = a if f (x) is defined at x = a, lim f (x) exists but lim f (x) ̸= f (a).
x→a x→a
In this type of functions, the discontinuity can be removed by changing the
value of the function f (x) at x = a.
(ii) Discontinuity of first kind: A function f (x) is said to have a discontinuity
of first kind at x = a if both lim− f (x) and lim+ f (x) exists but lim+ f (x) ̸=
x→a x→a x→a
lim− f (x). This is also known as jump discontinuity, because we see a jump
x→a
in the value of f (x) as we cross x = a from left to right or vice versa.
(iii) Discontinuity of second kind: A function f (x) is said to have a discontinuity
of second kind at x = a if neither lim+ f (x) nor lim− f (x) exists.
x→a x→a

4. Properties continuous functions:

(i) The sum, difference and product of two continuous functions are also continu-
ous functions.
(ii) The quotient of two continuous functions is also a continuous function provided
the denominator is non-zero.
(iii) The composition of two continuous functions is also a continuous function.

5. A function f (x) is said to be differentiable/derivable at x = a if


f (a + h) − f (a)
lim
h→0 h
exists. The limit is known as the derivative of the function f (x) at x = a and is
denoted by f ′ (a). Therefore,
f (a + h) − f (a)
f ′ (a) = lim .
h→0 h

6. Geometric Interpretation of Differentiability: The derivative f (a) of the function
f (x) at x = a is the slope of the tangent line to the curve y = f (x) at the point
(a, f (a)). The equation of the tangent line at point (a, f (a)) to the curve y = f (x) is
given by

y − f (a) = f (a)(x − a).
2.7. SUMMARY 53

Also, the equation of the normal line at point (a, f (a)) to the curve y = f (x) is given
by
1 ′
y − f (a) = ′ (x − a), provided f (a) ̸= 0.
f (a)

7. If y = f (x) shows a relation between a variable y depending on x, then we define

(i) The average rate of change of y with respect to x in interval [a, b] where
h > 0 as
f (b) − f (a)
Ravg = .
b−a
(ii) The instantaneous rate of change of y with respect to x at the point x = a
as
f (a + h) − f (a) ′
Rinst = lim = f (a),
h→0 h
provided the limit exists.

8. Every differentiable function is continuous but the converse is not necessarily true.

9. (Algebraic Properties) Let f and g be two differentiable functions. Then


d d d
(i) [f (x) + g(x)] = f (x) + g(x).
dx dx dx
d d d
(ii) [f (x) − g(x)] = f (x) − g(x).
dx dx dx
d d
(iii) [k · f (x)] = k · f (x), where k is a real constant.
dx dx
d d d
(iv) [f (x) · g(x)] = f (x) · g(x) + f (x) · g(x).
dx dx dx
d d

d f (x)
 f (x) · g(x) − f (x) · g(x)
(v) = dx 2
dx , provided g(x) ̸= 0.
dx g(x) [g(x)]

10. Chain Rule: Let f : A → B and g : C → D are two differentiable functions


such that g(C) ⊆ A, then their composition f ◦ g : C → B is also differentiable.
Moreover,
′ ′ ′
(f ◦ g) (x) = f (g(x)) · g (x).

11. Let y = f (x) be an invertible differentiable function in the domain (a, b) and let
x = g(y) be the inverse of y = f (x). Then

′ 1 ′ 1
g (y) = ′ or f (x) = ′
f (x) g (y)

12. Some applications of derivatives have been discussed.


54 LESSON - 2. CONTINUITY AND DIFFERENTIABILITY

2.8 Self-Assessment Exercises


1. Examine the continuity of the function f (x) = |x + 1| + |x − 1| on [−2, 2].
2. Obtain the points of discontinuity of the function f (x) defined on [0, 1] as follows:


 0, x=0

1
 2 − x, 0 < x < 21



f (x) = 12 , x = 12 .
 1 1
− x, <x<1




 2 2
1, x=1

3. Examine the continuity at x = 0 and x = 1 of the function f defined as follows:



2
−x ,
 x≤0
f (x) = 5x − 4, 0<x≤1.

 2
4x − 3x, x>1

Also write the type of discontinuity if any.


4. Determine the values of a and b for which the function defined as follows

ax2 + b, x≤0
f (x) = 3
1 − , x>0
2
x +1
is continuous.
5. Examine the continuity of the function f (x) = [x2 ], x ∈ R.
6. Discuss the differentiability/derivability of the function

√ x + 1

x≤0
f (x) = x−1
x2 + sin x x>0

at x = 0.

7. Find the value of a and b for which the function


(
ax2 + 2x, x≤2
f (x) =
bx − 1, x>2

is differentiable at x = 2. (Hint: First find value of a and b for which it is continuous and
then check for differentiability.)

8. Show that the function


( 2
2x + 3x− 4, x≤1
f (x) = πx
sin , x>1
2
2.9. SOLUTIONS TO IN-TEXT EXERCISES 55

is continuous but not differentiable at the point x = 1.

9. Find the equation of the tangent line to the curve y = x3 − 3x2 + 3 at the point (1, 1).

10. Find the equation of the normal line to the curve y = x2 +x cos x+2 at the point (0, 2).

11. Find the derivative of following functions:



(i) f (x) = cos x
x sin x
(ii) f (x) =
x2 + 3x
ln sin x
(iii) f (x) =
cos x2
2
(iv) f (x) = ex sin x + x2 e2x
 3 
x + 2x
(v) f (x) = ln
x2 + 5
12. An object is thrown from a building at a height of 128 meter above ground. The height
of the object can be modeled using the position function x(t) = 128 − 16t2 . Find the speed
and the acceleration of the object at time t = 5 seconds.

2.9 Solutions to In-text Exercises


Exercise 2.1

1. Not continuous at x = 1

2. Not continuous at x = 1

3. Continuous at x = 1

Exercise 2.2

1. No discontinuity

2. Discontinuity of first kind

3. Discontinuity of second kind

Exercise 2.3

1. x = nπ where n = 0, ±1, ±2, . . .

2. x = −1
n
3. x = where n = 0, ±1, ±2, . . .
2
56 LESSON - 2. CONTINUITY AND DIFFERENTIABILITY

Exercise 2.4

1. Not differentiable

2. Differentiable and f (0) = 0

3. Not differentiable

4. Differentiable and f (0) = 1

Exercise 2.5

1. x2 cos x + 2x sin x.
2x + cos x
2.
x2 + sin x

2.10 Suggested Readings


1. Narayan, S. & Mittal, P. K.(2019). Differential Calculus. S. Chand Publishing.
2. Anton, H., Bivens, I. C., & Davis, S. (2015). Calculus: Early Transcendentals. John
Wiley & Sons.
3. Singh, J.P. (2017). Calculus, 2nd Edition. Ane Books Pvt Ltd.
Lesson - 3

Successive Differentiation
Mr. Sandeep Bhatt
Ramlal Anand College
University of Delhi

Structure
3.1 Learning Objectives 57
3.2 Introduction 58
3.3 nth Derivatives of Some Standard Functions 61
3.3.1 nth Derivative of (ax + b)m , where a and b are Constants 61
3.3.2 nth Derivative of ln(ax + b), where a and b are Constants 62
3.3.3 nth Derivative of amx , where a and m are Constants 63
3.3.4 nth Derivative of sin(ax + b), where a and b are Constants 63
3.3.5 nth Derivative of cos(ax + b), where a and b are Constants 64
3.3.6 nth Derivative of eax sin(bx + c), where a, b and c are Constants 65
3.3.7 nth Derivative of eax cos(bx + c), where a, b and c are Constants 66
3.4 nth Derivative of Rational Functions 67
3.5 Leibnitz’s Theorem 70
3.6 Summary 73
3.7 Self-Assessment Exercises 75
3.8 Solutions to In-text Exercises 76
3.9 Suggested Readings 76

3.1 Learning Objectives


The learning objectives of this lesson are to:

• understand the concept of successive differentiation.

• to calculate the nth order derivatives of various functions.

• to study Leibnitz’s Theorem and its applications.

57
58 LESSON - 3. SUCCESSIVE DIFFERENTIATION

3.2 Introduction

In the last lesson, we discussed the derivative of a function f (x) with respect to the inde-
pendent variable x. Since the derivative of f (x) namely f ′ (x) is also a function of x, we
can talk about the derivative of f ′ (x) also. In this lesson we will discuss about the higher
order derivatives of f (x).
Successive differentiation is the process of differentiating a given function successively
and the derivatives obtained in this process are called successive derivatives.
Let y = f (x) be a function of x. Then the first derivative of y = f (x) with respect to
′ ′ dy d
x is denoted by f (x) or y (x) or y1 or or Dy where D ≡ is the differential operator.
dx dx
′ dy dy
Since f (x) = is also a function of x. If is differentiable, i.e. y = f (x) is twice
dx dx ′′
differentiable with respect to x, we denote its second derivative with respect to x by f (x)
′′ d2 y
or y (x) or y2 or 2 or D2 y.
dx
d2 y
Similarly, if 2 is differentiable, i.e. y = f (x) is thrice differentiable with respect to
dx
′′′ ′′′ d3 y
x, we denote its third derivative with respect to x by f (x) or y (x) or y3 or 3 or D3 y.
dx
In this manner, if f (x) is differentiable n times with respect to x, we denote the nth
dn y
derivative of f (x) with respect to x by f (n) (x) or y (n) (x) or yn or n or Dn y.
dx

Note. We denote the nth order derivative (nth derivative) of the function y = f (x) with
n

d y
respect to x at x = a by f (n) (a) or y (n) (a) or .
dxn x=a

Example 3.1. Consider the function f (x) = x cos 2x + e2x . We have

f ′ (x) = cos 2x − 2x sin 2x + 2e2x ,


′′
f (x) = −2 sin 2x − 2 sin 2x − 4x cos 2x + 4e2x (By using the chain rule)
= −4 sin 2x − 4x cos 2x + 4e2x ,
′′′
and f (x) = −8 cos 2x − 4 cos 2x + 8 sin 2x + 8e2x
= −12 cos 2x + 8 sin 2x + 8e2x .

ln x
Example 3.2. Consider the function y = . Let us calculate the second order derivative
x
d2 y
. We have
dx2
3.2. INTRODUCTION 59

1
dy x· − ln x · 1
= x
dx x2
1 − ln x
=
x2
−1
d2 y x2 · − (1 − ln x) · 2x
=⇒ = x
dx2 x4
−x − 2x (1 − ln x)
=
x4
−1 − 2 (1 − ln x)
=
x3
d2 y 2 ln x − 3
=⇒ =
dx2 x3

Example 3.3. Let y = x + tan x. We will prove that

d2 y
cos2 x + −2y + 2x = 0.
dx2

First we calculate the derivatives of y as desired.

dy
= 1 + sec2 x
dx
d2 y
and = 2 sec x · sec x tan x (By using the chain rule)
dx2
= 2 sec2 x tan x

Therefore, we have

d2 y
cos2 x + −2y + 2x = cos2 x · 2 sec2 x tan x − 2(x + tan x) + 2x
dx2
= 0.

d2 y
Example 3.4. Let x = a(t + sin t) and y = a(1 + cos t). We will find the value of 2 at
dx
π
t = . We have
2

dx
x = a(t + sin t) =⇒ = a(1 + cos t)
dt
dy
y = a(1 + cos t) =⇒ = −a sin t
dt
60 LESSON - 3. SUCCESSIVE DIFFERENTIATION

Therefore
dy dy dt
= ·
dx dt dx
dy/dt
=
dx/dt
−a sin t
=
a(1 + cos t)
t t
−2 sin cos
= 2 2
2
t
2 cos
2
t
= − tan
2
Now
d2 y
 
d dy
=
dx2 dx dx
 
d dy dt
= ·
dt dx dx
 
d dy
dt dx
=
dx
dt
−1 t
sec2
= 2 2
a(1 + cos t)
−1 t
sec2
= 2 2
t
2a cos2
2
−1 t
= sec4
4a 2
Therefore,
d2 y −1 √ 4 −1

−1 4 π
= sec = ( 2) =
dx2 t= π 4a 4 4a a
2
In-text Exercise 3.1. Solve the following questions:
′′′
1. Find f (x) where f (x) = sin(2x3 ).
′′
2. If y(x) = a cos 2x + b sin 2x, where a and b are real constants, then show that y +
4y = 0.
3. If x = sin t, y = sin at, show that
d2 y dy
(1 − x2 ) 2
− x + a2 y = 0.
dx dx
3.3. nth DERIVATIVES OF SOME STANDARD FUNCTIONS 61

4. If y = sin(sin x),show that

d2 y dy
2
+ tan x + y cos2 x = 0.
dx dx

3.3 nth Derivatives of Some Standard Functions


3.3.1 nth Derivative of (ax + b)m , where a and b are Constants
Let y = (ax + b)m . Differentiating with respect to x successively, we get

y1 = ma(ax + b)m−1
y2 = m(m − 1)a2 (ax + b)m−2
y3 = m(m − 1)(m − 2)a3 (ax + b)m−3
..
.
yn = m(m − 1)(m − 2) · · · (m − (n − 1))an (ax + b)m−n (3.1)

Case 1. m is a positive integer such that m ≥ n. Then from (3.1), we have

m!
yn = an (ax + b)m−n
(m − n)!

where m! = m(m − 1)(m − 2) · · · 3 · 2 · 1. Therefore,

dn m!
n
(ax + b)m = an (ax + b)m−n , m ≥ n, m > 0 (3.2)
dx (m − n)!

This also implies that the mth order derivative of (ax + b)m is m!am , which is a constant.
That is
dm
(ax + b)m = m!am (3.3)
dxm
1
Case 2. If m = −1, then y = = (ax + b)−1 . Then from (3.1), we have
ax + b

yn = (−1)(−2)(−3) · · · (−1 − (n − 1))an (ax + b)−1−n


= (−1)(−2) · · · (−n)an (ax + b)−(n+1)
(−1)n (n!)an
=
(ax + b)n+1

Therefore,
dn (−1)n (n!) an
 
1
= (3.4)
dxn ax + b (ax + b)n+1

Example 3.5. Find the 8th derivative of y = (8x + 7)10 .


62 LESSON - 3. SUCCESSIVE DIFFERENTIATION

Solution. We have,
y = (8x + 7)10
Therefore, by using (3.2), with m = 10 and n = 8, we get
10! 8
y8 = 8 (8x + 7)2 .
2!
Example 3.6. Find the nth derivative of the function
1
y= .
(x − 1)2
Solution. We have
1
y=
(x − 1)2
 
d 1
=− .
dx x − 1
Therefore,
1
nth derivative of y = −(n + 1)th derivative of
x−1
n+1
 
d 1
= − n+1
dx x−1
n+1
(−1) (n + 1)! 1n+1
=−
(x − 1)n+2
(−1)n+2 (n + 1)!
=
(x − 1)n+2

3.3.2 nth Derivative of ln(ax + b), where a and b are Constants


Let y = ln(ax + b). We have
a
y1 = .
ax + b
Since, yn = (n − 1)th derivative of y1 , therefore by using (3.4), we have
(−1)n−1 (n − 1)! an−1 (−1)n−1 (n − 1)! an
yn = a · =
(ax + b)n (ax + b)n
. Therefore,
dn (−1)n−1 (n − 1)! an
ln(ax + b) = (3.5)
dxn (ax + b)n

Example 3.7. Consider the function y = ln(4x−3). Then, by using (3.5), the nth derivative
of y is
(−1)n−1 (n − 1)!4n
yn = .
(4x − 3)n
3.3. nth DERIVATIVES OF SOME STANDARD FUNCTIONS 63

3.3.3 nth Derivative of amx , where a and m are Constants


Let y = amx . Differentiating successively with respect to x, we get

y1 = mamx ln a,
y2 = m ln a · mamx ln a = m2 (ln a)2 amx ,
y3 = m2 (ln a)2 · mamx ln a = m3 (ln a)3 amx ,
..
.
yn = mn (ln a)n amx .

Therefore,
dn mx
a = mn (ln a)n amx (3.6)
dxn

If a = e, then ln e = 1. Therefore,

dn mx
e = mn emx (3.7)
dxn

Example 3.8. Consider the function y = 53x + e2x . Then, by using (3.6) and (3.7), the nth
derivative of y is

yn = 3n (ln 3)n 53x + 2n e2x .

3.3.4 nth Derivative of sin(ax + b), where a and b are Constants


Let y = sin(ax + b). Differentiating successively with respect to x, we get
π   π  
y1 = a cos(ax + b) = a sin + ax + b ∵ sin + θ = sin θ ,
2 2  
π  π π  2π
2 2
y2 = a · a cos + ax + b = a sin + + ax + b = a sin + ax + b ,
2 2 2 2
   
2 2π 3 3π
y3 = a · a cos + ax + b = a sin + ax + b ,
2 2
..
.
 nπ 
yn = an sin + ax + b .
2

Therefore,
dn n
 nπ 
sin(ax + b) = a sin + ax + b (3.8)
dxn 2
64 LESSON - 3. SUCCESSIVE DIFFERENTIATION

3.3.5 nth Derivative of cos(ax + b), where a and b are Constants


Let y = cos(ax + b). Differentiating successively with respect to x, we get
π   π  
y1 = −a sin(ax + b) = a cos + ax + b ∵ cos + θ = − sin θ ,
2 2  
π  π π  2π
2 2
y2 = a · (−a) sin + ax + b = a cos + + ax + b = a cos + ax + b ,
2 2 2 2
   
2 2π 3 3π
y3 = a · (−a) sin + ax + b = a cos + ax + b ,
2 2
..
.
 nπ 
n
yn = a cos + ax + b .
2
Therefore,
dn n
 nπ 
cos(ax + b) = a cos + ax + b (3.9)
dxn 2
Example 3.9. Consider the function y = sin 2x sin x. To find the nth derivative, we proceed
as
y = sin 2x sin x
1
= · 2 sin 2x sin x
2
1
= [cos(2x − x) − cos(2x + x)] (∵ cos(x − y) − cos(x + y) = 2 sin x sin y)
2
1
= [cos x − cos 3x]
2
Therefore using (3.9), we have
1 h  nπ   nπ i
yn = cos + x − 3n cos + 3x .
2 2 2
2
Example 3.10. Consider the function y = cos x sin x. To find the nth derivative, we
proceed as
1
y = · 2 cos2 x sin x
2
1
∵ cos 2x = 2 cos2 x − 1

= (1 + cos 2x) sin x
2
1
= [sin x + cos 2x sin x]
2
1 1
= sin x + cos 2x sin x
2 2
1 1
= sin x + · 2 cos 2x sin x
2 4
1 1
= sin x + [sin 3x − sin x] (∵ sin(x + y) + sin(x − y) = 2 sin x cos y)
2 4
1 1
= sin x + sin 3x
4 4
3.3. nth DERIVATIVES OF SOME STANDARD FUNCTIONS 65

By using (3.8), we have


1  nπ  3n  nπ 
yn = sin +x + sin + 3x
4 2 4 2

3.3.6 nth Derivative of eax sin(bx + c), where a, b and c are Constants
Let y = eax sin(bx + c). Differentiating with respect to x, we get

y1 = aeax sin(bx + c) + beax cos(bx + c) = eax [a sin(bx + c) + b cos(bx + c)] (3.10)

To formulate a proper generalization, let us assume

a = r cos θ, b = r sin θ (3.11)

where r ≥ 0. Therefore, we have

a2 + b2 = r2 cos2 θ + r2 sin2 θ = r2 cos2 θ + sin2 θ = r2




=⇒ r = a2 + b2 (3.12)

and
b r sin θ sin θ
= = = tan θ
a r cos θ cos θ
b
=⇒ θ = tan−1 . (3.13)
a

Therefore, using (3.11), we can write (3.10) in the form

y1 = eax [r cos θ sin(bx + c) + r sin θ cos(bx + c)]


= reax sin(bx + c + θ) (∵ sin(x + y) = sin x cos y + cos x sin y)

We can see that y1 is obtained by multiplying y by r and increasing bx + c by the constant


θ. Using the same process we can calculate y2 from y1 and so on. Hence, we have

y2 = r2 eax sin(bx + c + 2θ),


y3 = r3 eax sin(bx + c + 3θ),
..
.
yn = rn eax sin(bx + c + nθ).

Therefore,

dn ax √
 
b
e sin(bx + c) = r n ax
e sin(bx + c + nθ), where r = a2 + b2 and θ = tan−1
dxn a
(3.14)
66 LESSON - 3. SUCCESSIVE DIFFERENTIATION

3.3.7 nth Derivative of eax cos(bx + c), where a, b and c are Constants
Let y = eax cos(bx + c). Differentiating with respect to x, we get

y1 = aeax cos(bx + c) − beax sin(bx + c) = eax [a cos(bx + c) + b sin(bx + c)] (3.15)

To formulate a proper generalization, let us assume

a = r cos θ, b = r sin θ (3.16)

where r ≥ 0. Therefore, we have



 
−1 b
r= a2 + b 2 , and θ = tan .
a
Therefore, using (3.16), we can write (3.15) in the form

y1 = eax [r cos θ cos(bx + c) − r sin θ sin(bx + c)]


= reax cos(bx + c + θ) (∵ cos(x + y) = cos x cos y − sin x sin y)

We can see that y1 is obtained by multiplying y by r and increasing bx + c by the constant


θ. Using the same process we can calculate y2 from y1 and so on.
Hence, we have

y2 = r2 eax cos(bx + c + 2θ),


y3 = r3 eax cos(bx + c + 3θ),
..
.
yn = rn eax cos(bx + c + nθ).

Therefore,

dn ax √
 
b
e cos(bx + c) = r n ax
e cos(bx + c + nθ), where r = a2 + b2 and θ = tan−1
dxn a
(3.17)

Example 3.11. Consider the function y = e2x sin(2x − 1). Comparing it with the function
y = eax sin(bx + c), we have a = 2, b = 2 and c = −1. Therefore,
√ √ √
r = a2 + b2 = 22 + 22 = 8,
 
−1 2 π
and θ = tan = tan−1 (1) = .
2 4

Therefore, using (3.14), the nth derivative of y is

yn = rn e2x sin(2x − 1 + nθ)


√  π
= ( 8)n e2x sin 2x − 1 + n .
4
3.4. nth DERIVATIVE OF RATIONAL FUNCTIONS 67

Example 3.12. Consider the function y = e2x sin 2x sin x. To find the nth derivative, we
proceed as

y = e2x sin 2x sin x


e2x
= (2 sin 2x sin x)
2
e2x
= (cos x − cos 3x)
2
1 2x
e cos x − e2x cos 3x

=
2

Therefore, using (3.17), the nth derivative of y is

1  n 2x
r1 e cos (x + nθ1 ) − r2n e2x cos (3x + nθ2 )

yn = (3.18)
2
where
√ √
r1 = 22 + 12 = 5,
 
−1 1
θ1 = tan ,
2
√ √
r2 = 22 + 32 = 13,
 
−1 3
and θ2 = tan .
2

In-text Exercise 3.2. Find the nth derivative of following functions:

1. y = (5x − 6)n .

3
2. y = .
(x + 2)2

3. y = e2x+3 .

4. y = cos2 x.

5. y = e2x cos 3x.

3.4 nth Derivative of Rational Functions


In order to find the nth derivative of a rational function, we use method of partial fractions.
If the denominator of each partial fraction obtained consists of real linear factors we use
formulae derived in the previous section to find the nth derivative of the given rational
function. We can take help of following table to calculate partial fractions:
68 LESSON - 3. SUCCESSIVE DIFFERENTIATION

S.N. Rational Fraction Partial Fraction Form


ax + b A B
1. +
(x − p)(x − q) x−p x−q
ax + b A B
2. +
(x − p)2 x − p (x − p)2
ax2 + bx + c A B C
3. + +
(x − p)(x − q)(x − r) x−p x−q x−r
ax2 + bx + c A B C
4. + +
(x − p)2 (x − q) x − p (x − p)2 x − q
ax2 + bx + c A Bx + C
5. + 2
(x − p)(x2 + qx + r) x − p x + qx + r

where A, B, C are constants in all the above partial fractions.


x
Example 3.13. Consider the function y = . Now, to resolve it into partial
(x + 3)(2x + 5)
fractions, let

x A B
= + , (A, B are constants) (3.19)
(x + 3)(2x + 5) x + 3 2x + 5
x A(2x + 5) + B(x + 3)
=⇒ =
(x + 3)(2x + 5) (x + 3)(2x + 5)
=⇒ x = A(2x + 5) + B(x + 3)
=⇒ x = (2A + B)x + 5A + 3B

By equating coefficients of x and the constant terms on both sides, we get

2A + B = 1
(3.20)
5A + 3B = 0

By solving (3.20), we get A = 3, B = −5. Substituting values of A and B in (3.19), we


have
x 3 5
y= = −
(x + 3)(2x + 5) x + 3 2x + 5
Hence, by using (3.4), we have

(−1)n n! 1n (−1)n n! 2n
yn = 3 · − 5 ·
(x + 3)n+1 (2x + 5)n+1
5 · 2n
 
n 3
= (−1) n! − .
(x + 3)n+1 (2x + 5)n+1

4x
Example 3.14. Consider the function y = . Now, to resolve it into partial
(x − 3)2 (x + 1)
3.4. nth DERIVATIVE OF RATIONAL FUNCTIONS 69

fractions, let
4x A B C
2
= + 2
+ , (A, B, C are constants) (3.21)
(x − 1) (x + 1) x − 1 (x − 1) x+1
4x A(x − 1)(x + 1) + B(x + 1) + C(x − 1)2
=⇒ =
(x − 1)2 (x + 1) (x − 1)2 (x + 1)
=⇒ 4x = A(x − 1)(x + 1) + B(x + 1) + C(x − 1)2
=⇒ 4x = (A + C)x2 + (B − 2C)x − A + B + C
By equating coefficients of x2 , x and the constant terms on both sides, we get
A+C =0
B − 2C = 4 (3.22)
−A + B + C = 0
By solving (3.22), we get A = 1, B = 2, C = −1. Substituting values of A, B and C in
(3.21), we have
4x 1 2 1
y= 2
= + 2

(x − 1) (x + 1) x − 1 (x − 1) x+1
Hence, by using (3.4), we have
(−1)n n! 1n (−1)n+2 (n + 1)! 1n (−1)n n! 1n
yn = + 2 · −
(x − 1)n+1 (x − 1)n+2 (x + 1)n+1
 
n 1 2(n + 1) 1
= (−1) n! + −
(x − 1)n+1 (x − 1)n+2 (x + 1)n+1
Note. If the denominator of the given rational function is not resolvable into real linear
factors, then nth derivative is calculated with the help of the De Moivre’s theorem, which
states that √
(cos θ ± i sin θ)n = cos nθ ± i sin nθ, where i = −1,
and by using the factorization x2 + a2 = (x + ia)(x − ia).
x
Example 3.15. Consider the function y = 2 . We can write
x + a2
x
y= 2
x + a2
x
=
(x + ia)(x − ia)
 
1 1 1
= + (Using partial fractions)
2 x + ia x − ia
(−1)n n!
 
1 1
=⇒ yn = + (Using (3.4)) (3.23)
2 (x + ia)n+1 (x − ia)n+1

To make the result free from i, we put x = r cos θ, a = r sin θ where r = x2 + a2 and θ =
a
tan−1 .
x
70 LESSON - 3. SUCCESSIVE DIFFERENTIATION

Therefore,
1 1
n+1
=
(x + ia) (r cos θ + ir sin θ)n+1
1
= n+1 (cos θ + i sin θ)−(n+1)
r
1
= n+1 [cos(n + 1)θ − i sin(n + 1)θ] (3.24)
r
1 1
and n+1
=
(x − ia) (r cos θ − ir sin θ)n+1
1
= n+1 (cos θ − i sin θ)−(n+1)
r
1
= n+1 [cos(n + 1)θ + i sin(n + 1)θ] (3.25)
r
Using (3.23)-(3.25), we have
(−1)n n!
 
1 1
yn = +
2 (x + ia)n+1 (x − ia)n+1
(−1)n n!
= [cos(n + 1)θ − i sin(n + 1)θ + cos(n + 1)θ + i sin(n + 1)θ]
2rn+1
(−1)n n!
= · 2 cos(n + 1)θ
2rn+1
(−1)n n! cos(n + 1)θ
=
rn+1
In-text Exercise 3.3. Find the nth derivative of the following functions:
x2
1. y =
(x + 2)(2x + 3)
1
2. y =
x 2 + a2

3.5 Leibnitz’s Theorem


Leibnitz’s theorem is used for finding the nth derivative of the product of two functions in
the terms of successive derivatives of the functions.
Theorem 3.1 (Leibnitz’s Theorem). If u and v are functions of x having n successive
derivatives, then for n > 1
       
n n n n
(uv)n = un v + un−1 v1 + un−2 v2 · · · + un−r vr + · · ·
0 1 2 r
   
n n
+ u1 vn−1 + uvn
n−1 n
where  
n n!
= , for r = 0, 1, 2, . . . , n.
r r!(n − r)!
3.5. LEIBNITZ’S THEOREM 71

Example 3.16. Consider the function y = x ln x. Let u = x and v = ln x. Therefore,

u1 = 1, ur = 0 for r = 2, 3, . . . , n
(−1)r−1 (r − 1)! (3.26)
and vr = for r = 1, 2, . . . , n.
xr
Therefore, by using the Libnitz’s Theorem, we have
         
n n n n n
yn = (uv)n = un v + un−1 v1 + un−2 v2 · · · + u1 vn−1 + uvn
0 1 2 n−1 n
   
n n
= u1 vn−1 + uvn ( Using (3.26))
n−1 n
(−1)n−2 (n − 2)! (−1)n−1 (n − 1)!
=n·1· + 1 · x ·
xn−1 xn
n−2
(−1) (n − 2)!
= [n − (n − 1)]
xn−1
(−1)n−2 (n − 2)!
=
xn−1
Note. We can also solve this example by choosing u = ln x and v = x.
Example 3.17. Consider the function y = x2 cos 3x. Let u = cos 3x and v = x2 . There-
fore,  rπ 
ur = 3r cos + 3x for r = 1, 2, . . . , n.
2 (3.27)
v1 = 2x, v2 = 2, vr = 0 for r = 3, 4, . . . n
Therefore, by Libnitz’s Theorem, we have
         
n n n n n
yn = (uv)n = un v + un−1 v1 + un−2 v2 · · · + u1 vn−1 + uvn
0 1 2 n−1 n
     
n n n
= un v + un−1 v1 + un−2 v2 ( Using (3.27))
0 1 2
 
n
 nπ 
2 n−1 (n − 1)π
= 1 · 3 cos + 3x · x + n · 3 cos + 3x · 2x
2 2
 
n(n − 1) n−2 (n − 2)π
+ ·3 cos + 3x · 2
2 2
  
n−2 2
 nπ  (n − 1)π
=3 9x cos + 3x + 6nx cos + 3x
2 2
 
(n − 2)π
+n(n − 1) cos + 3x
2

Note. We can also solve this example by choosing u = x2 and v = cos 3x.
In-text Exercise 3.4. Find the nth derivative of following functions:
1. y = x2 e3x
72 LESSON - 3. SUCCESSIVE DIFFERENTIATION

2. y = x3 cos x
Example 3.18. By using the Leibnitz’s Theorem, prove that

x2 yn+2 + (2n + 1)xyn+1 + (n2 + 1)yn = 0,

where y = a cos(ln x) + b sin(ln x).


Solution. Differentiating y with respect to x, we get
1 1
y1 = −a sin(ln x) · + b cos(ln x)
x x
=⇒ xy1 = −a sin(ln x) + b cos(ln x)

Differentiating both sides with respect to x, we have


1 1
xy2 + y1 = −a cos(ln x) ·− b sin(ln x) ·
x x
2
=⇒ x y2 + xy1 = −a cos(ln x) − b sin(ln x)
=⇒ x2 y2 + xy1 + y = 0

We now differentiate the above equation n times. To differentiating the product terms x2 y2
and xy1 , we use Leibnitz’s Theorem. Note that yn+2 = y2+n is the nth derivative of y2 . We
obtain
      
n 2 n n
yn+2 · x + yn+1 · 2x + yn · 2
0 1 2
    
n n
+ yn+1 · x + yn · 1 + yn = 0
0 1
=⇒ x2 yn+2 + 2nxyn+1 + n(n − 1)yn + xyn+1 + nyn + yn = 0
=⇒ x2 yn+2 + (2n + 1)xyn+1 + (n2 + 1)yn = 0

Example 3.19. For the function y = sin−1 x, prove that

(1 − x)2 yn+2 − (2n + 1)xyn+1 − n2 yn = 0.

Solution. Differentiating y = sin−1 x with respect to x we have,


1
y1 = √
1 − x2
1
=⇒ y12 =
1 − x2
=⇒ (1 − x2 )y12 = 1

Again differentiating the above equation with respect to x, we get

− 2xy12 + (1 − x2 ) · 2y1 y2 = 0
=⇒ (1 − x2 )y2 − xy1 = 0 (Dividing both sides by 2y1 )
3.6. SUMMARY 73

Now, differentiating n times by using the Leibnitz’s Theorem, we get


      
n 2 n n
yn+2 · (1 − x ) + yn+1 · (−2x) + yn · (−2)
0 1 2
    
n n
− yn+1 · x + yn · 1 = 0
0 1
 
2 n(n − 1)
=⇒ (1 − x ) · yn+2 + n · (−2x) · yn+1 + · (−2) · yn
2
− [x · yn+1 + n · 1 · yn ] = 0
2
=⇒ (1 − x )yn+2 − 2nxyn+1 − n(n − 1)yn − xyn+1 − nyn = 0
=⇒ (1 − x2 )yn+2 − (2n + 1)xyn+1 − n2 yn = 0

In-text Exercise 3.5. Solve the following questions:


1. If y = tan−1 x, show that

(1 − x)2 yn+2 + 2(n + 1)xyn+1 + n(n + 1)yn = 0.


−1
2. If y = em sin x
, show that

(1 − x)2 yn+2 − (2n + 1)xyn+1 − (n2 + m2 )yn = 0.

3.6 Summary
In this lesson we have discussed the following points:
1. If the function y = f (x) is differentiable successively, then its successive derivatives
are denoted as:

′ ′ dy
First derivative: f (x) or y (x) or y1 or or Dy
dx
′′ ′′ d2 y
Second derivative: f (x) or y (x) or y2 or 2
or D2 y
dx
′′′ ′′′ d3 y
Third derivative: f (x) or y (x) or y3 or or D3 y
dx3
..
.
dn y
nth derivative: f (n) (x) or y (n) (x) or yn or or Dn y
dxn

2. nth derivative of some well known functions are as follows:


dn m!
(i)n
(ax + b)m = an (ax + b)m−n , m ≥ n, m > 0.
dx (m − n)!
dn (−1)n n! an
 
1
(ii) =
dxn ax + b (ax + b)n+1
74 LESSON - 3. SUCCESSIVE DIFFERENTIATION

dn (−1)n−1 (n − 1)! an
(iii) ln(ax + b) =
dxn (ax + b)n
dn mx
(iv) n
a = mn (ln a)n amx
dx
dn mx
(v) e = mn emx
dxn
dn n
 nπ 
(vi) sin(ax + b) = a sin + ax + b
dxn 2
dn n
 nπ 
(vii) cos(ax + b) = a cos + ax + b
dxn 2
dn ax √
(viii) n
e sin(bx + c) = rn eax sin(bx + c + nθ), where r = a2 + b2 and θ =
dx  
b
tan−1
a
n
d ax √
(ix) n
e cos(bx + c) = rn eax cos(bx + c + nθ), where r = a2 + b2 and θ =
dx  
b
tan−1
a
3. nth derivative of a rational function can be calculated with the help of partial fractions
and De Moivre’s theorem. We can take help of following table to calculate partial
fractions:

S.N. Rational Fraction Partial Fraction Form


ax + b A B
1. +
(x − p)(x − q) x−p x−q
ax + b A B
2. +
(x − p)2 x − p (x − p)2
ax2 + bx + c A B C
3. + +
(x − p)(x − q)(x − r) x−p x−q x−r
ax2 + bx + c A B C
4. + +
(x − p)2 (x − q) x − p (x − p)2 x−q
ax2 + bx + c A Bx + C
5. + 2
(x − p)(x2 + qx + r) x − p x + qx + r

where A, B and C are constants.


4. Leibnitz’s Theorem: If u and v are functions of x having n successive derivatives,
then
       
n n n n
(uv)n = un v + un−1 v1 + un−2 v2 · · · + un−r vr + · · ·
0 1 2 r
   
n n
+ u1 vn−1 + uvn
n−1 n
3.7. SELF-ASSESSMENT EXERCISES 75

where  
n n!
= , for r = 0, 1, 2, . . . , n.
r r!(n − r)!

3.7 Self-Assessment Exercises


1. Find the nth derivative of

(i) y = sin3 x

(ii) y = sin x sin 2x sin 3x

(iii) y = sin2 x cos x

(iv) y = ex cos x sin x

(v) y = ex sin2 x
x
(vi) y =
(x − 1)(2x + 7)

x2 + 1
(vii) y =
(x − 1)(x2 − 4)

(viii) y = tan−1 x

2. Show that

dn (−1)n n!
   
ln x 1 1 1
= ln x − 1 − − − · · · − .
dxn x xn+1 2 3 n
√ 
3. If y = ln x + 1 + x2 , show that

(i) (1 + x2 )y2 + xy1 = 0,

(ii) (1 + x2 )yn+2 + (2n + 1)xyn+1 + n2 yn = 0.


−1
4. If y = em cos x
, show that

(1 − x2 )yn+2 − (2n + 1)xyn+1 − (n2 + m2 )yn = 0.

5. If y = (sin−1 x)2 , show that

(i) (1 − x)2 y2 − xy1 − 2 = 0,

(ii) (1 − x)2 yn+2 − (2n + 1)xyn+1 − n2 yn = 0.


76 LESSON - 3. SUCCESSIVE DIFFERENTIATION

3.8 Solutions to In-text Exercises


Exercise 3.1

1. 12 [(1 − 18x6 ) cos(2x3 ) − 18x3 sin(2x3 )]

Exercise 3.2

1. 5n n!
(−1)n+2 (n + 1)! 2n
2.
(2x + 3)n+2
3. 2n e2x+3
 nπ 
n−1
4. 2 cos + 2x
2
5. 10n/2 sin(3x + n tan−1 3)

Exercise 3.3
(−1)n n! 9 · 2n
 
8
1. −
2 (2x + 3)n+1 (x + 2)n+1
(−1)n n! sin(n + 1)θ √
, where r = a2 + b2 and θ = tan−1 b

2. n+1 a
.
ar
Exercise 3.4

1. 3n−2 e3x [9x2 + 6nx + n(n − 1)]


   
3
 nπ  3 (n − 1)π (n − 2)π
2. x cos x + + 3nx cos x + + 3n(n − 1)x cos x + +
2  2 2
(n − 3)π
n(n − 1)(n − 2) cos x +
2

3.9 Suggested Readings


1. Narayan, S. & Mittal, P. K.(2019). Differential Calculus. S. Chand Publishing.
2. Anton, H., Bivens, I. C., & Davis, S. (2015). Calculus: Early Transcendentals. John
Wiley & Sons.
3. Singh, J.P. (2017). Calculus , 2nd Edition, Ane Books Pvt Ltd.
Lesson - 4

Partial Differentiation
Mr. Sandeep Bhatt
Ramlal Anand College
UUniversity of Delhi

Structure
4.1 Learning Objectives 77
4.2 Introduction 78
4.3 Function of Two Variables 78
4.4 Partial Derivatives 79
4.4.1 Partial Derivatives of Function of Two Variables 79
4.4.2 Geometric Interpretation of Partial Derivatives 81
4.4.3 Partial Derivative of Function of Three Variables 82
4.4.4 Partial Derivatives of Higher Order 83
4.5 Homogeneous Functions 85
4.5.1 Euler’s Theorem on Homogeneous Functions 86
4.6 Summary 90
4.7 Self-Assessment Exercises 92
4.8 Solutions to In-text Exercises 93
4.9 Suggested Readings 94

4.1 Learning Objectives


The learning objectives of this lesson are to:

• understand the concept of partial differentiation.

• learn to determine the partial derivatives of various functions.

• learn the geometric interpretation of partial derivatives.

• use the Euler’s Theorem on homogeneous functions to solve various problems.

77
78 LESSON - 4. PARTIAL DIFFERENTIATION

4.2 Introduction
In this lesson, we will extend the concept of differentiation of function of a single variable
to function of several variables. If a function of more than one variable is differentiated
with respect to one independent variable while keeping other variables as constants, the
derivative we obtain is known as a partial derivative. Partial derivatives have application in
finding maxima or minima of functions of several variables. For the limiting scope of this
book, we will discuss mainly function of two variables. We will also discuss the Euler’s
Theorem on homogeneous function that displays relation between the dependent variable,
independent variable, the partial derivatives of the dependent variable and the order of the
homogeneous function in consideration.

4.3 Function of Two Variables


We are already familiar with a function of a single variable. We now define functions of
two variables. However, the definition can be extended to the functions of more than two
variables.
Definition 4.1. Let D be a subset of R2 ≡ R × R = {(x, y) : x, y ∈ R}}. A real valued
function f on D is a rule that assigns a unique real number z = f (x, y) to each element
(x, y) in D. Here
(i) D is called the domain of the function f.
(ii) The set {f (x, y) : (x, y) ∈ D}is called the range set of f.
(iii) z is the dependent variable and x and y are the independent variables.
Note. The graph of a function of two variables is called a surface.
Example 4.1. Following are some illustrations of functions of two variables:
1. z = f (x, y) = x2 + y 2 with domain D = {(x, y) : −3 ≤ x ≤ 3, −4 ≤ y ≤ 4}.

Figure 4.1: Graph of f (x, y) = x2 + y 2 .


4.4. PARTIAL DERIVATIVES 79

2. z = f (x, y) = 2 sin x + sin y with domain D = {(x, y) : −2π ≤ x, y ≤ 2π} .

Figure 4.2: Graph of f (x, y) = 2 sin x + sin y.

4.4 Partial Derivatives


4.4.1 Partial Derivatives of Function of Two Variables
Consider z = f (x, y) be a function of two variables x and y. If we treat y as a constant,
then z = f (x, y) can be considered as a function of x alone and we can talk about the
derivative of z = f (x, y) with respect to x (keeping y as a constant). Similarly, we can also
talk about the derivative of z = f (x, y) with respect to y (keeping x as a constant)

Definition 4.2. Let z = f (x, y) be a function of two variables x and y. We define

(i) The partial derivative of z = f (x, y) with respect to x at the point (x, y) = (a, b) as

∂f f (a + h, b) − f (a, b)
= fx (a, b) = lim , (4.1)
∂x (a,b)
h→0 h

provided the limit exists.

(ii) The partial derivative of z = f (x, y) with respect to y at the point (x, y) = (a, b) as

∂f f (a, b + k) − f (a, b)
= fy (a, b) = lim , (4.2)
∂y (a,b)
k→0 k

provided the limit exists.

Example 4.2. Consider the function

f (x, y) = 2x2 + 5xy + y 2 .

Find the partial derivatives fx (1, 3) and fy (1, 3).


80 LESSON - 4. PARTIAL DIFFERENTIATION

Solution. Since
f (x, y) = 2x2 + 5xy + y 2 .
Therefore, we have
f (1 + h, 3) − f (1, 3)
fx (1, 3) = lim
h→0 h
[2(1 + h)2 + 5 · (1 + h) · 3 + 32 ] − [2 · 12 + 5 · 1 · 3 + 32 ]
= lim
h→0 h
2
2h + 19h
= lim
h→0 h
= lim (2h + 19)
h→0
= 19

Alternately, to obtain fx (1, 3), we treat y as a constant in f (x, y) and differentiate it with
respect to x. Therefore,

fx (x, y) = 4x + 5y,
=⇒ fx (1, 3) = 4 · 1 + 5 · 3 = 19.

Similarly,
f (1, 3 + k) − f (1, 3)
fy (1, 3) = lim
k→0 k
[2 · 12 + 5 · 1 · (3 + k) + (3 + k)2 ] − [2 · 12 + 5 · 1 · 3 + 32 ]
= lim
h→0 k
2
k + 11k
= lim
k→0 k
= lim (k + 11)
k→0
= 11

Alternately, to obtain fy (1, 3), we treat x as a constant in f (x, y) and differentiate it with
respect to y. Therefore,

fy (x, y) = 5x + 2y,
=⇒ fy (1, 3) = 5 · 1 + 2 · 3 = 11.

Note. We note the following:


1. Equations (4.1) and (4.2) define the partial derivatives of f at the point (a, b). If the
partial derivative of f with respect to x (or y) exists at all points of the domain, then
we define

(i) The partial derivative of z = f (x, y) with respect to x as



∂f f (x + h, y) − f (x, y)
= fx (x, y) = lim , (4.3)
∂x (x,y)
h→0 h
4.4. PARTIAL DERIVATIVES 81

(ii) The partial derivative of z = f (x, y) with respect to y



∂f f (x, y + k) − f (x, y)
= fy (x, y) = lim , (4.4)
∂y (x,y)
k→0 k

where f, fx and fy have common domain.

2. The derivatives in (4.3) and (4.4) are called first order partial derivatives.
2 +y 3
Example 4.3. Consider the function f (x, y) = xex + y 2 . Therefore,

d
fx (x, y) = f (x, y)
dx y=constant
d h x2 +y3 i
2
= xe +y
dx y=constant
x2 +y 3 x2 +y 3
=e + xe · 2x + 0
2 x2 +y 3
= (1 + 2x )e

d
and fy (x, y) = f (x, y)
dy x=constant
d h x2 +y3 i
2
= xe +y
dy x=constant
x2 +y 3 2
= xe · 3y + 2y
x2 +y 3
= 3xy 2 e + 2y

In-text Exercise 4.1.


1. Calculate fx (x, y) and fy (x, y) for the following function:

(i) f (x, y) = (x2 + y 2 ) sin(x3 − y 3 )


(ii) f (x, y) = ln(x2 + y 2 + x)
∂z ∂z
2. Find and for the function z given by the equation
∂x ∂y
yz − ln x = x + y.

[Hint: Differentiate the given equation with respect to x, treating y as a constant to


∂z ∂z
obtain . Similarly, obtain by differentiating the given equation with respect to
∂x ∂y
y by treating x as a constant. ]

4.4.2 Geometric Interpretation of Partial Derivatives


We have studied earlier that for a function of single variable y = f (x), the derivative of
′ 
f (x) at the point x = a namelyf (a) is the slope of the tangent to curve of y = f (x) at
x = a. In a similar manner, we can also relate partial derivatives to slope of the tangents.
82 LESSON - 4. PARTIAL DIFFERENTIATION

Let S be the surface representing the graph of z = f (x, y) in Figure 4.3 and P (a, b, c)
be the point on the surface where c = f (a, b). In this figure C1 represents the curve
z = f (x, b), which is the intersection of the surface S with the vertical plane y = b.
Then it represents a function of the single variable x. Let us denote it by p(x). That is

p(x) = f (x, b). Therefore, fx (a, b) = p (a) is the slope of the tangent T1 to the curve C1 at
the point P .
Similarly C2 represents the curve z = f (a, y), which is the intersection of the surface
S with the horizontal plane x = a. Then it represents a function of the single variable y.

Let us denote it by q(y). That is q(y) = f (a, y). Therefore, fy (a, b) = q (b) is the slope of
the tangent T2 to the curve C2 at the point P .

In short,
∂f
1. fx (a, b) = (a, b) represents the slope of the tangent line to the intersection of the
∂x
graph of f with the plane y = b at the point (a, b).

∂f
2. fy (a, b) = (a, b) represents the slope of the tangent line to the intersection of the
∂y
graph of f with the plane x = a at the point (a, b).

Figure 4.3: Intersection of planes and surface.

4.4.3 Partial Derivative of Function of Three Variables


Let us consider z = f (x, y, t) be function of three variables x, y and t. Then the partial
derivative of f with respect to x (or y or z) is calculated using differentiating the function
4.4. PARTIAL DERIVATIVES 83

f with respect to x (or y or z) while treating the other two variables as constants. The three
first order partial derivatives are denoted by
∂f ∂f ∂f
, and .
∂x ∂y ∂t
Example 4.4. Consider the function

f (x, y, t) = x2 y + xyt + xt3 + yt + x3 sin t.

We have
∂f ∂
x2 y + xyt + xt3 + yt + x3 sin t ,

=
∂x ∂x
= 2xy + yt + t3 + 3x2 sin t,
∂f ∂
x2 y + xyt + xt3 + yt + x3 sin t ,

=
∂y ∂y
= x2 + xt + t,
∂f ∂
x2 y + xyt + xt3 + yt + x3 sin t ,

and =
∂t ∂t
= xy + 3xt2 + y + x3 cos t.
∂f ∂f ∂f
In-text Exercise 4.2. Calculate , and for the following functions:
∂x ∂y ∂t
1. f (x, y) = x2 + y 2 + xyt + ln xy
2. f (x, y) = ext + sin(x2 y + y 2 t + xt2 )
xy 2 + 2t
3. f (x, y) =
x+t

4.4.4 Partial Derivatives of Higher Order


For a functionz = f (x, y) of two variable x and y , the partial derivatives fx and fy may be
constants or functions of x and y. So these functions can also have partial derivatives with
respect to x and y. In this way, we obtain the second order partial derivatives of f with
respect to x and y, defined as follows:
∂ 2f
 
∂ ∂f
(i) fxx = (fx )x = = (Differentiating with respect to x two times
∂x ∂x ∂x2
treating y as a constant)
∂ 2f
 
∂ ∂f
(ii) fyy = (fy )y = = (Differentiating with respect to y two times treating
∂y ∂y ∂y 2
x as a constant)
∂ 2f
 
∂ ∂f
(iii) fxy = (fx )y = = (First differentiating with respect to x treating y
∂y ∂x ∂y∂x
as a constant, then with respect to y treating x as a constant)
84 LESSON - 4. PARTIAL DIFFERENTIATION

∂ 2f
 
∂ ∂f
(iv) fyx = (fy )x = = (First differentiating with respect to y treating x
∂x ∂y ∂x∂y
as a constant, then with respect to x treating y as a constant)
Theorem 4.1 (Equality of Partial Derivatives). Let f be a function of two variables x
and y having continuous second order partial derivatives fxy and fyx , then
fxy = fyx .
Similarly, we can also define third order, forth order and higher order partial derivatives
like
∂ ∂ 2f ∂ 3f
 
(i) fxxx = = ,
∂x ∂x2 ∂x3
∂ ∂ 2f ∂ 3f
 
(ii) fyyx = = ,
∂x ∂y 2 ∂x∂y 2
∂ 3f
 
∂ ∂f
(iii) fxyx = = and so on.
∂x ∂y∂x ∂x∂y∂x
Example 4.5. Consider the function
f (x, y) = x3 y 3 + 2x2 y + 4xy 2 + 2x + 3y − 1.
We have
∂f
= 3x2 y 3 + 4xy + 4y 2 + 2
∂x
∂f
and = 3x3 y 2 + 2x2 + 8xy + 3.
∂y
Therefore,
∂ 2f
 
∂ ∂f ∂
3x2 y 3 + 4xy + 4y 2 + 2 = 6xy 3 + 4y,

2
= =
∂x ∂x ∂x ∂x
2
 
∂ f ∂ ∂f ∂
3x2 y 3 + 4xy + 4y 2 + 2 = 9x2 y 2 + 4x + 8y,

= =
∂y∂x ∂y ∂x ∂y
2
 
∂ f ∂ ∂f ∂ 3 2 2
= 6x3 y + 8x,

2
= = 3x y + 2x + 8xy + 3
∂y ∂y ∂y ∂y
2
 
∂ f ∂ ∂f ∂
3x3 y 2 + 2x2 + 8xy + 3 = 9x2 y 2 + 4x + 8y.

= =
∂x∂y ∂x ∂y ∂x
∂ 2f ∂ 2f
Here, we note that = .
∂y∂x ∂x∂y
2 +y 3
Example 4.6. Consider the function f (x, y) = xex + y 2 . Since we have already calcu-
lated
2 +y 3
fx (x, y) = (1 + 2x2 )ex
2 +y 3
and fy (x, y) = 3xy 2 ex + 2y.
4.5. HOMOGENEOUS FUNCTIONS 85

Therefore,
∂  2 x2 +y 3

fxy = (1 + 2x )e
∂y
2 3
= (1 + 2x2 )ex +y · 3y 2
2 3
= 3(1 + 2x2 )y 2 ex +y
∂  2 3

and fyx = 3xy 2 ex +y + 2y
∂x
2 3 2 3
= 3y 2 ex +y + 3xy 2 · ex +y · 2x
2 +y 3
= 3(1 + 2x2 )y 2 ex

Here also we note that fxy = fyx .

4.5 Homogeneous Functions


Definition 4.3 (Homogeneous Function). A function f of two variables x and y is said to
be a homogeneous function of degree (order) r ∈ R, if

f (αx, αy) = αr f (x, y), α ̸= 0 (4.5)

or y
r
f (x, y) = x g (4.6)
x
y
where g is a function of.
x
Example 4.7. Consider the function

f (x, y) = x5 + 3x2 y 3 + 6xy 4 − 4y 5 .

Therefore, for α ̸= 0

f (αx, αy) = (αx)5 + 3(αx)2 (αy)3 + 6(αx)(αy)4 − 4(αy))5


= α5 x5 + 3x2 y 3 + 6xy 4 − 4y 5


= α5 f (x, y).

Therefore, f (x, y) = x5 + 3x2 y 3 + 6xy 4 − 4y 5 is a homogeneous function of degree 5.


Alternately,

f (x, y) = x5 + 3x2 y 3 + 6xy 4 − 4y 5


y3 y4 y5
 
5
=x 1+3 3 +6 4 −4 5
x x x
  y 3  y 4  y 5 
5
=x 1+3 +6 −4
x x x
y
= x5 g
x
86 LESSON - 4. PARTIAL DIFFERENTIATION
y  y 3  y 4  y 5
where g = 1+3 +6 −4 . Therefore, the given function is a homo-
x x x x
geneous function of degree 5.
x2 + y 2
Example 4.8. Let f (x, y) = . We can write
x3 − y 3

x2 + y 2
f (x, y) =
x3 − y 3
y2
 
2
x 1+ 2
x
=  3

y
x3 1 − 3
x
 y 2
1+
= x−1 xy 3
1−
 y x
−1
=x g
x
 y 2
y 1+
where g = x
y 3
. Hence, the given function is homogeneous of degree −1.
x
1−
x
In-text Exercise 4.3. Verify that the following functions are homogeneous and find out the
degree:
x3 + y 3
1. f (x, y) =
x−y
p
3
x2 − y 2
2. g(x, y) = 2
x + y2

4.5.1 Euler’s Theorem on Homogeneous Functions


Euler’s Theorem displays a relation between the dependent variable, the independent vari-
ables, the partial derivatives of dependent variable with respect to the independent variables
and the degree (order) of the homogeneous function.
Theorem 4.2 (Euler’s Theorem). If z = f (x, y) is a homogeneous function of two vari-
ables x and y of degree r, then
∂f ∂f
x +y = rf (x, y). (4.7)
∂x ∂y
Proof. Since z = f (x, y) is a homogeneous function of degree r. Therefore,
y
f (x, y) = xr g . (4.8)
x
4.5. HOMOGENEOUS FUNCTIONS 87

y
where g is a function of . Therefore, we have
x

∂f y    −y 
r−1 r ′ y
= rx g +x g ·
∂x x x x2
y ′
 y 
= rxr−1 g − xr−2 yg (4.9)
x x
∂f ′
y 1
and = xr g ·
∂y x x

y
= xr−1 g (4.10)
x

Therefore, using (4.9) and (4.10), we have

∂f ∂f h
r−1
y
r−2 ′
 y i
r−1 ′ y
 
x +y = x · rx g − x yg +y·x g
∂x ∂y x x x
y ′
y ′
y
= rxr g − xr−1 yg + xr−1 yg
x
y x x
r
= rx g
x
= rf (x, y)

Example 4.9. Verify that the function

x2 + y 2
f (x, y) =
x3 − y 3

is a homogeneous function of x and y and it satisfies the Euler’s Theorem.

Solution. We have
  y 2 
2
x 1+
x2 + y 2 x
f (x, y) = 3 =   y 3 
x − y3
x3 1 −
x
 y 2
1+
=  x
 y 3 
x 1−
x
 y 2
1+
= x−1 xy 3
1−
x
88 LESSON - 4. PARTIAL DIFFERENTIATION

Therefore, f is a homogeneous function of degree −1. Now we have


∂ x2 + y 2
 
∂f
=
∂x ∂x x3 − y 3
(x3 − y 3 ) · 2x − (x2 + y 2 ) · 3x2
=
(x3 − y 3 )2
2x4 − 2xy 3 − 3x4 − 3x2 y 2
=
(x3 − y 3 )2
−x4 − 2xy 3 − 3x2 y 2
=
(x3 − y 3 )2
and
∂ x2 + y 2
 
∂f
=
∂y ∂y x3 − y 3
(x3 − y 3 ) · 2y − (x2 + y 2 ) · (−3y 2 )
=
(x3 − y 3 )2
2x3 y − 2y 4 + 3x2 y 2 + 3y 4
=
(x3 − y 3 )2
2x3 y + 3x2 y 2 + y 4
=
(x3 − y 3 )2
Therefore,
 4
−x − 2xy 3 − 3x2 y 2
 3
2x y + 3x2 y 2 + 2y 4
 
∂f ∂f
x +y =x +y
∂x ∂y (x3 − y 3 )2 (x3 − y 3 )2
−x5 − 2x2 y 3 − 3x3 y 2 + 2x3 y 2 + 3x2 y 3 + y 5
=
(x3 − y 3 )2
−x5 − x3 y 2 + x2 y 3 + y 5
=
(x3 − y 3 )2
x2 (y 3 − x3 ) + y 2 (y 3 − x3 )
=
(x3 − y 3 )2
(y 3 − x3 )(x2 + y 2 )
=
(x3 − y 3 )2
−(x2 + y 2 )
=
(x3 − y 3 )
= (−1)f (x, y)

Hence, the Euler’s Theorem is satisfied.


In-text Exercise 4.4. Verify Euler’s Theorem for the following functions:
y
1. f (x, y) = x ln
x
2. f (x, y) = 9x3 + 5x2 y + y 3
4.5. HOMOGENEOUS FUNCTIONS 89

x3 + y 3
 
−1
Example 4.10. For z = tan , prove that
x+y

∂z ∂z
x +y = sin 2z.
∂x ∂y

Solution. We have

x3 + y 3
 
−1
z = tan
x+y
  y 3    y 3 
3
x 1+ 1+
x3 + y 3 x 2 x
=⇒ tan z = = h y i =x · h yi . (4.11)
x+y x 1+ 1+
x x
Let tan z = u, then
  y 3 
1+
2 x
u=x · h yi
1+
x
is a homogeneous function of degree 2. Therefore, by the Euler’s Theorem

∂u ∂u
x +y =2·u (4.12)
∂x ∂y

Also,
∂u ∂u ∂z ∂z
= · = sec2 z
∂x ∂z ∂x ∂x
(4.13)
∂u ∂u ∂z ∂z
and = · = sec2 z
∂y ∂z ∂y ∂y
Therefore, from (4.12) and (4.13), we get

∂z ∂z
x · sec2 z + y · sec2 z = 2 tan z
∂x ∂y
∂z ∂z tan z
=⇒ x +y =2 2
∂x ∂y sec z
= 2 sin z cos z
= sin 2z.

x2 +y 2
Example 4.11. For z = e x+y , prove that

∂z ∂z
x +y = z ln z.
∂x ∂y
90 LESSON - 4. PARTIAL DIFFERENTIATION

Solution. We have
x2 +y 2
z=e x+y
  y 2 
1+
x2 + y 2 x
=⇒ ln z = =x· h yi . (4.14)
x+y 1+
x
Let ln z = u, then
  y 2 
1+
x
u=x· h yi
1+
x
is a homogeneous function of degree 1. Therefore, by the Euler’s Theorem
∂u ∂u
x +y =1·u (4.15)
∂x ∂y
Also,
∂u ∂u ∂z 1 ∂z
= · =
∂x ∂z ∂x z ∂x
(4.16)
∂u ∂u ∂z 1 ∂z
and = · =
∂y ∂z ∂y z ∂y
Therefore, from (4.15) and (4.16), we get
1 ∂z 1 ∂z
x· +y· = ln z
z ∂x z ∂y
∂z ∂z
=⇒ x +y = z ln z
∂x ∂y
In-text Exercise 4.5.
 3
x + y3

−1 ∂z ∂z
1. If z = sin , show that x +y = 2 tan z.
x+y ∂x ∂y
 5
x + y5

∂z ∂z
2. If z = ln , show that x + y = 2.
x3 + y 3 ∂x ∂y

4.6 Summary
In this lesson we have discussed the following points:
1. Let D be a subset of R2 ≡ R × R = {(x, y)|x, y ∈ R}}. A real valued function f
on D is a rule that assigns a unique real number z = f (x, y) to each element (x, y)
in D. Here

(i) D is called the domain of the function f.


4.6. SUMMARY 91

(ii) The set {f (x, y) : (x, y) ∈ D} is called the range set of f.


(iii) If z = f (x, y), then z is the dependent variable and x and y are the independent
variables.
2. The graph of a function of two variables represents a surface.
3. Let z = f (x, y) be a function of two variables x and y. We define
(i) The partial derivative of z = f (x, y) with respect to x at the point (x, y) = (a, b)
as
∂f f (a + h, b) − f (a, b)
= fx (a, b) = lim , (4.17)
∂x (a,b)
h→0 h
provided the limit exists.
(ii) The partial derivative of z = f (x, y) with respect to y at the point (x, y) = (a, b)
as
∂f f (a, b + k) − f (a, b)
= fy (a, b) = lim , (4.18)
∂y (a,b)
k→0 k
provided the limit exists.
4. Geometric Interpretation of Partial Derivative:
∂f
(i) fx (a, b) = (a, b) represents the slope of the tangent line to the intersection
∂x
of the graph of f with the plane y = b at the point (a, b).
∂f
(ii) fy (a, b) = (a, b) represents the slope of the tangent line to the intersection
∂y
of the graph of f with the plane x = a at the point (a, b).
5. The second order partial derivatives of f with respect to x and y are defined as
follows:
∂ 2f
 
∂ ∂f
(i) fxx = (fx )x = = (Differentiating with respect to x two times
∂x ∂x ∂x2
treating y as a constant)
∂ 2f
 
∂ ∂f
(ii) fyy = (fy )y = = (Differentiating with respect to y two times
∂x ∂y ∂y 2
treating x as a constant)
∂ 2f
 
∂ ∂f
(iii) fxy = (fx )y = = (First differentiating with respect to x
∂y ∂x ∂y∂x
treating y as a constant, then with respect to y treating x as a constant)
∂ 2f
 
∂ ∂f
(iv) fyx = (fy )x = = (First differentiating with respect to y
∂x ∂y ∂x∂y
treating x as a constant, then with respect to x treating y as a constant)
6. Equality of second order partial derivatives: Let f be a function of two variables
x and y having continuous second order partial derivatives fxy and fyx , then
fxy = fyx .
92 LESSON - 4. PARTIAL DIFFERENTIATION

7. Homogeneous function:A function f of two variables x and y is said to be a homo-


geneous function of degree (order) r, r ∈ R if

f (αx, αy) = αr f (x, y), α ̸= 0

or y
f (x, y) = xr g
x
y
where g is a function of .
x
8. Euler’s Theorem on homogeneous functions: If f (x, y) is a homogeneous function
of two variables x and y of degree r, then

∂f ∂f
x +y = rf (x, y).
∂x ∂y

4.7 Self-Assessment Exercises


1. Find the partial derivatives fx and fy for the following functions:

(i) f (x, y) = 2x2 y + y 3 − 3xy 2

(ii) f (x, y) = sin(x2 − y 2 ) cos(x2 + y 2 )


x +xy 2
(iii) f (x, y) = ee

(iv) f (x, y) = ln(x3 + 2x2 y + y 2 )

x1/4 + y 1/4
(v) f (x, y) =
x1/5 + y 1/5
x(x3 − y 3 )
(vi) f (x, y) =
x3 + y 3
2. Find the partial derivatives fxx , fxy , fyx and fyy for the following functions:
2
(i) f (x, y) = xy 2 + ex+y sin x

(ii) f (x, y) = sin(x2 + y 3 ) cos(x + y)


xy + x2
(iii) f (x, y) =
x2 + xy 3
 2
x + y2

(iv) f (x, y) = ln
x+y

(v) f (x, y) = esin(x+2y+xy)

3. Verify the Euler’s Theorem for the following functions:


4.8. SOLUTIONS TO IN-TEXT EXERCISES 93

1
(i) z =
x2 + y 2
y
(ii) z = x3 ln
x
x(x3 − y 3 )
(iii) z =
x3 + y 3

x1/4 + y 1/4
(iv) z =
x1/5 + y 1/5
x2 + y 2
 
−1 ∂f ∂f
4. Using Euler’s Theorem, show that if f (x, y) = sin , then x +y = 0.
x+y ∂x ∂y
x+y ∂z ∂z cos z
5. If cos z = √ √ , then prove that x +y + = 0.
x
 + y  ∂x ∂y 2 sin z
−1 x+y ∂z ∂z sin 2z
6. If z = cot √ √ , then prove that x +y + = 0.
x+ y ∂x ∂y 4

4.8 Solutions to In-text Exercises


Exercise 4.1

1. (i) fx (x, y) = 2x sin(x3 − y 3 ) + 3x2 (x2 + y 2 ) cos(x3 − y 3 ),


fy (x, y) = 2y sin(x3 − y 3 ) − 3y 2 (x2 + y 2 ) cos(x3 − y 3 ).
2x + 1 2y
(ii) fx (x, y) = 2 2
, fy (x, y) = 2
x +y +x x + y2 + x
∂z x2 + 1 ∂z 1−z
2. = , = .
∂x xy ∂x y
Exercise 4.2
1 1
1. fx (x, y, t) = 2x + yt + , fy (x, y, t) = 2y + xt + , ft (x, y, t) = xy
x y
2. fx (x, y, t) = text + (2xy + t2 ) cos(x2 y + y 2 t + xt2 ),
fy (x, y, t) = (x2 + 2yt) cos(x2 y + y 2 t + xt2 ),
ft (x, y, t) = xext + (y 2 + 2xt) cos(x2 y + y 2 t + xt2 ).

t(y 2 − 2) 2xy x(2 − y 2 )


3. fx (x, y, t) = , f y (x, y, t) = , f t (x, y, t) = .
(x + t)2 (x + t)2 (x + t)2
Exercise 4.3

1. Degree = 2
−4
2. Degree =
3
94 LESSON - 4. PARTIAL DIFFERENTIATION

4.9 Suggested Readings


1. Narayan, S. & Mittal, P. K.(2019). Differential Calculus. S. Chand Publishing.
2. Anton, H., Bivens, I. C., & Davis, S. (2015). Calculus: Early Transcendentals. John
Wiley & Sons.
Unit 2: Mean Value Theorems

Unit Overview
This unit on Mean Value Theorems is in continuation to the unit 1. The topics discussed
in unit 1, such as limits, continuity and differentiability help us to prove important Mean
Value Theorems, which have wide range of applications almost in every field. The other
topics discussed such as Indeterminate forms, extrema of a function and Taylor’s series
expansions add to the values of this unit. This unit is further divided into four lesson.

In Lesson 5, we have discussed the two mean theorems, namely Rolle’s Theorem and La-
grange’s Mean Value Theorem. Basic applications of these theorems to establish some
important inequalities and to check the monotonic behavior of functions are discussed.

In Lesson 6, the concept of convergence of a sequence and series and the concept of ex-
trema of a function are discussed with applications.

In Lesson 7, we introduce some indeterminate forms on limits. The study of these indeter-
minate forms help us to evaluate many limits which are otherwise difficult to be evaluated.

In Lesson 8, we discussed Cauchy’s Mean Value Theorem and the Taylor’s Theorem. Tay-
lor’s series expansion and Maclaurin’s series expansions of some functions are also dis-
cussed in this lesson.

The topics discussed in the above lessons are supported by examples, in-text exercises and
self-assessment exercises.

95
Lesson - 5

Mean Value theorem


Ms. Setu Rani
Satyawati College (E)
University of Delhi

Structure
5.1 Learning Objectives 96
5.2 Introduction 97
5.3 Rolle’s Theorem 97
5.3.1 Algebraic interpretation of Rolle’s Theorem 99
5.3.2 Geometrical interpretation of Rolle’s Theorem 99
5.4 Lagrange’s Mean Value Theorem 103
5.4.1 Geometrical Interpretation of Lagrange’s Mean Value Theorem 104
5.5 Applications of Mean Value Theorem to monotonic functions and
inequalities 110
5.5.1 Monotone functions 111
5.5.2 Inequalities 115
5.6 Summary 117
5.7 Self-Assessment Exercises 118
5.8 Solutions to In-text Exercises 119
5.9 Suggested Readings 120

5.1 Learning Objectives


The learning objectives of this lesson are to:

• learn Rolle’s Theorem and its applications.

• learn Lagrange’s Mean Value Theorem and its applications.

• understand the concept of monotonicity of functions.

96
5.2. INTRODUCTION 97

5.2 Introduction
In the previous lessons, we have learnt about the concept of continuity and differentiabil-
ity of functions. In this lesson, we will learn how these concepts can be used to establish
some standard theorems named as Rolle’s Theorem and Lagrange’s Mean Value Theorem.
Rolle’s Theorem is a special case of Mean Value Theorem. We will learn about the ge-
ometrical interpretation of both these theorems. The Mean Value Theorem is one of the
important theorem in calculus, as it lays the foundation to many important results. We look
at some of its applications to check the monotonic behavior of functions and to establish
some inequalities at the end of this lesson.

5.3 Rolle’s Theorem


This theorem is named after Michel Rolle a French mathematician. It states that if any real
valued differentiable function have equal values at two distinct points, then it must have
at least one stationary point (a point at which the derivative of the function become zero)
somewhere between them.

Mathematically Rolle’s Theorem can be stated as follows:

Theorem 5.1 (Rolle’s Theorem). Let f (x) be a function defined on the closed interval
[a, b], such that

1. f (x) is continuous on the closed interval [a, b],

2. f (x) is differentiable on the open interval (a, b),

3. f (x) has same value at x = a and b i.e. f (a) = f (b),

then there exists at-least one point c in (a, b) such that f ′ (c) = 0.

Proof. We know that a continuous function on a closed interval is bounded and attains its
bounds therein. Since the given function f (x) is continuous on the closed interval [a, b],
therefore it is bounded on [a, b]. Let m and M denote the bounds of f (x) on [a, b]

i.e. m ≤ f (x) ≤ M ∀ x ∈ [a, b]. (5.1)

Since, f attains its bounds, so there exist points d, c in [a, b], such that

f (d) = m and f (c) = M. (5.2)

From equation (5.1) and (5.2), we get

f (d) ≤ f (x) ≤ f (c) ∀ x ∈ [a, b]. (5.3)

Case I. Let m = M.
98 LESSON - 5. MEAN VALUE THEOREM

Then from equation (5.1), we obtain f (x) = m ∀ x ∈ [a, b]. That is, f (x) is a constant
function. Therefore, f ′ (x) = 0 ∀ x ∈ [a, b]. Thus, the theorem is true in this case.

Case II. Let M ̸= m.

Since f (a) = f (b) and M ̸= m, so at least one of the number M and m is different from
f (a) and f (b).
Suppose M ̸= f (a) and M ̸= f (b).
That is f (c) ̸= f (a) and f (c) ̸= f (b), then using equation (5.2) we get c ̸= a and c ̸= b,
where c ∈ [a, b].
Thus c ∈ (a, b) and so by given condition (ii), f ′ (c) exists
i.e.
f ′ (c) = Lf ′ (c) = Rf ′ (c) (5.4)
From (5.3),
f (x) − f (c) ≤ 0 ∀ x ∈ [a, b]. (5.5)
f (x) − f (c)
Now, Lf ′ (c) = limx→c− ≤ 0, using (5.5). (Notice that x → c− ⇒ x < c,
(x − c)
i.e., x − c < 0. )
⇒ f ′ (c) ≥ 0, using (5.4). (5.6)
f (x) − f (c)
Now, Rf ′ (c) = limx→c+ ≤ 0, using (5.5). (Notice that x → c+ ⇒ x > c,
x−c
i.e., x − c > 0.)
⇒ f ′ (c) ≤ 0, using (5.4). (5.7)
Hence, from (5.6) and (5.7), we get f ′ (c) = 0, c ∈ (a, b). This proves the theorem.

Remark. If any of the conditions of Rolle’s Theorem is not satisfied, then the conclusion
may not hold. This is illustrated in the following examples.
Example 5.1. Consider the function
(
2x, 0 ≤ x < 1
f (x) =
3, x=1
Here, f (x) is not continuous at x = 1. Therefore, the condition (i) of the hypothesis of
Rolle’s Theorem is violated The conclusion also does not hold as f ′ (x) ̸= 0 at any point
x ∈ (0, 1).
Example 5.2. Consider the function
f (x) = |x| x ∈ [−1, 1].
Here f (x) is not differentiable at 0 ∈ (−1, 1). Therefore, the condition (ii) of the hypothesis
of Rolle’s Theorem is violated The conclusion also does not hold as f ′ (x) ̸= 0 at any point
x ∈ (−1, 1) excluding x = 0.
5.3. ROLLE’S THEOREM 99

Example 5.3. Consider the function

f (x) = x, x ∈ [1, 2].

Here, f (1) = 1 and f (2) = 2, which means f (1) ̸= f (2). Therefore, the condition (iii)
of the hypothesis of Rolle’s Theorem is violated The conclusion also does not hold as
f ′ (x) = 1 ̸= 0 at any point x ∈ (1, 2).

Thus, Rolle’s Theorem does not hold true if any one of its of the condition is excluded.

5.3.1 Algebraic interpretation of Rolle’s Theorem

Algebraically, Rolle’s Theorem implies that, Between any two zeros of a function satisfying
the conditions of the Rolle’s Theorem, there exists at least one zero of its derivative.

Let y = f (x) is any function defined on closed interval [a, b] and satisfying the conditions
of the Rolle’s Theorem on [a, b] and f (a) = f (b) = 0. That is, a and b are zeroes of f (x).
Then from Rolle’s Theorem we can conclude that there exist c belongs to open interval
(a, b) such that f ′ (c) = 0 or c is the zero of the function f ′ (x).

5.3.2 Geometrical interpretation of Rolle’s Theorem

If y = f (x) is any real valued function defined on [a, b] such that

(i) It is continuous on [a, b] (i.e. continuous curve can be drawn from x = a to x = b.)

(ii) It is differentiable on (a, b) (i.e. unique tangent can be drawn at each point x ∈ (a, b).)

(iii) f (a) = f (b) (i.e. ordinates at the end points are equal),

then there exist at least one point P (c, f (c)), a < c < b on the curve y = f (x) such that
tangent at the point P is parallel to x-axis.
100 LESSON - 5. MEAN VALUE THEOREM

Figure 5.1: Geometrical interpretation of Rolle’s Theorem.

Figure 5.2: Geometrical interpretation of Rolle’s Theorem

In the above figure 5.2 f ′ (x) = 0 at two points x = c1 and x = c2 while, the Rolle’s
5.3. ROLLE’S THEOREM 101

Theorem guarantees the existence of at least one such point.



Example 5.4. Verify Rolle’s Theorem for f (x) = 1 − x2 on the interval [−1, 1].
1
Solution. Given f (x) = (1 − x2 ) 2 , x ∈ [−1, 1]. Therefore, f (x) is a defined real function
on [−1, 1]. To check the applicability of Rolle’s Theorem we check the validity of all the
three conditions of the hypothesis.

(i) Since f (x) is an algebraic function in x and every algebraic function is continuous.
Therefore, f (x) is continuous in [−1, 1].

(ii) Also,
1 1 −x
f ′ (x) = (1 − x2 ) 2 −1 . − 2x = √ , x ∈ (−1, 1).
2 1 − x2
⇒ f (x) is differentiable on (−1, 1).

(iii) f (−1) = 0 = f (1)

Thus, f (x) satisfies all the condition of Rolle’s Theorem. Now to verify the conclusion, we
have
−x
f ′ (x) = √ , x ∈ (−1, 1)
1 − x2
⇒ f ′ (x) = 0 at x = 0.

Therefore, there exist a point c = 0 ∈ (−1, 1) such that f ′ (c) = 0. Hence, the given
function f (x) satisfies all the conditions of the hypotheses as well as the conclusion of the
Rolle’s Theorem.

Example 5.5. Verify Rolle’s Theorem for f (x) = log(x2 + 2) − log 3 in [−1, 1]

Solution. Here,

f (x) = log(x2 + 2) − log 3, x ∈ [−1, 1]


2x
∴ f ′ (x) = 2 , x ∈ (−1, 1)
x +2
Therefore

(i) f (x) is continuous on [−1, 1], as f (x) is the difference of the continuous function
log(x2 + 2) and log 3.

2x
(ii) f (x) is derivable on (−1, 1) and f ′ (x) = .
x2+2
(iii) f (−1) = log(1 + 2) − log 3 = 0 = f (1)
∴ f (−1) = f (1).
102 LESSON - 5. MEAN VALUE THEOREM

Thus, all the conditions of Rolle’s Theorem are satisfied. Hence, there exists at-least one
point c ∈ (−1, 1), such that f ′ (c) = 0. We have
2x
f ′ (x) =
x2 + 2
2c
f ′ (c) = 2 = 0 for c = 0 ∈ (−1, 1).
c +2
Hence, Rolle’s Theorem is verified.
Example 5.6. Show that between any two roots of the equation ex cos x = 1, there exists
at least one root of the equation ex sin x − 1 = 0.
Solution. Let α and β be the two roots of ex cos x = 1. Let us define a function f (x) as

f (x) = e−x − cos x for all x ∈ [α, β]. (5.8)

Then
1. f (x) is continuous on [α, β], as cos x and e−x are continuous on [α, β].
2. f (x) is differentiable on [α, β] and
3. f (α) = f (β) = 0, from (5.8).
Also f ′ (x) = −e−x + sin x. Thus, all the conditions of Rolle’s Theorem are satisfied by
f (x) on [α, β]. Therefore, there exist at least one c ∈ (α, β) such that f ′ (c) = 0.

i.e., sin c − e−c = 0


or ec sin c − 1 = 0

i.e. c is a root of the equation ex sin x − 1 = 0. Thus, there exist at least one root of the
equation ex sin x − 1 = 0 in [α, β].
In-text Exercise 5.1. Solve the following questions:
1. Discuss the applicability of Rolle’s Theorem for the following functions on the indi-
cated intervals:
(i) f (x) = 2x2 − 5x + 3 on [1, 3]
(
−4x + 5 if 0 ≤ x ≤ 1
(ii) f (x) = on [0, 2].
2x − 3 if 1 < x ≤ 2
(iii) f (x) = ex (sin x − cos x) on π4 , 5π
 
4
.
(iv) f (x) = |x − 1| on [−2, 2].
2. Using Rolle’s Theorem, find a point on the curve y = 16 − x2 , x ∈ [−1, 1], where
the tangent is parallel to x- axis.
3. If the Rolle’s Theorem holds for the function f (x) = x3 + bx2 + cx, x ∈ [1, 2] at the
4
point . Find the values of b and c.
3
5.4. LAGRANGE’S MEAN VALUE THEOREM 103

5.4 Lagrange’s Mean Value Theorem


Lagrange’s Mean Value Theorem (LMV Theorem) is a further extension of Rolle’s The-
orem. In this theorem the third condition that f (a) = f (b) is removed. It concludes that
there exists at least one point c ∈ (a, b), such that the tangent line at P (c, f (c)) on the curve
y = f (x) is parallel to the secant line joining the points A(a, f (a)) and B(b, f (b)) on the
curve. This theorem is also known as the Fundamental Mean Value Theorem. It is stated
as following:
Theorem 5.2. Let f be a function defined on the closed interval [a, b], b > a such that it
satisfies the following conditions:
(i) f (x) is continuous in closed interval [a, b],
(ii) f (x) is derivable in open interval (a, b).
Then, there exist at-least one point c ∈ (a, b) such that
f (b) − f (a)
f ′ (c) =
b−a

Figure 5.3: Graphical representation of Lagrange’s Mean Value Theorem

Proof. Consider a function


ϕ(x) = f (x) + Ax, (5.9)
where A is a constant and we chose it in such a way that ϕ(a) = ϕ(b). Now
ϕ(a) = ϕ(b)
=⇒ f (a) + Aa = f (b) + Ab
=⇒ f (b) − f (a) = A(a − b)
 
f (b) − f (a)
=⇒ A=− . (5.10)
b−a
104 LESSON - 5. MEAN VALUE THEOREM

Now, the function ϕ(x) in (5.9), where A is given by (5.10), satisfies the conditions:
1. ϕ(x) is continuous on [a, b], as both f (x) and Ax are continuous.
2. ϕ(x) is differentiable on [a, b], as both f (x) and Ax are differentiable on (a, b) and
ϕ′ (x) = f ′ (x) + A.
3. ϕ(a) = ϕ(b), by the choice of A.
Thus, all the conditions of Rolle’s Theorem are satisfied by ϕ(x) on [a, b]. Hence, there
exist at-least a point c ∈ (a, b) such that ϕ′ (c) = 0. Now
ϕ(x) = f (x) + Ax
=⇒ ϕ′ (x) = f ′ (x) + A
=⇒ ϕ′ (c) = f ′ (c) + A = 0 =⇒ A = −f ′ (c). (5.11)
From equation (5.10) and (5.11), we get
f (b) − f (a)
f ′ (c) = .
b−a
This proves the theorem.

5.4.1 Geometrical Interpretation of Lagrange’s Mean Value Theorem


The conditions of Lagrange’s Mean Value Theorem implies that
(i) f (x) is continuous in the closed interval [a, b] That is, the curve y = f (x) is smooth
from the point A(a, f (a)) to the point B(b, f (b)) and it has no break.
(ii) f (x) is derivable in the open interval (a, b) That is, the tangent at each point of (a, b)
is unique and non-vertical.

Figure 5.4: Geometrical interpretation of Lagrange’s Mean Value Theorem


5.4. LAGRANGE’S MEAN VALUE THEOREM 105

Let the end points A(a, f (a)) and B(b, f (b)) are joined by the chord AB and it makes an
angle ψ with x-axis. Then from the triangle ARB the slope of the chord is
BR f (b) − f (a)
tan ψ = = . (5.12)
AR b−a
Also from Lagrange’s Mean Value Theorem, we have
f (b) − f (a)
f ′ (c) = , c ∈ (a, b). (5.13)
b−a
Thus, from 5.12 and 5.13, we have

f ′ (c) = tan ψ

slope of the tangent at P (c, f (c)) = Slope of the chord (secant) AB


Hence, in geometrical form Lagrange’s Mean Value Theorem can be stated as ‘if there is
a continuous curve between the points A and B on the curve y = f (x) having a unique
tangent at each point between A and B, then there is at-least one point on the curve between
A and B, where the tangent is parallel to the chord AB.
Note. 1. If in the hypothesis of Lagrange’s Mean Value Theorem one more condition
is added that is the value of the function at the end points are same i.e. f (a) = f (b)
Then by the conclusion of Lagrange’s mean value theorem, there exist a point c ∈
(a, b) such that
f (b) − f (a)
f ′ (c) =
b−a
Since f (b) = f (a), therefore f ′ (c) = 0 which is the conclusion of the Rolle’s Theo-
rem. Thus, Rolle’s Theorem is a special case of Lagrange’s Mean Value Theorem.
2. We also obtain from Lagrange’s Mean Value Theorem that the average rate of change
of a function on an interval is equal to the actual rate of change of the function at some
point of the interval.
3. Lagrange’s Mean Value Theorem may not hold if any one condition of the hypothesis
is not satisfied. This is illustrated in the following example.
Example 5.7. Check the validity of the Lagrange’s Mean Value Theorem for the following
function f (x), x ∈ [1, 2].

2
x
 if 1 < x < 2
f (x) = 2 if x = 1

1 if x = 2

Solution. Let, the Lagrange’s Mean Value theorem be applicable for the given function.
Then there exist at-least one point c in (1, 2), such that
f (2) − f (1)
= f ′ (c),
2−1
106 LESSON - 5. MEAN VALUE THEOREM

1−2
=⇒ = 2c
2−1
or
−1
=⇒ c= ∈
/ (1, 2).
2
Hence, the conclusion of the Lagrange’s Mean Value theorem does not hold for the given
function. It may be noticed that the given function is not continuous at x = 1 and x = 2 as

lim f (x) = lim x2 = 1 ̸= f (1),


x→1+ x→1+

lim f (x) = lim x2 = 4 ̸= f (2)


x→2− x→2−

Note. There may be some functions for which one or both the conditions of the hypothesis
of Lagrange’s theorem are not true but still a point c ∈ (a, b) can be obtained for which
the conclusion of the theorem holds true. In other words the conditions of the Lagrange’s
theorem are only sufficient but not necessary for the conclusion. This is illustrated in the
next example.
Example 5.8. Consider a function

1
 if 0 ≤ x < 1/4
f (x) = x if 1/4 ≤ x < 1/2

(x/2) + 1 if 1/2 ≤ x ≤ 2

Then the function f is neither continuous in [0, 2] nor derivable in (0, 2), but at the point
x = 1/2, the conclusion of the theorem holds. Since,
x 1
lim− = 1 and lim+ = lim1 = .
x→ 41 x→ 14 x→ 4 2 8

=⇒ lim1 f (x) does not exist.


x→ 4
1
=⇒ f (x) is not continuous at x = and hence on [0, 2].
4
1 1 1
Also, left hand derivative at x = is 0 and right hand derivative at x = is .
4 4 2
1
=⇒ f (x) is not differentiable at x = and hence on (0, 2). But
4
f (2) − f (0) 2−1 1
= = ,
2−0 2−0 2
1 1
and left and right derivative at x = is . Hence
2 2
 
f (2) − f (0) 1
= f′
2−0 2
1
That is the conclusion of LMV Theorem holds true at x = .
2
5.4. LAGRANGE’S MEAN VALUE THEOREM 107
 
π 5π
Example 5.9. Verify Lagrange’s Mean Value Theorem for f (x) = sin x in , .
2 2
 
π 5π
Solution. Here f (x) = sin x. Then f (x) is a real function defined in , . Also,
2 2
 
π 5π
(i) since, lim f (x) = lim sin x = sin a = f (a) ∀ a ∈ , . Therefore, f (x) is
x→a
 x→a 2 2
π 5π
continuous on , .
2 2
 
′ π 5π
. Therefore, f (x) is derivable in π2 , 5π

(ii) f (x) = cos x for x ∈ , 2
.
2 2
Thus, both the conditions of Lagrange’s Mean Value Theorem are satisfied. Hence, there
exists at-least one point c ∈ (a, b) such that
f (b) − f (a) π 5π
= f ′ (c), where a = , b = (5.14)
b−a 2 2
We have,

f (x) = sin x,
π  π
f (a) = f = sin = 1,
2  2
5π 5π
f (b) = f = sin = 1,
2 2
Also f ′ (x) = cos x
=⇒ f ′ (c) = cos c

Therefore, from (5.14), we get


1−1
5π = cos c
− π2
2
cos c = 0
 
3π π 5π
c= ∈ , .
2 2 2
Hence, Lagrange’s Mean Value Theorem is verified.
Example 5.10. Find the point ‘c’ of the Lagrange’s Mean Value Theorem if f (x) = (x −
1)(x − 2)(x − 3) and a = 0, b = 4.
Solution. Here f (x) = (x − 1)(x − 2)(x − 3) = x3 − 6x2 + 11x − 6.
(i) Since, f (x) is a polynomial in x, therefore it is continuous in [0, 4].

(ii) Also, f ′ (x) = 3x2 − 12x + 11 which exists for all x ∈ (0, 4). Thus, f (x) is derivable
in (0, 4).
108 LESSON - 5. MEAN VALUE THEOREM

Since, both the condition of Lagrange’s Mean Value Theorem are satisfied. Hence there
must exist at-least one point c ∈ (a, b) such that
f (b) − f (a)
= f ′ (c), where a = 0, b = 4 (5.15)
b−a
Now,
f ′ (x) = 3x2 –12x + 11
=⇒ f ′ (c) = 3c2 –12c + 11
f (b) = f (4) = (4–1)(4 − 2)(4 − 3) = 3.2.1. = 6
and f (a) = f (0) = (−1)(−2)(−3) = −6.
Substituting all these values in (5.15), we get
6 − (−6)
= 3c2 − 12c + 11
4−0
3c2 − 12c + 8 = 0
√ √
12 ± 144 − 96 6±2 3
i.e. c = = .
6 3
Both the above value of c lie in between (0, 4). We note that, Lagrange’s Mean Value
Theorem guarantees the existence of at least one such point. Here, we have two points for
which Lagrange’s Mean Value Theorem is satisfied.
Example
√ 5.11. Use Lagrange’s Mean Value Theorem to determine a point on the curve
y = x2 − 4 defined in [2, 4], where the tangent is parallel to the chord joining the end
points of the curve.

Solution. Given the function y = f (x) = x2 − 4, it is defined for x ∈ [2, 4]. Also,
1. f (x) is continuous on [2, 4].
1 x
2. f ′ (x) = √ · 2x = √ , which exists for all x ∈ (2, 4). Therefore, f (x)
2 x2 − 4 x2 − 4
is derivable in (2, 4).
Thus, both the conditions of Lagrange’s Mean Value Theorem are satisfied. Hence, there
exists at-least one point c ∈ (a, b) such that
f (b) − f (a)
= f ′ (c), where a = 2, b = 4 (5.16)
b−a
Now,
x
f ′ (x) = √
x2 − 4
c
=⇒ f ′ (c) = √
c2 − 4
√ √
f (b) = f (4) = 42 − 4 = 2 3

and f (a) = f (2) = 22 − 4 = 0.
5.4. LAGRANGE’S MEAN VALUE THEOREM 109

Therefore, substituting all these values in (5.16), we get



2 3−0 c
=√
4−2 c2 − 4
c √
=⇒ √ = 3
c2 − 4
=⇒ 2c2 = 12

i.e. c = ± 6.
√ √ √ √ √
Now, c = + 6 ∈ (2, 4). Also for, x = √ 6,√y = x2 − 4 = 6 − 4 = 2. Thus, the
tangent to the given curve at the point ( 6, 2) is parallel to the chord joining the end
points of the curve for [2, 4].

Theorem 5.3. If f satisfies all the conditions of Lagrange’s Mean Value Theorem and if
f ′ (x) = 0 ∀ x ∈ (a, b), then f is constant on [a, b].

Proof. Let x1 and x2 be any two points in [a, b] such that x1 < x2 . Let f ′ (x) = 0 ∀x ∈
(a, b). Then by the Lagrange’s Mean value theorem, we have

f (x2 ) − f (x1 )
= f ′ (c) = 0, for some c ∈ (x1 , x2 ).
x2 − x1

=⇒ f (x1 ) = f (x2 ) ∀ x1 , x2 ∈ [a, b] and so f is constant on [a, b].

Alternative form of Lagrange’s Mean Value Theorem


Let us take h = b − a. Then the interval [a, b] becomes [a, a + h]. A point c in [a, a + h] can
be written in the form a + θh, where 0 < θ < 1. Hence Lagrange’s theorem can be stated
as follows:
Let f be a function defined on [a, a + h], such that

(i) f is continuous in [a, a + h]

(ii) f is derivable in [a, a + h]

Then there exists at least one real number θ, 0 < θ < 1, such that :
f (a + h) − f (a)
f ′ (a + θh) = ,
h
equivalently, f (a + h) = f (a) + hf ′ (a + θh).

Example 5.12. Prove that for any quadratic function px2 + qx + r, the value of θ in the
Lagrange’s Theorem is always 1/2 irrespective of the values of p, q, r, a, h.

Solution. Let f (x) = px2 + qx + r and the interval is [a, a + h]. Since f (x) is a polynomial
function, therefore
110 LESSON - 5. MEAN VALUE THEOREM

(i) f (x) is continuous on [a, a + h]

(ii) f (x) is derivable on (a, a + h)


Therefore, by the Lagrange’s Mean Value Theorem there exist at least one θ ∈ (0, 1),
satisfying
f (a + h) = f (a) + hf ′ (a + θh)
i.e.
p(a + h)2 + q(a + h) + r = pa2 + qa + r + h(2p(a + θh) + q)
⇒ p(a + h)2 –a2 + qh = 2aph + 2pθh2 + gh
⇒ ph(2a + h) = 2aph + 2pθh2
⇒ θ = 1/2
Since, θ is independent of p, q, r, a, h. So, the value of θ is 1/2, irrespective of the values of
p, q, r, a and h.
In-text Exercise 5.2. Solve the following questions:
1. Examine the applicability of Lagrange’s Mean Value Theorem for the following func-
tions:

(i) (
2 + x3 if x ≤ 1
f (x) =
3x if x > 1
on [−1, 2].
(ii) f (x) = log x on [1, e].
(iii) f (x) = (x − 1)(x − 2)(x − 3) on [0, 4].
(iv) f (x) = x3 − 5x2 − 3x on [1, 3].

2. If a and b are distinct real numbers, show that there exist a real number c between a
and b such that
a2 + ab + b2 = 3c2 .

3. Show that Lagrange’s Mean Value Theorem is not applicable to the function f (x) =
1
x
on [−1, 1].

4. Find a point on the parabola y = (x − 4)2 , where the tangent is parallel to the chord
joining (4, 0) and (5, 1).

5.5 Applications of Mean Value Theorem to monotonic


functions and inequalities
In this section, we will study the application of Mean Value Theorem for finding the mono-
tonic functions and establish inequalities using the concept of monotone functions.
5.5. APPLICATIONS OF MEAN VALUE THEOREM TO MONOTONIC FUNCTIONS AND INEQUALIT

5.5.1 Monotone functions


Definition 5.1. A function f defined on a interval [a, b] is said to be monotonically in-
creasing or simply increasing, if for x1 , x2 in [a, b]

f (x1 ) ≤ f (x2 ) whenever x1 ≤ x2 .

Definition 5.2. A function f defined on a interval [a, b] is said to be strictly increasing, if


for x1 , x2 in [a, b]
f (x1 ) < f (x2 ) whenever x1 < x2 .

Definition 5.3. A function f defined on a interval [a, b] is said to be monotonically de-


creasing or simply decreasing, if for x1 , x2 in [a, b]

f (x1 ) ≥ f (x2 ) whenever x1 ≤ x2 .

Definition 5.4. A function f defined on a interval [a, b] is said to be strictly decreasing, if


for x1 , x2 in[a, b]
f (x1 ) > f (x2 ) whenever x1 < x2 .

Definition 5.5. A function f defined on a interval [a, b] is said to be monotone or strictly


monotone, if f is either an increasing (or strictly increasing) function or a decreasing (or
strictly decreasing) function.

Figure 5.5: Graphs for monotonic functions: Figure (a) represent monotonically increasing
function while Figure (b) represent monotonically decreasing function.

Theorem 5.4. (Necessary and sufficient condition) Let f : (a, b) → R be a differentiable


function on (a, b). Then

1. f is increasing on (a, b) if and only if f ′ (x) ≥ 0 for all x ∈ (a, b).

2. f is decreasing on (a, b) if and only if f ′ (x) ≤ 0 for all x ∈ (a, b).


112 LESSON - 5. MEAN VALUE THEOREM

Proof. (i) Necessary condition

Consider an arbitrary point x0 ∈ (a, b). If the function f is increasing on (a, b), then by
definition, we can write;
∀x ∈ (a, b) : x ≥ x0 =⇒ f (x) ≥ f (x0 );
∀x ∈ (a, b) : x ≤ x0 =⇒ f (x) ≤ f (x0 ).
By above result, we can write as
f (x) − f (x0 )
≥ 0, where x ̸= x0 (5.17)
x − x0
In the limit as x → x0 , the left hand side of the inequality is equal to the derivative of the
function at the point x0 , that is
f (x) − f (x0 )
lim = f ′ (x0 ) ≥ 0, (5.18)
x→x0 x − x0
This relation is valid for any x0 ∈ (a, b).

Sufficient condition
Let x1 and x2 be any two points of [a, b] such that x1 ≤ x2 . Let f ′ (x) ≥ 0 ∀x ∈ (a, b).
Then by the Lagrange’s Mean Value Theorem,

f (x2 ) − f (x1 )
= f ′ (c), for some c ∈ (x1 , x2 ).
x2 − x1
⇒ f (x2 ) − f (x1 ) = (x2 − x1 )f ′ (c),
Since, f ′ (c) ≥ 0 and x2 − x1 ≥ 0, therefore f (x2 ) − f (x1 ) ≥ 0. Hence, f (x2 ) ≥ f (x1 )
when x2 ≥ x1 , x1 , x2 ∈ (a, b). Thus, f is increasing on (a, b).

(ii) By proceeding as in part (i), we can show that f is decreasing on (a, b) if and only if
f ′ (x) ≤ 0 for all x ∈ (a, b).
Example 5.13. Find the intervals in which the function f (x) = 2x3 + 9x2 + 12x + 20 is
(i) increasing (ii) decreasing.
Solution. We have
f (x) = 2x3 + 9x2 + 12x + 20.
∴ f ′ (x) = 6x2 + 18x + 12 = 6(x2 + 3x + 2).
(i) For f (x) to be increasing, we must have f ′ (x) ≥ 0
⇒ 6(x2 + 3x + 2) ≥ 0,
⇒ (x2 − 3x + 2) ≥ 0,
⇒ (x + 1)(x + 2) ≥ 0,
⇒ x ≤ −2 or x ≥ −1,
⇒ x ∈ (−∞, −2] ∪ [−1, ∞),
5.5. APPLICATIONS OF MEAN VALUE THEOREM TO MONOTONIC FUNCTIONS AND INEQUALIT

Figure 5.6: Signs of f ′ (x) for different values of x.

So, f (x) is increasing on (−∞, −2] ∪ [−1, ∞).

(ii) For f (x) to be decreasing, we must have f ′ (x) ≤ 0

⇒ 6(x2 + 3x + 2) ≤ 0,
⇒ (x2 + 3x + 2) ≤ 0,
⇒ (x + 1)(x + 2) ≤ 0,
⇒ −2 ≤ x ≤ −1,

Figure 5.7: Signs of f ′ (x) for different values of x.

So, f (x) is decreasing on [−2, −1].

x3
Example 5.14. Find the intervals in which the function f (x) = x4 − is increasing or
3
decreasing.

Solution. We have
x3
f (x) = x4 −
3
∴ f ′ (x) = 4x3 − x2 = x2 (4x − 1)
(i) For f (x) to be increasing, we must have f ′ (x) ≥ 0

⇒ x2 (4x − 1) ≥ 0
⇒ (4x − 1) ≥ 0 and x ̸= 0
 
1 1
⇒ 4x ≥ 1 and x ̸= 0 ⇒ x ≥ ⇒ x ∈ , ∞ .
4 4
 
1
So, f (x) is increasing on , ∞ .
4
114 LESSON - 5. MEAN VALUE THEOREM

(ii) For f (x) to be decreasing, we must have f ′ (x) ≤ 0


⇒ x2 (4x − 1) ≤ 0
⇒ (4x − 1) ≤ 0 and x ̸= 0 [∵ x2 > 0]
 
1 1
⇒ 4x ≤ 1 and x ̸= 0 ⇒ x ≤ and x ̸= 0 ⇒ x ∈ (−∞, 0) ∪ 0, .
4 4
 
1
So, f (x) is decreasing on (−∞, 0) ∪ 0, .
4
Note. The above mentioned theorem 5.4 is stated regarding monotonic functions. Similar
theorem, as stated below, holds for strictly monotonic functions.
Theorem 5.5. (Necessary and sufficient condition) Let f : (a, b) → R be a differentiable
function on (a, b). Then
1. f is strictly increasing on (a, b) if and only if f ′ (x) > 0 for all x ∈ (a, b).
2. f is strictly decreasing on (a, b) if and only if f ′ (x) < 0 for all x ∈ (a, b).
Example 5.15. Find the intervals in which f (x) = 2 log(x − 2) − x2 + 4x + 1 is strictly
increasing or strictly decreasing.
Solution. We have, f (x) is well defined for all x > 2.
Now,
f (x) = 2 log(x − 2) − x2 + 4x + 1
′ 2 −2x2 + 8x − 6 −2(x − 1)(x − 3)
⇒ f (x) = − 2x + 4 = = .
x−2 x−2 x−2
For f (x) to be increasing, we must have f ′ (x) > 0

−2(x − 1)(x − 3)
or >0
x−2
(x − 1)(x − 3)
or < 0.
x−2
(x − 1)(x − 3)
Since, x > 2 we have x − 2 > 0 and x − 1 > 0. Therefore, < 0 when
x−2
x − 3 < 0 That is, when x < 3.
Thus, f ′ (x) > 0 when x ∈ (2, 3). =⇒ f (x) is strictly increasing on (2, 3).

For f (x) to be decreasing, we must have f ′ (x) < 0

−2(x − 1)(x − 3)
or <0
x−2
(x − 1)(x − 3)
or >0
x−2
That is when x−3 > 0 or x > 3, as x−1 > 0 and x−2 > 0. So, f (x) is strictly decreasing
on (3, ∞).
5.5. APPLICATIONS OF MEAN VALUE THEOREM TO MONOTONIC FUNCTIONS AND INEQUALIT

5.5.2 Inequalities
Here, we will establish some important inequalities by using Lagrange’s Mean Value The-
orem and also by using the concept of monotone functions.
Example 5.16. Use Mean Value Theorem to prove that
1 + x < ex < 1 + xex ∀x > 0
Solution. Consider f (x) = ex , x ∈ [0, x]. Here, f is continuous on [0, x] and derivable on
(0, x), therefore by the Mean Value Theorem there exists some c ∈ (0, x) such that
f (x) − f (0)
= f ′ (c)
x−0
ex − e0
or = ec
x−0
ex − 1
or = ec . (5.19)
x

Now 0 < c < x


=⇒ e0 < ec < ex , as ex is an increasing function on(0, ∞).
or 1 < ec < ex
ex − 1
or 1 < < ex [Using 5.19]
x
or x < ex − 1 < x · ex
=⇒ 1 + x < ex < 1 + xex , ∀x > 0.
Example 5.17. Using Lagrange’s Mean Value Theorem, show that
x
< loge (1 + x) < x, x > 0
1+x
Solution. Let f (x) = loge (1 + x), x > 0
1
=⇒ f ′ (x) = .
1+x
Then, f is continuous in [0, x] and derivable in (0, x). Therefore, by Lagrange’s Mean
Value Theorem, there exists θ ∈ (0, 1), such that
f (x) − f (0)
= f ′ (θx)
x−0
or
x
loge (1 + x) = [∵ f (0) = loge 1 = 0] (5.20)
1 + θx
Now 0 < θ < 1 and x > 0 ⇒ θx < x
1 1
⇒ 1 + θx < 1 + x ⇒ >
1 + θx 1+x
x x x x
⇒ > ⇒ < (5.21)
1 + θx 1+x 1+x 1 + θx
116 LESSON - 5. MEAN VALUE THEOREM

Again 0 < θ < 1 and x > 0 ⇒ 1 < 1 + θx


1 x
⇒ <1⇒ <x (5.22)
1 + θx 1 + θx
From (5.21) and (5.22), we obtain
x x
< <x (5.23)
1+x 1 + θx
Now, from (5.20) and (5.23), we obtain
x
< loge (1 + x) < x.
1+x
Example 5.18. Prove that tan x > x whenever 0 < x < π2 .

Solution. Let c be any real number such that 0 < c < π2 . Let us consider the function

f (x) = tan x − x ∀ x ∈ [0, c]

Then, f is continuous on [0, c] as well as derivable on (0, c). Now,

f ′ (x) = sec2 x − 1 = tan2 x > 0 for 0 < x < c.

Thus, f is strictly increasing in [0, c]

⇒ f (c) > f (0) for c > 0.

But f (0) = 0. Therefore f (c) > 0 ⇒ tan c − c > 0. Since c is any real number such that
0 < c < π2 , therefore
π
tan x − x > 0, or tan x > x whenever 0 < x < .
2
Example 5.19. Show that, for all x > 0

ex > 1 + x

Solution. We define the function f (x) as

f (x) = ex − (1 + x), x > 0.

Then f (x) is a differentiable function for x > 0. Let us define another function g(x) as

g(x) = f ′ (x) = ex − 1 ∀ x > 0.

=⇒ g ′ (x) = ex > 0 for all x > 0.


⇒ g is a strictly increasing function for x > 0. Therefore

x > 0 ⇒ g(x) > g(0)


5.6. SUMMARY 117

i.e. ex − 1 > e0 − 1
or ex − 1 > 0
⇒ f ′ (x) > 0 ∀ x > 0.
⇒ f is an increasing function of x.
∴ x > 0 ⇒ f (x) > f (0)
i.e. ex − (1 + x) > e0 − (1 + 0) = 0
⇒ ex > (1 + x) ∀ x > 0.
In-text Exercise 5.3. Solve the following questions:
1. Separate the interval in which f (x) = x3 + 8x2 + 5x − 2 is increasing or decreasing.
2. Use Lagrange’s Mean Value Theorem to show that
(b − a) sec2 a < tan b − tan a < (b − a) sec2 b,
where 0 < a < b < π2 .
3. Using Lagrange’s Mean Value Theorem, prove that
e x > 1 + x + x2 .

4. Show that x(sin x)−1 increases for 0 < x < π2 .


x
5. Find the interval in which f (x) = is increasing or decreasing.
log x
6. Find the value of k for which f (x) = kx3 − 9kx2 + 9x + 3 is increasing on R.

5.6 Summary
We have discussed following topics in this lesson:
1. Rolle’s Theorem is a particular case of Lagrange’s Mean Value Theorem.
2. Lagrange’s Mean Value Theorem: If f be a function defined in the closed interval
[a, b] such that it is continuous in closed interval [a, b], derivable in open interval
(a, b), then there exist at-least one point c ∈ (a, b) such that
f (b) − f (a)
f ′ (c) = .
b−a
3. We obtain from the Lagrange’s Mean Value Theorem that the average rate change
in the value of the function in an interval is equal to the actual rate of change of the
function at some point in the interval.
4. Applications of the Mean Value Theorem.
5. Monotone functions and their applications to establish some inequalities.
118 LESSON - 5. MEAN VALUE THEOREM

5.7 Self-Assessment Exercises


2
1. Let f (x) = x 3 , a = −1, b = 8. Show that there is no real number c ∈ (a, b) such
that
f (b) − f (a)
f ′ (c) = .
b−a
2. If f : [−5, 5] → R is differentiable and if f ′ (x) doesn’t vanish anywhere, then prove
that f (−5) ̸= f (5).

3. Discuss the applicability of Rolle’s Theorem for the following functions on the indi-
cated intervals:

(i) f (x) = x(x − 4)2 on [0, 4].


(ii) f (x) = sin4 x + cos4 x on 0, π2 .
 

(iii) f (x) = [x] for −1 ≤ x ≤ 1, where [x] denotes the greatest integer not exceed-
ing x.

4. Verify that on the curve f (x) = ax2 + bx + c, the chord joining the points (p, f (p))
p+q
and (q, f (q)) is parallel to the tangent at the point x = .
2
5. Discuss the validity of the Rolle’s Theorem for

f (x) = (x − c)m (x − d)n

in [a, b]; where m, n being positive integers.

6. Use Rolle’s Theorem to show that the equation

x3 + 4x − 1 = 0,

has exactly one real root.

7. Verify Lagrange’s Mean Value Theorem for the following functions:

(i) f (x) = sin x − sin 2x − x on [0, π].


(ii) f (x) = |x| on [−5, 5].

(iii) f (x) = x2 − 4 on [2, 4].
2
(iv) f (x) = 1 − (x − 1) 3 on [0, 2].

8. Using Lagrange’s Mean Value Theorem show that


x3
(i) x − < sin x < x.
6
(ii) | sin x − sin y| ≤ |x − y| for all x, y ∈ R.

9. Let f and g be differentiable function on [0, 1] such that f (0) = 2, g(0) = 0, f (1) = 6
and g(1) = 2. Show that there exist c ∈ (0, 1) such that f ′ (c) = 2g ′ (c).
5.8. SOLUTIONS TO IN-TEXT EXERCISES 119

10. Show that the function 3x3 − 9x2 + 9x + 7 is strictly increasing in every interval.

11. Find the intervals in which the function f (x) = 2x3 + 9x2 + 12x + 20 is (i) increasing
(ii) decreasing.

12. Find the interval in which the given function

f (x) = (x4 + 6x3 + 17x2 + 32x + 32)e−x

is increasing and decreasing.

13. Using Lagrange’s Mean Value Theorem show that


x
2
< tan−1 x < x for x > 0.
1+x

14. Find the intervals in which the function f given by


4 sin x − 2x − x cos x
f (x) = , 0 ≤ x ≤ 2π.
2 + cos x
is increasing and decreasing.

15. Prove that the function

f (x) = x3 − 3x2 + 3x − 100

is increasing on R.

16. Establish the Jordan’s Inequality


x π π
1< ≤ for 0 < x ≤ .
sin x 2 2

17. Establish the Bernoulli’s inequality

(1 + x)p ≥ 1 + px for x > −1 and p > 1.

5.8 Solutions to In-text Exercises


Exercise 5.1
1. (i) Not applicable
(ii) Not applicable
(iii) Rolle’s Theorem is applicable at the point is c = π.
(iv) Not applicable

2. (0, 16)

3. b = −5, c = 8.
120 LESSON - 5. MEAN VALUE THEOREM

Exercise 5.2

1. (i) Since the function f (x) is continuous and differential at [−1, 2], hence

the Mean
5
Value Theorem is applicable. The value of c is obtained as c = ± 3 .
(ii) The Mean Value Theorem is applicable and the value of c is obtained as c =
e − 1.
(iii) The Mean Value Theorem is applicable and the value of c is obtained as c = 3.
(iv) The Mean Value Theorem is applicable and the value of c is obtained as c = 37 .

2. Applying Lagrange’s Mean Value Theorem to f (x) = x3 in [a, b], the required result
can be obtained.

3. As the function is not continuous and differentiable at [−1, 1].


 
9 1
4. , .
2 4
Exercise 5.3

1. The given function is increasing on (−∞, −5], − 13 , ∞ and decreasing on −5, − 13 .


   

5. The given function is increasing on (e, ∞), and decreasing on (0, e) − {1}.

6. f (x) is increasing on R, if k ∈ 0, 31 .


5.9 Suggested Readings


1. Narayan, Shanti (Revised by Mittal, P. K.). Differential Calculus. S. Chand, Delhi,
2019.

2. Prasad, Gorakh (2016). Differential Calculus (19th ed.) Pothishala Pvt. Ltd. Alla-
habad.

3. Thomas Jr., George B., Weir, Maurice D.,Hass, Joel (2014). Thomas Calculus (13th
ed.). Pearson Education, Delhi. Indian Reprint 2017.
Lesson - 6

Extremum and Convergence of Series


Ms. Setu Rani
Satyawati College (E)
University of Delhi

Structure
6.1 Learning Objectives 121
6.2 Introduction 122
6.3 Extremum of a Function 122
6.3.1 Local Maxima and Local Minima 123
6.3.2 Global Maximum and Global Minimum 124
6.3.3 A Necessary condition for Local Extrema 124
6.3.4 How to find Maxima and Minima of a Function 125
6.3.5 Absolute Maximum and Absolute Minimum in a closed interval 130
6.3.6 Applications of Maxima and Minima 131
6.4 Sequences 132
6.4.1 Bounded sequence 134
6.4.2 Convergence of a Sequence 134
6.4.3 Non-Convergent Sequences 136
6.5 Infinite Series 139
6.6 Convergence and Divergence of an Infinite Series 141
6.7 Summary 145
6.8 Self-Assessment Exercises 146
6.9 Solutions to In-text Exercises 147
6.10 Suggested Readings 149

6.1 Learning Objectives


The learning objectives of this lesson are to:
• use differentiation to locate the stationary points of a function.

121
122 LESSON - 6. EXTREMUM AND CONVERGENCE OF SERIES

• distinguish whether the stationary points are the points of maxima, minima or the
points of inflexion.

• understand the difference between local and global maxima and minima.

• to impart the knowledge of convergence of sequences and summation of series.

• to understand the concept of the sequence of partial sums in order to understand the
convergence of series.

6.2 Introduction
In Lesson 5, we have learnt about various applications of differentiation. In this lesson,
we will use differentiation to find the maximum and minimum values of differentiable
functions useful for solving some applied problems. The terms maxima and minima refer
to extreme values of a function, that is, the maximum and minimum values that the function
attains. Also, we will learn here about the convergence of sequence and series of real
numbers.

6.3 Extremum of a Function


Let y = f (x) be a function of real variable defined in an interval [a, b]. The extremum of
f (x) in [a, b] is the extreme value of f (x) in [a, b]. That is, it is either the maximum value
(maxima) or the minimum value (minima) of f (x) in [a, b]. Geometrically, maxima and
minima of a function are its peaks and valleys, as shown in the following figure 6.3.

Figure 6.1: Maxima and Minima of a function.

The maxima and minima of a function are of two types,

1. Local Maxima and Local Minima

2. Absolute (Global) Maximum and Absolute (Global) Minimum


6.3. EXTREMUM OF A FUNCTION 123

6.3.1 Local Maxima and Local Minima


Local maxima and minima are the maxima and minima of the function which arise in
a particular interval. Local maxima would be the value of a function at a point in a
particular interval for which the values of the function near that point are always less than
the value of the function at that point. Whereas local minima would be the value of the
function at a point where the values of the function near that point are greater than the value
of the function at that point. It is possible for a function to have as many local maxima and
minima as it needs.

Definition 6.1. (Local Maxima) Let y = f (x) be a function defined on [a, b]. Then f (x) is
said to attain a local maximum at x = c, if there exist a neighbourhood of c (c − δ, c + δ) ⊆
[a, b] such that

f (x) ≤ f (c) ∀ x ∈ (c − δ, c + δ).

In this case, f (c) is called the local maximum value of f (x) at x = c.

Definition 6.2. (Local Minima) Let y = f (x) be a function defined on [a, b]. Then f (x) is
said to attain a local minimum at x = c if there exist a neighbourhood of c (c − δ, c + δ) ⊆
[a, b], such that

f (x) ≥ f (c) ∀ x ∈ (c − δ, c + δ).

In this case, f (c) is called the local minimum value of f (x) at x = c.

Note. 1. A local maxima (or local minima) is also known as a relative maxima (or
relative minima).

2. A function f (x) defined in a given domain [a, b] may possess many local maxima
and local minima as shown in the following figure 6.3.1.

Figure 6.2: Local maxima at c1 , c3 and Local minima at c2 and c4 .


124 LESSON - 6. EXTREMUM AND CONVERGENCE OF SERIES

6.3.2 Global Maximum and Global Minimum


The highest point of a function within the entire domain is known as the absolute maxi-
mum of the function whereas the lowest point of the function within the entire domain of
the function, is known as the absolute minimum of the function. There can only be one
absolute maximum of a function and one absolute minimum of the function over the en-
tire domain. The absolute maximum and minimum of the function are also called as the
global maximum and global minimum of the function. The Global (Absolute) maximum of
a function f (x) defined on [a, b] is the greatest value of f (x) in [a, b]. Similarly, the Global
(Absolute) minimum of a function f (x) defined on [a, b] is the least value of f (x) in [a, b].
Precisely, we have the following definitions:

Definition 6.3. (Absolute Maximum) Let f (x) be a real function defined on an interval
I = [a, b]. Then, f (x) is said to attain the global maximum at x = c, if

f (x) ≤ f (c) ∀ x ∈ I.

Here, f (c) is called the global maximum value of f (x) in the interval I.

Definition 6.4. (Absolute Minimum) Let f (x) be a real function defined on an interval
I = [a, b]. Then, f (x) is said to attain the global minimum at x = c, if

f (x) ≥ f (c) ∀ x ∈ I.

Here, f (c) is called the global minimum value of f (x) in the interval I.

Figure 6.3: (Global maximum and Global minimum of a function.)

Note. The Global extrema of a function f (x) defined on [a, b] occurs either at the points of
local extrema or at the end points a, b of [a, b].

6.3.3 A Necessary condition for Local Extrema


Let y = f (x) be a differentiable function on (a, b). If f (x) has a local extremum at x =
c ∈ (a, b), then f ′ (c) = 0.
6.3. EXTREMUM OF A FUNCTION 125

Note. (i) The necessary condition stated above holds for a differentiable function. How-
ever, a local extremum may occur at a point c at which f (x) is not differentiable (see
Figure 6.3.4).

(ii) A point c ∈ [a, b], such that f ′ (c) = 0 or f ′ (c) does not exist, is called a critical
point. If f ′ (c) = 0, then c is called a stationary point.

(iii) The condition cited above is not a sufficient condition.

Example 6.1. Consider the function f (x) = x3 , x ∈ R. Then at x = 0 function is differen-


tiable and derivative is 0. Clearly f (x) is differentiable function and f ′ (x) = 3x2 , x ∈ R.
=⇒ f ′ (0) = 0.
But f (x) = x3 is neither local maximum nor local minimum at x = 0 as shown in figure
6.3.3.

Figure 6.4: (Graph of y = x3 )


.

6.3.4 How to find Maxima and Minima of a Function


We have the following tests to find the local extrema of a differentiable function.

1. First Order Derivative Test

2. Second-Order Derivative Test

First Order Derivative Test


The first order derivative test gives a sufficient condition for f (x) to have local extremum
at x = c. It is stated as following:

Let f (x) be a differentiable function on (a, b). Then


126 LESSON - 6. EXTREMUM AND CONVERGENCE OF SERIES

(i) if f ′ (x) changes sign from positive to negative as x passes through c from left to
right, then f (x) has a local maximum at x = c.

(ii) if f ′ (x) changes sign from negative to positive as x passes through c from left to
right, then f (x) has a local minimum at x = c.

(iii) if f ′ (x) does not changes sign as x passes through c, then f (x) has no local extrema
at x = c.

Figure 6.5: Graph for first derivative test.

Algorithm for Finding the Local Maxima and Local Minima of a Func-
tion Using First Derivative Test
dy
Step-1 : For the function y = f (x), find = f ′ (x).
dx
dy
Step-2 : Put = 0 and solve this equation for x. Let c1 , c2 , ..., cn be the roots of this
dx
equation, then these points are the stationary points.

Step-3 : Choose one stationary point c1 and check the change in the sign of the function as
x passes through c1 from left to right.

dy
(a) If changes its sign from positive to negative as x passes through c1 , then the
dx
function attains a local maximum at x = c1 .
6.3. EXTREMUM OF A FUNCTION 127

dy
(b) If changes its sign from negative to positive as x passes through c1 , then the
dx
function attains a local minimum at x = c1 .

Step-4 : Repeat the process for all other vales of x = c2 , c3 , ..., cn .

Example 6.2. Find all the points of local maxima and minima of the function

f (x) = x3 − 6x2 + 9x − 8.

Hence, find the corresponding local maximum and minimum values.


Solution. Let y = f (x) = x3 − 6x2 + 9x − 8. Then,
dy
= f ′ (x) = 3(x2 − 4x + 4) = 3(x − 1)(x − 3).
dx
The stationary points of f (x) are given by f ′ (x) = 0. Thus,
dy
= f ′ (x) = 0 ⇒ x = 1, 3.
dx
Now,

if x < 1, then (x − 1) < 0 and x − 3 < 0 =⇒ f ′ (x) > 0;


if 1 < x < 3, then (x − 1) > 0 and x − 3 < 0 =⇒ f ′ (x) < 0 and
if x > 3, then (x − 1) > 0 and x − 3 > 0 =⇒ f ′ (x) > 0.

+ − +

−∞ 1 3 ∞

Signs of f ′ (x) for different values of x

Clearly, f ′ (x) changes sign from positive to negative as x passes through 1, therefore, x = 1
is a point of local maxima. The corresponding local maximum value is f (1) = −4.

Also, f ′ (x) changes sign from negative to positive as x passes through 3. So, x = 3 is a
point of local minimum and the corresponding local minimum value is f (3) = −8.
Example 6.3. Find the points at which f given by

f (x) = (x − 2)4 (x + 1)3

has (i) local maxima (ii) local minima (iii) no local extremum.
Solution. We have,
f (x) = (x − 2)4 (x + 1)3
⇒ f ′ (x) = 4(x − 2)3 (x + 1)3 + 3(x − 2)4 (x + 1)2
128 LESSON - 6. EXTREMUM AND CONVERGENCE OF SERIES

⇒ f ′ (x) = (x − 2)3 (x + 1)2 (7x − 2).


⇒ f ′ (x) = (x − 2)2 (x + 1)2 (x − 2)(7x − 2).

Now, for stationary points


2
f ′ (x) = 0 ⇒ x = −1, 2, .
7
2
=⇒ The stationary points are x = −1, 2 and . Since (x − 2)2 (x + 1)2 is always positive,
7
therefore the sign of f ′ (x) depends upon the sign of (x − 2)(7x − 2). The changes in sign
2
of f ′ (x) as x increases through −1, and 2 are given by
7
f ′ (x) > 0 if x < −1,
2
f ′ (x) > 0 if − 1 < x < ,
7
2
f ′ (x) < 0 if < x < 2,
7
and f ′ (x) > 0 if x > 2.

+ + − +

−∞ −1 2 2 ∞
7

Signs of f ′ (x) for different values of x.


2
Therefore, f ′ (x) changes its sign from positive to negative as x passes through . So,
7
2
x = is a point of local maximum.
7
f ′ (x) changes its sign from negative to positive as x passes through 2. So, x = 2 is a point
of local minimum.
Since, there is no change in the sign of f ′ (x) as increases through −1. Therefore, no local
extremum exist at x = −1.
In-text Exercise 6.1. Solve the following questions:
1. Find all the points of local maxima and local minima of f (x) = x3 − 6x2 + 12x − 8.
2. Find the local maxima and local minima of the function f (x) = sin x + cos x, 0 <
x < π2 using the first derivative test.
3. Find all the points of local maxima and local minima of f (x) = cos x, 0 < x < π
using the first derivative test.
4. Find the points at which the function f (x) = (x − 1)(x + 2)2 has (i) local maxima
(ii) local minima (iii) no local extremum.
6.3. EXTREMUM OF A FUNCTION 129

Second-Order Derivative Test


Let y = f (x) be a function defined on (a, b) and f ′′ (x) exists at each x ∈ (a, b). Then,
1. f (x) has a local maximum at x = c ∈ (a, b), if f ′ (c) = 0 and f ′′ (c) < 0.

2. f (x) has a local minimum at x = c ∈ (a, b), if f ′ (c) = 0 and f ′′ (c) > 0.

3. The test fails if f ′ (c) = 0 and f ′′ (c) = 0.


Example 6.4. Examine the following function for local maximum and minimum values

f (x) = x5 − 5x4 + 5x3 − 1.

Solution. Given

f (x) = x5 − 5x4 + 5x3 − 1.


=⇒ f ′ (x) = 5x4 − 20x3 + 15x2 .

Now,

f ′ (x) = 0
=⇒ 5x4 − 20x3 + 15x2 = 0
=⇒ 5x2 (x − 1)(x − 3) = 0
=⇒ x = 0, 1, 3.

Therefore, the stationary (critical) points are x = 0, 1 and 3. Now,

f ′′ (x) = 20x3 − 60x2 + 30x

For x = 1, f ′′ (1) = 20 − 60 + 30 = −10 < 0.


For x = 3, f ′′ (3) = 20(3)3 − 60(3)2 + 30(3) = 540 − 540 + 90 = 90 > 0.

Therefore, by the Second Order Derivative Test, x = 1 is a point of local maximum and
x = 3 is a point of local minimum. Also f ′′ (0) = 0, therefore, the Second Order Derivative
test fails for x = 0.
In-text Exercise 6.2. Solve the following questions:
1. Find all the points of local maxima and minima of the function f (x) = 2x3 − 21x2 +
36x − 20. Also, find the corresponding maximum and minimum values.

2. Show that the function f (x) = x3 + x2 + x + 1 doesn’t has a point of local maxima
and local minima.

3. Find the points of local maxima and minima for the following functions

(i) f (x) = (x − 1)(x + 2)2 .



(ii) f (x) = x + 1 − x, x ≤ 1.
130 LESSON - 6. EXTREMUM AND CONVERGENCE OF SERIES

(iii) f (x) = sin x + cos x, 0 < x < π2 .


(iv) f (x) = 2 cos x + x, 0 < x < π.
4. Find the maximum profit that a company can make, if the profit function is given as
P (x) = 24x − 18x2 + 41.
5. If f (x) = a log |x| + bx2 + x has extreme values at x = −1 and at x = 2, then find
the value of constants a and b.
 x
1 1
6. Show that the maximum value of is e( e ) .
x

6.3.5 Absolute Maximum and Absolute Minimum in a closed interval


Let y = f (x) be a function defined on a closed interval [a, b]. Let f ′ (x) exists and it is
continuous at x ∈ (a, b). In order to find the Absolute (Global) extrema of f (x) in [a, b],
we first find the local extrema of f (x) in (a, b) and then find f (a) and f (b). Then
(i) the absolute maximum of f (x) in [a, b] = max { local maxima of f (x) in (a, b), f (a),
f (b). }
(ii) the absolute minimum of f (x) in [a, b] = min { local minima of f (x) in (a, b), f (a),
f (b). }
Example 6.5. Find the largest and smallest values of the polynomial x3 − 18x2 + 96x in
the interval [0, 9].
Solution. The given polynomial function

f (x) = x3 − 18x2 + 96x

is differentiable on (0, 9) and

f ′ (x) = 3x2 − 36x + 96.

For the critical points we have

f ′ (x) = 3x2 − 36x + 96 = 0


=⇒ (x − 8)(x − 4) = 0
=⇒ x = 4, 8 ∈[0, 9].

Therefore, x = 4, 8 are the points of local maxima and local minima. Now, to find the
absolute extrema of f (x) in [0, 9], we consider the values of f (x) at the points of local
extrema (i.e. x = 4, 8) as well as the values f (a) and f (b) at the end points a = 0, b = 9.

f (0) = 0
f (4) = (4)3 − 18(4)2 + 96(4) = 160
f (8) = (8)3 − 18(8)2 + 96(8) = 128
f (9) = (9)3 − 18(9)2 + 96(9) = 135.
6.3. EXTREMUM OF A FUNCTION 131

Therefore, the absolute minima is smallest value of the given polynomial occurring at x = 0
and the largest value of the given polynomial is occurring at x = 4. Thus the largest value
is 160 and smallest value is 0.
In-text Exercise 6.3. Solve the following questions:
1. Find the maximum (largest) and minimum (smallest) values of f (x) = sin x in the
interval [π, 2π].
2. Find the absolute maximum and absolute minimum values of the function f (x) =
x2 − 2x + 4 = 0 in the interval [−3, 1].
3. Find both the maximum and minimum values of the f (x) = 2x3 − 15x2 + 36x + 1
on the interval [1, 5].
4. Find the global extrema of the given function f (x) = x+sin 2x in the interval [0, 2π].

6.3.6 Applications of Maxima and Minima


In the following section, we shall apply the theory of maxima and minima to solve practical
problems involving the use of the same. For example to maximize the area, volume, profit
etc.
Example 6.6. Show that all the rectangles with a given perimeter, the square has the largest
area.
Solution. Let x and y be the lengths of two sides of the rectangle of fixed parameter P, and
let A be its area. Then,
P = 2(x + y) (6.1)
and
A = xy (6.2)
Substituting the value of y from (6.1) into (6.2), we get
   
P Px 2
A = xy = x −x = −x
2 2

d2 A
 
dA P
=⇒ = − 2x and = −2.
dx 2 dx2
dA
The critical points of A are given by = 0.
dx
 
P
∴ − 2x = 0 =⇒ P = 4x =⇒ 2x + 2y = 4x =⇒ x = y.
2
Also,
d2 A
= −2 < 0 at x = y.
dx2
Hence A is maximum when x = y i.e. the rectangle is a square.
132 LESSON - 6. EXTREMUM AND CONVERGENCE OF SERIES

Example 6.7. Show that the height of an cylinder of given surface area and greatest volume
is equal to the radius of its base.
Solution. Let r be the radius of the circular base, h be the height, S be the surface area and
V the volume of the cylinder. Therefore,

S = πr2 + 2πrh, (6.3)

and

V = πr2 h (6.4)

Since surface is given, there S is constant and V is a variable. Also, h, r, are variables.
Substituting the value of h, as obtained from (6.3), in (6.4), we get
S − πr2 Sr − πr3
   
2
V = πr = , (6.5)
2πr 2
which gives V in terms of single variable r. Now,
S − 3πr2
 
dV
= , (6.6)
dr 2
r
S
is 0 when r = . Thus V has only one stationary value. As V must be positive, we

have
r
S
Sr − πr3 > 0 i.e. Sr > πr3 or r < .
π
r r
S S
Thus r varies in the interval (0, ). Now V = 0 for the points r = 0 and and is
π r π
S
positive for every other admissible value of x. Hence V is greatest for r = .

Substituting this value of r in (6.3), we get
S
S − πr2 S − π · 3π
h= = q
2πr 2π 3π S

r
S
=

Hence, for a cylinder of greatest volume and given surface h = r.

6.4 Sequences
Sequences occur frequently in analysis, and they appear in many contexts. While we are
all familiar with sequences, it is useful to have a formal definition.
6.4. SEQUENCES 133

Definition 6.5. A sequence of real number is defined as a function F : N → R, where N is


the set of natural number and R is the set of real numbers. A sequence may be written as

< f1 , f2 , f3 ..., fn , ... > or < fn > or (fn ).

The real numbers f1 , f2 , f3 ..., fn , ... are called the terms or elements of the sequence.
f1 is called the first term, f2 is called the second term,...,fn is called the nth term of the
sequence < fn > . Analogous definitions can be given for the sequence of natural numbers,
integers, etc. In this lesson, we shall consider only sequences of real numbers.

Example 6.8. Following are the sequences of real numbers:

1. < n >=< 1, 2, 3, 4, ... >

2. < n2 >=< 1, 4, 9, 16, ... >

3. < (−1)n >=< −1, 1, −1, 1, ... >

n 1 2 3 4
4. < >=< , , , , ... >
n+1 2 3 4 5

5. < 1 + (−1)n >=< 0, 2, 0, 2, ... >

From the above examples, we can observe that in a sequence all the terms can be distinct
or repeating. Also the sequence has always an infinite number of elements.

Definition 6.6 (Range of a Sequence). The set of all distinct elements of a sequence is
called the range set of the given sequence. The range set of the sequence < an > is the set
{an : n ∈ N}.

Example 6.9. Range sets of the sequences in Example 6.8 are:

1. {1, 2, 3, 4, ...}

2. {1, 4, 9, 16, ...}

3. {−1, 1}

1 2 3 4
4. { , , , , ...}
2 3 4 5

5. {0, 2}

Thus, it can be observed that the range set of a sequence may be finite or infinite.
134 LESSON - 6. EXTREMUM AND CONVERGENCE OF SERIES

6.4.1 Bounded sequence


Definition 6.7. A sequence < an > is said to be bounded above, if there exists a real
number M such that an ≤ M ∀ n ∈ N and the real number M is called an upper bound
of the sequence < an > .

Definition 6.8. A sequence < an > is said to be bounded below, if there exists a real
number M0 such that an ≥ M0 ∀ n ∈ N and the real number M0 is called a lower bound
of the sequence < an > .

Definition 6.9. A sequence which is bounded above as well as bounded below is called a
bounded sequence. Eventually, < an > is bounded if there exist two real numbers M0 and
M such that

M0 ≤ an ≤ M ∀ n ∈ N

Example 6.10. 1. The sequence < n >=< 1, 2, 3, 4, ... > is bounded below by 1 but it
is not bounded above.

2. The sequence < n2 >=< 1, 4, 9, 16, ... > is bounded below by 1 but not bounded
above.

3. The sequence < (−1)n >=< −1, 1, −1, 1, ... > is bounded, −1 being a lower bound
and 1 is an upper bound.
n 1 2 3 4 1
4. The sequence < >=< , , , , ... > is bounded below by but it is not
n+1 2 3 4 5 2
bounded above.

5. The sequence < 1 + (−1)n >=< 0, 2, 0, 2... > is bounded below by 0 and bounded
above by 2.

6.4.2 Convergence of a Sequence


A fundamental concept in mathematics is that of convergence. Consider the sequences
listed in Example 6.8 and observe the way how a sequence < an > vary as n becomes
larger and larger.

Example 6.11. 1. < n >=< 1, 2, 3, 4, ... >. In this sequence, the terms becomes larger
and larger and tends to +∞ as n → +∞ .

2. < n2 >=< 1, 4, 9, 16, ... >. In this sequence also the terms becomes larger and
larger and tends to +∞ as n → +∞.
n 1 2 3 4
3. < >=< , , , , ... >. In this sequence the terms come closer and closer
n+1 2 3 4 5
n
to 1 as n becomes larger and larger. We write < >→ 1 as n → ∞.
n+1
6.4. SEQUENCES 135

4. < 1 + (−1)n >=< 0, 2, 0, 2... >. In this sequence, the terms of the sequence
oscillates with values 0 and 2, and does not come closer to any number as n becomes
larger and larger.
Now, we make the precise definition of a convergent sequence of real numbers.
Definition 6.10. A sequence < an > in R is said to converge to a real number a if for every
ϵ > 0, there exists positive integer k (in general depending on ϵ) such that

|an − a| < ϵ, ∀ n ≥ k.

The number a is then called the limit of the sequence < an > and < an > is called a
convergent sequence.
Note. 1. If < an > converges to a, then we denote the convergence by writing lim <
n→∞
an >= a, or < an >→ a as n → ∞ or sometimes simply we write an → a.

2. The inequality
|an − a| < ϵ ∀ n ≥ k
is also written as

a − ϵ < an < a + ϵ ∀ n ≥ k
or
an ∈ (a − ϵ, a + ϵ) ∀ n ∈ k
Thus, lim an = a, if and only if for every ϵ > 0, there exists k ∈ N such that
n→∞

an ∈ (a − ϵ, a + ϵ) ∀ n ≥ k.

3. Suppose < an > is a sequence and a ∈ R. Then to show that < an > does not
converge to a, we should be able to find an ϵ > 0 such that infinitely many terms of
the sequence are outside the interval (a − ϵ, a + ϵ) or there exist k ∈ N, such that
an ∈
/ (a − ϵ, a + ϵ) ∀ n ≥ k.

4. The different values of ϵ can result in different N , i.e. the number N may vary as ϵ
varies.
Example 6.12. Prove that every constant sequence is a convergent sequence.
Solution. Let < an >=< c > be a constant sequence, where c ∈ R. Then, for any given
ϵ > 0, there exists positive integer k = 1 ∈ N

|an − c| = |c − c| = 0 < ϵ ∀ n ≥ k = 1. (6.7)

Therefore, by the definition an converges to c. Thus, the given constant sequence is con-
vergent and converges to the constant term of the sequence.
1
Example 6.13. Show that the sequence < n
> is convergent and it converges to 0.
136 LESSON - 6. EXTREMUM AND CONVERGENCE OF SERIES

Solution. Since the given sequence is < an >=< n1 >=< 1, 21 , 13 , 41 , ... >. If the given
sequence is convergent then according to definition of convergence, for every ϵ > 0, there
exists positive integer N (in general depending on ϵ) such that |an − l| < ϵ, for n ≥ N.
Let ϵ be an arbitrary positive real number. Then

1 1 1
|an − 0| = − 0 = < ϵ, for n > (6.8)
n n ϵ

Let k be a positive integer such that k > 1ϵ . Then,

|an − 0| < ϵ, for n ≥ k,

1
Hence, the given sequence is convergent and converges to 0. That is lim = 0.
n→∞ n

6.4.3 Non-Convergent Sequences


A sequence which does not converge is called a divergent sequence.
Definition 6.11. If a sequence < an > is such that for every M > 0, there exists k ∈ N
such that
an > M, ∀ n ≥ k,
then we say that < an > diverges to +∞ and it is denoted as lim an = +∞.
n→∞

Definition 6.12. If a sequence < an > is such that for every M > 0, there exists k ∈ N
such that
an < −M, ∀ n ≥ k,
then we say that < an > diverges to −∞ and it is denoted as lim an = −∞.
n→∞

Definition 6.13. A sequence that diverges to either +∞ or −∞ is said to be a divergent


sequence.
Definition 6.14. A sequence that diverges to neither +∞ nor −∞ is said to be a non
divergent sequence.
Example 6.14. Following sequences are non convergent sequences:
1. The sequence < n2 >=< 1, 4, 9, 16, ... > diverges to +∞.

2. The sequence < −4n >=< −4, −8, −12, −16, ... > diverges to −∞.

3. The sequence < (−1)n .n >=< −1, 2, −3, 4, ... > neither diverges to +∞ nor −∞.
Definition 6.15. A bounded sequence is said to oscillate finitely, if it is neither convergent
nor divergent.
Definition 6.16. A sequence is said to oscillate infinitely, if
1. it is not bounded and
6.4. SEQUENCES 137

2. it neither converges nor diverges.


Example 6.15.
The sequence < (−1)n >=< −1, 1, −1, 1, ... > oscillates finitely.
The sequence < (−1)n .n >=< −1, 2, −3, 4, −5, 6, ... > oscillates infinitely.
The sequence < 1 + (−1)n >=< 0, 2, 0, 2... > oscillates finitely.
Definition 6.17 (Cauchy Sequence). A sequence < an > is said to be a Cauchy sequence
if for every ϵ > 0, there exists k ∈ N such that
|an − am | < ϵ ∀ n, m ≥ k.
Theorem 6.1. If < an >, < bn > be two convergent sequences such that lim an = a,
lim bn = b, then
(i) lim(an ± bn ) = lim an ± lim bn = a ± b.
(ii) lim(an bn ) = (lim an ) · (lim bn ) = ab.
 
an lim an a
(iii) lim = = . (b ̸= 0, bn ̸= 0 ∀n)
bn lim bn b
1
Example 6.16. Prove that lim = 0.
n→∞ n2

1
Solution. We know, lim = 0. Therefore by Theorem 6.1, we have
n→∞ n
 
1 1 1 1 1
lim 2 = lim · = lim · lim = 0.
n→∞ n n→∞ n n n→∞ n n→∞ n

Theorem 6.2 (Sandwich (Squeeze) Theorem). Let < an >, < bn > and < cn > are three
real-valued sequences such that an ≤ bn ≤ cn for all n and if furthermore
< an >→ l, < cn >→ l,
then
< bn >→ l.
Example 6.17. Using the above inequality prove that
sin x
lim = 1.
x→0 x

Solution. We know,
sin x
cos x < < 1.
x
Also, it is clear that lim cos x = 1 and limx→0 1 = 1. Hence, by squeeze theorem 6.2,
x→0

sin x
lim = 1.
x→0 x

Hence, we proved.
138 LESSON - 6. EXTREMUM AND CONVERGENCE OF SERIES

Theorem 6.3 (D’Alemberts Limit


 Theorem).
 Let < an > be a sequence of positive real
an+1
numbers such that L = lim exists. If L < 1, then < an > converges and
n→∞ an
lim(an ) = 0.
Example 6.18. Prove that
xn
lim= 0.
n→∞ n!

xn
Solution. Let < an >=< > be a sequence of real numbers. Then
n!
xn x(n+1)
an = , an+1 =
n! (n + 1)!
and
an+1 x(n+1) n! x
= · n = . (6.9)
an (n + 1)! x n+1
Also,
an+1 x
lim = lim = 0 < 1.
n→∞ an n→∞ n + 1

Hence, by the above theorem 6.3


xn
lim = 0.
n→∞ n!

Remark. Some important results based on the theorems of sequences are listed below
1. A sequence cannot converge to more than one limit.
2. Every convergent sequence is bounded but the converse is not true.
3. A sequence of real numbers converges if and only if it is a Cauchy Sequence.
a1 + a2 + ... + an
4. If lim an = l, then lim = l.
n→∞ n→∞ n
In-text Exercise 6.4. Solve the following questions:
1. By using the definition of convergence, show that
1 + 2 + 3... + n 1
lim 2
=
n→∞ n 2

2. Show that the sequence < (−3)n > does not converges.
2n2 + 3 2
3. Prove that lim 2
=
n→∞ 3n + 5n 3
1
4. If an = 2 − , find lim an .
2n n→∞

1 − cos x
5. Prove that lim = 0.
x→0 x
6.5. INFINITE SERIES 139

6.5 Infinite Series


An infinite series of real numbers is the sum of infinitely many terms of a sequence of real
numbers and it is written in the form

X
an = a1 + a2 + a3 + ...
n=1


X
where each an is a real number. an is called the nth term of the series an .
n=1

Example 6.19. Following are the examples of series of real numbers:



X 1 1 1 1
1. 2
= 2 + 2 + 2 + ...
n=1
n 1 2 3

X (−1)n+1 1 1 1
2. =1− + − + ...
n=1
n 2 3 4

Now the next question arise, how we can find the sum of infinite series. Because adding an
infinite number of terms in not a easy task. Therefore, instead of finding the sum of infinite
terms we will find the limit of its sequence of partial sums. A partial sum of an infinite
series is a finite sum of the form
k
X
an = a1 + a2 + a3 + ... + ak
n=1


X
Therefore, the sequence of partial sums of the series an is the sequence < Sn >, where
n=1

S1 = a1
S2 = a1 + a2
S3 = a1 + a2 + a3
..
.
Sn = a1 + a2 + a3 + ... + an .

Let take an example to see how partial sums can be used to evaluate the sum of an infinite
series.
Example 6.20. Suppose water is flowing from a tank into a pond such that 1000 liters
enters the pond in the first hour. During the second hour, an additional 500 liters of water
enters the pond. The third hour, 250 liters more water enters into the pond. Assume this
pattern continues such that each hour half as much water enters the pond as did the previous
hours. If this continues forever, what can we say about the amount of water in the pond?
140 LESSON - 6. EXTREMUM AND CONVERGENCE OF SERIES

Will the amount of water continue to get arbitrarily large, or is it possible that it approaches
some finite amount? To answer this question, we look at the amount of water in the pond
after k hours. Let Sk denote the amount of water in the pond (measured in thousands of
liters) after k hours, we see that

S1 = 1
1
S2 = 1 + 0.5 = 1 +
2
1 1
S3 = 1 + 0.5 + 0.25 = 1 + +
2 4
1 1 1
S4 = 1 + 0.5 + 0.25 + 0.125 = 1 + + +
2 4 8
1 1 1 1
S5 = 1 + 0.5 + 0.25 + 0.125 + 0.0625 = 1 + + + +
2 4 8 16
..
.

From the above we can observe that the obtained sums follows a pattern, therefore we can
find the amount of water in the pond after k hours as
k  n−1
1 1 1 1 1 X 1
Sk = 1 + + + + + ... + k−1 = .
2 4 8 16 2 n=1
2

Thus, we have the sequence of partial sums as < Sk >=< S1 , S2 , ..., Sk , ... >. Now, we
wants to find what happens as k → ∞. Symbolically, the amount of water in the pond as
k → ∞ is given by the infinite series
∞  n−1
X 1 1 1 1 1
=1+ + + + + ...
n=1
2 2 4 8 16

At the same time, as k → ∞ , the amount of water in the pond can be calculated by
evaluating lim Sk . Therefore, the behavior of the infinite series can be determined by
k→∞
looking at the behavior of the sequence of partial sums Sk . If the sequence of partial
sums < Sk > converges, we say that the infinite series converges, and its sum is given by
lim Sk . If the sequence < Sk > diverges, we say the infinite series diverges. Now we will
k→∞
determine the limit of the sequence of partial sums < Sk > . By, simplifying some of the
obtained partial sums, we see that

S1 = 1
1 3
S2 = 1 + =
2 2
1 1 7
S3 = 1 + + =
2 4 4
1 1 1 15
S4 = 1 + + + =
2 4 8 8
1 1 1 1 31
S5 = 1 + + + + = .
2 4 8 16 16
6.6. CONVERGENCE AND DIVERGENCE OF AN INFINITE SERIES 141

Plotting some of these values, it appears that the sequence < Sk > could be approaching 2.

Figure 6.6: The graph shows that the sequence of partial sums < Sk >→ 2 as n → ∞.

Since, this sequence of partial sums converges to 2, we say the infinite series converges to
2 and write
∞  n−1
X 1
= 2.
n=1
2
Thus, we conclude that the amount of water in the pond will get arbitrarily close to 2000
liters as the amount of time gets sufficiently large.
This series is an example of a geometric series. We will provide an analytic way later that
can be used to prove that lim Sk = 2.
k→∞

6.6 Convergence and Divergence of an Infinite Series



X ∞
X
Consider the infinite series an . Let < Sn > be the sequence of partial sums of an .
n=1 n=1
Then,

S1 = a1
S2 = a1 + a2
S3 = a1 + a2 + a3
.. ..
. .
Sk = a1 + a2 + a3 + ... + ak .
.. ..
. .

X
Definition 6.18. The infinite series an is said to converge to the sum S if and only if its
n=1
142 LESSON - 6. EXTREMUM AND CONVERGENCE OF SERIES

sequence of partial sums < Sn > converges to S. Then we write



X
an = S.
n=1

Also, if the sequence of partial sums diverges, we have the divergence of the series, and if
the sequence of partial sums oscillates, then the series also oscillates.

Geometric series
A geometric series

X
rn−1 = 1 + r + r2 + r3 + . . . (r > 0)
n=1

is the sum of an infinite number of terms that have a constant ratio between successive
terms. This geometric series converges if r < 1 and it diverges if r ≥ 1.

Example 6.21. Check the convergence and divergence of the following geometric series:
X 1 1 1 1
(a) n
= + 2 + 3 + . . . is convergent. (∵ r = 14 < 1)
4 4 4 4
X
(b) The series 1 = 1 + 1 + 1 + . . . is divergent. (∵ r = 1)
X
(c) 4n = 4 + 42 + 43 + . . .is divergent. (∵ r = 4 > 1)

Example 6.22. Show that the series

1 1 1
+ + + ...
1.2 2.3 3.4
is convergent.

Solution. Let < Sn > be the sequence of partial sums of the given series. Then

1 1 1 1
Sn = + + + ... +
1.2 2.3 3.4 n(n + 1)
       
1 1 1 1 1 1 1
= 1− + − + − + ... + −
2 2 3 3 4 n n+1
1
=1−
n+1
 
1 1
∴ lim < Sn >= lim 1 − = 1 − 0 = 1, as lim = 0.
n→∞ n→∞ n+1 n→∞ n + 1

Since, the sequence of partial sums < Sn > converges to 1, therefore the given series also
converges to 1.
6.6. CONVERGENCE AND DIVERGENCE OF AN INFINITE SERIES 143

p-series
The infinite series of real numbers

X 1
, p ∈ R,
n=1
np

is known as a p-series. It converges if p > 1 and diverges if p ≤ 1.

Example 6.23. Check the convergence or divergence of the following p-series:


P 1
1. n3
= 113 + 213 + 313 + · · · converges. (∵ p = 3 > 1)
P1
2. n
= 1 + 21 + 13 + · · · diverges. (∵ p = 1)
P 1
3. √ = 1 + √1 + √1 + · · · diverges. (∵ p = 12 < 1)
n 2 3

1 5
P
4. n5/2
is convergent. (∵ p = 2
> 1)

X
Theorem 6.4 (A necessary condition for convergence). If the series an converges,
n=1
then lim an = 0.
n→∞
P
Proof. Let < Sn > be the sequence of partial sums of the series an .

Then
Sn = a1 + a2 + . . . + an−1 + an ,
Sn−1 = a1 + a2 + . . . + an−1 .
Now

Sn − Sn−1 = an . (6.10)
P
Since, the series an converges, therefore < Sn > converges. Let lim Sn = l, then
n→∞

lim Sn−1 = l (6.11)


n→∞

From equation (6.10) and (6.11), we have

lim an = lim Sn − lim Sn−1 = l − l = 0.


n→∞ n→∞ n→∞

Hence, lim an = 0.
n→∞

Example 6.24. Show, by an example, that the converse of above theorem is not true i.e. if
lim an = 0 then series may or may not be convergent.
n→∞
144 LESSON - 6. EXTREMUM AND CONVERGENCE OF SERIES

X X1
Solution. The series an = is divergent (by p-series).
n
1
But lim an = lim = 0.
n→∞ n→∞ n
X X1
Thus, we see that lim an = 0, but the series an = is divergent.
n→∞ n
P
Remark. If lim an ̸= 0, then the series an cannot converge.
n→∞
P
Proof. Suppose an converges. Then by the above theorem, lim an = 0, which is con-
n→∞
trary to the given condition. Hence the remark.

Example 6.25. Show that series


X n 1 2 3
= + + + ···
n+1 2 3 4
is not convergent
n 1
Solution. We have an = =
n+1 1 + n1
1
Therefore lim an = lim = 1 ̸= 0.
n→∞ n→∞ 1 + 1
n

Hence, by the above remark, the given series is not convergent.

Theorem 6.5 X(Cauchy’s Principle of Convergence). A necessary and sufficient condition


for a series an to converge is that to each ε > 0, there exists a positive integer m, such
that
|am+1 + am+2 + . . . + an | < ε, for all n ≥ m.

Example 6.26. Show that the series


∞  n 1
X 1
n=1
n

is not convergent.
  n1
1 1 1
Solution. Let an = , so that log an = log .
n n n
or
log 1 − log n − log n
log an = =
n n

log n ∞
lim log an = − lim , which is form
n→∞ n→∞ n ∞
1
n
= − lim (by L’Hôpital’s Rule )
n→∞ 1
6.7. SUMMARY 145

1
= − lim = 0.
n→∞ n
Now,
lim log an = 0 ⇒ log ( lim an ) = 0
n→∞ n→∞
0
⇒ lim an = e = 1 ̸= 0.
n→∞
P
Hence an is not convergent.
In-text Exercise 6.5. Solve the following questions:
∞  
X 1
1. Test for convergence of the series cos .
n=1
n

X
2. Show that the series (−1)n−1 oscillates.
n=1

3. Show that the series


r r r
1 2 n
+ + ... + + ...
4 6 2(n + 1)
is not convergent.

X
4. Test for the convergence of the series (−1)n · n.
n=1

6.7 Summary
Following points have been discussed in this lesson:
1. Maxima and minima are the peaks and valleys in the curve of a function.
2. There can be only one absolute maxima of a function and one absolute minimum of
a function over the entire domain, whereas there may be several local maxima and
local minima.
3. The maxima and minima are collectively known as the “Extrema”.
4. If there is a function that is continuous, it must have maxima and minima or local
extrema. Also, if the given function is monotonic, the maximum and minimum values
lie at the endpoints of the domain of the definition of that function.
5. The concept of Maxima and Minima is used to solve some practical problems.
6. A sequence < an > in R is said to converge to a real number a if for every ϵ > 0,
there exists positive integer N (in general depending on ϵ) such that
|an − a| < ϵ, ∀ n ≥ N,
where the number a is called the limit of the sequence.
146 LESSON - 6. EXTREMUM AND CONVERGENCE OF SERIES


X
7. Geometric series rn−1 converges for |r| < 1 and diverges for |r| ≥ 1.
n=1


X 1
8. The series converges for p > 1 and diverges for p ≤ 1.
n=1
np
P
9. If the series an converges, then lim an = 0.
n→∞

6.8 Self-Assessment Exercises


1. Find the local maxima and local minima of the function f (x) = sin4 x + cos4 x, 0 <
x < π2 using the first derivative test.

2. Find the point of local maxima and local minima of the function f (x) = x 1 − x, x >
0 using the first derivative test. Also, find the local maximum and local minimum val-
ues.
r
p q −1 p
3. Show that sin θ cos θ attains a maximum, when θ = tan .
q
log x
4. Show that has a maximum value at x = e.
x
5. Find all the points of local maxima and local minima and the corresponding maxi-
mum and minimum values of the function
3 45
f (x) = − x4 − 8x3 − x2 + 105.
4 2

6. Show that minimum value of the function f (x) = x50 − x20 in the interval [0, 1] is
 2
3 2 3
− .
5 5
7. Find the point of local maxima and local minima, if any for the given function
1 π
f (x) = sin x + cos 2x, where 0 ≤ x ≤ .
2 2

8. Find the maximum and the minimum value of the function

f (x) = 3x4 –8x3 + 12x2 –48x + 25

on the closed interval [0, 3].

9. Find the global maxima and global minima of the function f (x) = sin x+sin x cos x,
in the interval [0, π].

10. Show that of all the rectangles of given area, the square has the smallest perimeter.
6.9. SOLUTIONS TO IN-TEXT EXERCISES 147

11. Show that of all the rectangles inscribed in a given circle, the square has the maximum
area.
 ( n1 )   n1
1 1
12. Show that lim = 1 and lim = 1.
n→∞ n n→∞ n2
13. Show that < 1 + (−1)n > is not convergent.

14. By using the definition of convergence, show that


1 + 3 + 5 + ... + 2n − 1
lim = 1.
n→∞ n2
∞ r
X n
15. Show that the series diverges.
n=1
n + 1

∞   1
X 1 n
16. Test for the convergence of the series 2
.
n=1
n

6.9 Solutions to In-text Exercises


Exercise 6.1

1. The point x = 2 is neither a point of local maxima nor minima. It is a point of


inflexion.
π
2. f (x) attains a local maximum at x = .
4
3. None in the interval (0, π)

4. x = 0 is a point of local minima and minimum value is f (x) = −4. x = −2 is a


point of local maxima and maximum value is f (x) = 0. Also, there is no point of
inflexion.

Exercise 6.2

1. x = 1 is a point of local maxima and the maximum value is −3,


x = 6 is a point of local minima and the local minimum value is −128.

2. Since f ′ (x) = 0 does not have any real root, therefore f (x) does not have a maximum
or minimum.

3. (i) Local maximum at x = −2 and the local maximum value is 0,


Local minima at x = 0 and local minimum value is −4.
3 5
(ii) Local maximum at x = and the local maximum value is .
4 4
π √
(iii) Local maximum at x = and the local maximum value is 2.
4
148 LESSON - 6. EXTREMUM AND CONVERGENCE OF SERIES

π √ π
(iv) Local maximum at x = and the local maximum value is 3 + ,
6 6
5π √ 5π
Local minima at x = and the local minimum value is − 3 + .
6 6
2
4. Profit is maximum when x = and the maximum value of profit is 49.
3
1
5. a = 2, b = − .
2
Exercise 6.3

1. The maximum value of f (x) is 0 which is attained at x = π and x = 2π, and the

minimum value is −1 which is attained at x = .
2
2. Absolute maximum value = 19 at x = −3, Absolute minimum value= 3 at x = 1.

3. Absolute maximum value = 56 at x = 5, Absolute minimum = 24 at x = 1.

4. The maximum value of f (x) is 2π and the minimum value is 0.

Exercise 6.4

1 1 1
1. an − = < ϵ for 2n > .
2 2n ϵ


1
2 + 3 lim
2n2 + 3 n→∞ n2 2
3. lim 2
=   = .
n→∞ 3n + 5n 1 3
3 + 5 lim
n→∞ n

4. lim an = 2.
n→∞

Exercise 6.5

1. Since lim an ̸= 0. Hence the given series is not convergent.


n→∞

2. Since the sequence of partial sums oscillates, hence the series oscillates.

1
3. Since lim an = √ . Hence the given series is not convergent.
n→∞ 2

4. Sine the sequence of partial sums is unbounded, hence not convergent. Thus the
series is not convergent.
6.10. SUGGESTED READINGS 149

6.10 Suggested Readings


1. Narayan, Shanti (Revised by Mittal, P. K.). Differential Calculus. S. Chand, Delhi,
2019.

2. Prasad, Gorakh (2016). Differential Calculus (19th ed.) Pothishala Pvt. Ltd. Alla-
habad.

3. Thomas Jr., George B., Weir, Maurice D.,Hass, Joel (2014). Thomas Calculus (13th
ed.). Pearson Education, Delhi. Indian Reprint 2017.
Lesson - 7 Indeterminate Forms
MsSetu Rani
Satyawati College (E)
University of Delhi

Structure
7.1 Learning Objectives 150
7.2 Introduction 151
7.3 Indeterminate Forms of Limits 151
0
7.4 Indeterminate Form 0 152

7.5 Indeterminate Form ∞ 155
7.6 Indeterminate Form ∞ − ∞ 157
7.7 Indeterminate Form 0 × ∞ 158
7.8 Indeterminate Forms 1∞ , ∞0 and 00 160
7.9 Summary 164
7.10 Self-Assessment Exercises 164
7.11 Suggested Readings 167

7.1 Learning Objectives


The learning objectives of this lesson are to:

• understand the various types of indeterminate forms and conditions to their forma-
tion.

• learn the L’Hôpital’s Rule.

• use L’Hôpital’s Rule to evaluate indeterminate forms arising from limits of products,
differences, quotient and exponentials.

• apply the methods of indeterminate forms for evaluating various tedious limits.

150
7.2. INTRODUCTION 151

7.2 Introduction
In Mathematics, the solution to a problem becomes indeterminate when the information
available is insufficient to solve the problem. The problem of evaluation of the limit of
a function becomes indeterminate if the method discussed in the previous lessons do not
work to completely evaluate the limit. in order to evaluate the limit in indeterminate form,
we usually use L’Hôpital’s Rule. In this lesson, we shall discuss the indeterminate forms
of limits with examples. Also, we will learn how to apply L’Hôpital’s Rule to evaluate
the limits of the several indeterminate forms. In most of the cases, the indeterminate form
occurs while taking the ratio of two functions, such that both of the functions approaches
0
zero in the limit. Such cases are called “indeterminate form ”. Similarly, the indeterminate
0
form can be obtained in addition, subtraction, multiplication, exponential operations also.

7.3 Indeterminate Forms of Limits


Some forms of limits are called indeterminate if the limiting behavior of individual parts
of the given expression is not able to determine the limit. Indeterminate forms occur in
various types. To understand the indeterminate form, it is important to learn about its
types. In Calculus, following types of indeterminate forms of limits occur frequently:

0 ∞
form, form, ∞ − ∞ form, 0 × ∞ form, and forms of the type 1∞ , ∞0 , and 00 .
0 ∞

For example, on the basis of the information available from the previous lessons, we can
f (x)
not decide on lim when lim g(x) = 0. In order to evaluate this limit (if it exists) of
x→a g(x) x→a
f (x)
, ( when lim g(x) = 0), we require the additional information that lim f (x) should
g(x) x→a x→a
f (x)
also be 0. On the contrary, let us assume that lim f (x) ̸= 0 and lim = l,(finite). Then
x→a x→a g(x)

f (x)
f (x) = · g(x), g(x) ̸= 0.
g(x)

f (x)
=⇒ lim f (x) = lim · lim g(x)
x→a x→a g(x) x→a

= l · 0 = 0,
f (x)
which is a contradiction to our supposition. Hence, for lim to exist provided lim g(x) =
x→a g(x) x→a
0, we must have lim f (x) = 0. Thus this form of limit is an indeterminate form of limit.
x→a
We now discuss these indeterminate forms of limits, separately.
152 LESSON - 7. INDETERMINATE FORMS

0
7.4 Indeterminate Form 0
f (x) 0
The limit of the type lim is said to be an indeterminate form of the type , if
x→a g(x) 0
lim f (x) = 0 = lim g(x).
x→a x→a

It is evaluated by using the L’Hôpital’s Rule as following:

(i) lim f (x) = lim g(x) = 0


x→a x→a

(ii) Derivatives f ′ (x) and g ′ (x) exist and g ′ (x) ̸= 0 ∀x ∈ [a − δ, a + δ] δ > 0.


f ′ (x)
(iii) lim exists.
x→a g ′ (x)

Then,
f (x) f ′ (x)
lim = lim ′ .
x→a g(x) x→a g (x)

Remark. Some basic remarks on L’Hôpital’s Rule


f ′ (x) 0
1. If lim does not exist and again it is an indeterminate form of the type , then
x→a g ′ (x) 0
f ′ (x) f ′′ (x)
the Rule is repeated again i.e. lim ′ = lim ′′ , and hence
x→a g (x) x→a g (x)

f (x) f ′ (x) f ′′ (x)


lim = lim ′ = lim ′′ .
x→a g(x) x→a g (x) x→a g (x)

f (n) (x)
2. This Rule can be extended to any order, That is, if lim is of the indeterminate
x→a g (n) (x)
0
form , then
0
f (n) (x) f (n+1) (x)
lim = lim ,
x→a g (n) (x) x→a g (n+1) (x)

and hence
f (x) f ′ (x) f ′′ (x) f (n+1) (x)
lim = lim ′ = lim ′′ = . . . = lim (n+1) .
x→a g(x) x→a g (x) x→a g (x) x→a g (x)

Example 7.1. Evaluate


sin x
lim .
x→0 x

Solution. Let f (x) = sin x and g(x) = x. Then

lim f (x) = lim sin x = 0


x→0 x→0
0
7.4. INDETERMINATE FORM 0
153

lim g(x) = lim x = 0.


x→0 x→0

f (x) sin x 0
Therefore, lim = lim is an indeterminate form of the type . Therefore, by
g(x)
x→0 x→0 x 0
using the L’Hôpital’s Rule, we have
f (x) f ′ (x)
lim = lim ′
x→0 g(x) x→0 g (x)

sin x cos x
i.e. lim = lim = 1.
x→0 x x→0 1
Example 7.2. Evaluate
log(1 − x2 )
lim
x→0 log(cos x)

Solution. We have f (x) = log(1 − x2 ) and g(x) = log(cos x),

lim f (x) = lim log(1 − x2 ) = log[lim (1 − x2 )] = log 1 = 0.


x→0 x→0 x→0

and
lim g(x) = lim log(cos x) = log 1 = 0.
x→0 x→0

0
Hence, the given limit is of the indeterminate form . Therefore, using the L’Hôpital’s Rule
0
d
log(1 − x2 ) dx
log(1 − x2 )
lim = lim d
x→0 log(cos x) x→0 log(cos x)
dx
( −2x2 )
= lim −1−x
x→0 ( sin x )
cos x
−2x
( 1−x 2)
= lim
(− tan x)
x→0
 
2x 0
= lim form .
x→0 (1 − x2 ) tan x 0
0
Since, the above expression is again in the form of , hence we again apply L’ Hospital
0
Rule
2x 2 2
∴ lim 2
= lim 2 2
= = 2.
x→0 (1 − x )(tan x) x→0 (1 − x )(sec x) + tan x · (−2x) 1·1+0
Thus, finally we have
log(1 − x2 )
lim = 2.
x→0 log(cos x)

Example 7.3. Find the value of


ex − 1
lim
x→0 x2 + x
154 LESSON - 7. INDETERMINATE FORMS

Solution. Let f (x) = ex − 1 and g(x) = x2 + x.

=⇒ lim f (x) = lim (ex − 1) = 1 − 1 = 0


x→0 x→0
and
lim g(x) = lim (x2 + x) = 0
x→0 x→0
0
Hence, the given limit is of the indeterminate form . Therefore, by using the L’ Hospital
0
Rule , we get
d
ex − 1 dx
(ex − 1)
lim = lim d
x→0 x2 + x x→0 (x2 + x)
dx
x
e
= lim
x→0 2x + 1
e0
=
2·0+1
= 1.
Example 7.4. Find the value of a and b for which
x(1 + a cos x) − b sin x
lim
x→0 x3
exists and is equal to 1.
Solution. Since
x(1 + a cos x) − b sin x
lim
x→0 x3
0
is of the indeterminate form , therefore by using the L’ Hospital Rule , we have
0
d
x(1 + a cos x) − b sin x dx
+ a cos x) − b sin x)
(x(1
lim = lim d
x→0 x3 x→0
dx
(x3 )
1 + a cos x − a sin x − b cos x
= lim
x→0 3x2
1+a−b
= . (7.1)
0
Since, it is given that
x(1 + a cos x) − b sin x
lim
x→0 x3
is finite and is 1. Therefore, to get the finite limit, we should have
1 + a − b = 0. (7.2)
0
Then the limit in 7.1 is again in form
0
 
1 + a cos x − a sin x − b cos x 0
i.e. lim is form .
x→0 3x2 0

7.5. INDETERMINATE FORM ∞
155
 
−a sin x − a sin x − ax cos x + b sin x 0
= lim form again, by L’Hôpital’s Rule
x→0 6x 0
−2a cos x + ax sin x − a cos x + b cos x
= lim
x→0 6
−2a − a + b
=
6
−3a + b
= . by the L’Hospital’s Rule.
6
Since, the given limit is 1

−3a + b
∴ =1
6
−3a + b = 6. (7.3)

On Solving 7.2 and 7.3 for a and b, we get

5 3
a=− and b = − .
2 2


7.5 Indeterminate Form ∞
f (x) ∞
A limit of the form lim is said to be an indeterminate form of the type , if lim f (x) = ∞ = lim g(x).
x→a g(x) ∞ x→a x→a

It is evaluated by using the L’Hôpital’s Rule as following:

If

(i) lim f (x) = lim g(x) = ∞.


x→a x→a

(ii) Derivatives f ′ (x) and g ′ (x) exist and g ′ (x) ̸= 0 for x ∈ (a − δ, a + δ) δ > 0.

f ′ (x)
(iii) lim exists.
x→a g ′ (x)

Then,
f (x) f ′ (x)
lim = lim ′ .
x→a g(x) x→a g (x)

Remark. On applying the L’Hôpital’s Rule once, if the resulting limit of quotient is again
is an indeterminate form, then we apply the L’Hôpital’s Rule repeatedly until we get a finite
limit of the quotient.

Example 7.5. Compute the following limit

2x2 + 3
lim .
x→∞ 5x2 + x
156 LESSON - 7. INDETERMINATE FORMS

Solution. Since
2x2 + 3
lim
x→∞ 5x2 + x

is an indeterminate form of type . Therefore, using L’Hôpital’s Rule, we get

d
2x2 + 3 dx
(2x2 + 3) 4x ∞ 
lim = lim d
= lim form
x→∞ 5x2 + x x→∞ (5x2 + x) x→∞ 10x + 1 ∞
dx
4 2
= = .
10 5
Example 7.6. Evaluate
ex
lim .
x→∞ x2

Solution. Here,
lim ex = ∞ = lim x2 .
x→∞ x→∞

ex ∞
Therefore, lim 2 is of indeterminate form. Hence, by applying the L’Hôpital’s Rule,
x→∞ x ∞
we get
ex ex  ∞ 
lim 2 = lim form ,
x→∞ x x→∞ 2x ∞
x
e
= lim = ∞.
x→∞ 2

Example 7.7. Evaluate


log(x − a)
lim .
x→a log(ex − ea )

Solution. Since
log(x − a) ∞
lim x a
is of indeterminate form,
x→a log(e − e ) ∞
therefore, using L’Hôpital’s Rule, we get
1
log(x − a) (x−a)
lim = lim ex
x→a log(ex − ea ) x→a
ex −ea
e − ea
x
 
0
= lim x is of form
x→a e (x − a) 0
ex
= lim x
x→a e (x − a) + ex
ea
= a
e
=1

In-text Exercise 7.1. Solve the following questions:


7.6. INDETERMINATE FORM ∞ − ∞ 157

1. If
sin 2x + a sin x
lim
x→0 x3
is finite, find the value of a and the limit.

2. Evaluate
tan x − x
lim .
x→0 x2 tan x

3. Prove that
tan x
lim = 1.
x→0 x
4. Evaluate
xex − log(1 + x)
lim .
x→0 x2
5. Evaluate
log(x − 1) + tan πx
2
lim
x→1 cot πx
6. Find the value of
ex sin x − x − x2
lim
x→0 x3

7.6 Indeterminate Form ∞ − ∞


If lim f (x) = lim g(x) = ∞, then lim (f (x) − g(x)) is said to be of the indeterminate form
x→a x→a x→a
∞ − ∞.
0
This type of limit is evaluated by rearranging the terms and converting it into the inde-
0
terminate form. We write
 
f (x) − g(x)
lim (f (x) − g(x)) = lim f (x)g(x)
x→a x→a f (x)g(x)
 
1 1
g(x)
− f (x)
= lim 1
x→a
f (x)g(x)

0 1
which is of the form as lim f (x) = ∞ =⇒ lim = 0 and lim g(x) = ∞ =⇒
0 x→a x→a f (x) x→a
1 0
lim = 0. The resulting form is then evaluated by using L’Hôpital’s Rule.
x→a g(x) 0
Example 7.8. Find the value of
 
1 1
lim −
x→0 x sin x
158 LESSON - 7. INDETERMINATE FORMS

Solution. Since  
1 1
lim − is of ∞ − ∞ form,
x→0 x sin x
therefore, it can be rearranged as
     
1 1 sin x − x 0
lim − = lim form
x→0 x sin x x→0 x sin x 0
   
cos x − 1 0
= lim form
x→0 sin x + x cos x 0
 
− sin x
= lim
x→0 cos x − x sin x + cos x
 
− sin x
= lim
x→0 −x sin x + 2 cos x
=0

Alternately,
   
1 1 sin x − x
lim − = lim
x→0 x sin x x→0 x sin x
 
sin x − x  x 
= lim · lim
x→0 x2 x→0 sin x
 
sin x − x sin x
= lim ∵ lim =1
x→0 x2 x→0 x
 
cos x − 1 0
= lim form
x→0 2x 0
− sin x
= lim
x→0 2
= 0.

7.7 Indeterminate Form 0 × ∞


If lim f (x) = 0 and lim g(x) = ±∞, then lim (f (x) · g(x)) is said to be a limit of the
x→a x→a x→a
indeterminate form 0 × ∞.
0 ∞
To evaluate this type of limit it is converted to or form first and then evaluated.
0 ∞
 
f (x) 0
We write lim f (x) · g(x) = lim 1 form
x→a x→a
g(x)
0
g(x)  ∞ 
or = lim 1 form
x→a
f (x)

which can be solved by using L’Hôpital’s Rule.


7.7. INDETERMINATE FORM 0 × ∞ 159

Example 7.9. Evaluate 



2 1
lim x · sin
x→∞ x2
   
2 1 2 1
Solution. Since, lim x = ∞ and lim sin 2
= 0, therefore lim x · sin is an
x→∞ x→∞ x x→∞ x2
indeterminate form of type 0 × ∞. We write it as
sin x12
  
2 1 0
lim x · sin 2
= lim 1
 which is of the form .
x→∞ x x→∞
x2
0
cos x12 · −2
 
x3
= lim −2 , by using L’Hospital’s Rule
x→∞
x3
 
1
= lim cos
x→∞ x2
= 1.
Example 7.10. Find the limit
 
1
lim (x + 6) ·
x→∞ x2 + 3
Solution. Since,  
1
lim (x + 6) ·
x→∞ x2 + 3
is of ∞ · 0 form, therefore it can be written as
 
1 (x + 6)  ∞ 
lim (x + 6) · = lim 2 form
x→∞ x2 + 3 x→∞ (x + 3) ∞
1
= lim , by using L’Hospital’s Rule
x→∞ 2x
= 0.
In-text Exercise 7.2. Solve the following questions:
1. Prove that limx→0+ xm (log x)n ; m.n ∈ N is zero.
2. Evaluate the value of  
1 1
lim − .
x→0 x 2 sin2 x
3. Solve  
1 csc x
lim 2
− csc x = cosecx.
x→0 x x
4. Evaluate  
1 1
lim − .
x→4 log(x − 3) x−4
5. Evaluate  
1
lim x tan .
x→∞ x
160 LESSON - 7. INDETERMINATE FORMS

7.8 Indeterminate Forms 1∞, ∞0 and 00


Consider lim (f (x))g(x) . It is a indeterminate form of the type
x→a

(i) 1∞ , if lim f (x) = 1 and lim g(x) = ∞.


x→a x→a

(ii) ∞0 , if lim f (x) = ∞ and lim g(x) = 0.


x→a x→a

(iii) 00 , if lim f (x) = 0 and lim g(x) = 0.


x→a x→a

To evaluate these types of limits, we write

y = (f (x))g(x) .

Taking log on both sides, we have

log y = g(x) log(f (x))


=⇒ lim log y = lim [g(x) log(f (x))]. (7.4)
x→a x→a

∞ 0
Now, the limit on the right side of 7.4 can be reduced either in the form or form,
∞ 0
which can be evaluated by using the L’Hôpital’s Rule. Suppose,

lim (g(x) log(f (x)) = l


x→a

then equation 7.4 becomes,

lim log y = l
x→a
or log lim y = l
x→a
lim y = el
a→a
g(x)
Thus lim (f (x)) = el where l = lim g(x) log(f (x)).
x→a x→a

Example 7.11. Find the value of


2
limπ (sin x)(tan x)
x→ 2

Solution. Since,
π 
limπ f (x) = sin =1
x→ 2 2
  π 2
limπ g(x) = tan =∞
x→ 2 2
7.8. INDETERMINATE FORMS 1∞ , ∞0 AND 00 161

2 2
Therefore, limπ (sin x)(tan x) is of the form (1)∞ . Let y = (sin x)(tan x) , then taking log on
x→ 2
both side, we get

log y = (tan x)2 log(sin x)


=⇒ limπ log y = limπ (tan x)2 log(sin x) (∞ × 0 form)
x→ 2 x→ 2
 
log(sin x) 0
= limπ form
x→ 2 (cot x)2 0
cos x
sin x
= limπ
x→ 2 −2 cot x · (csc x)2
cot x
= limπ
x→ 2 −2 cot x · (csc x)2

−1 1
= limπ 2
=−
x→ 2 2(csc x) 2
1
limπ log y = log( limπ y) = − .
x→ 2 x→ 2 2
Hence,
−1 1
limπ y = e 2 =√ .
x→ 2 e
2 1
i.e. limπ (sin x)(tan x) =√ .
x→ 2 e
Example 7.12. Evaluate the limit
lim xx .
x→0

Solution. Since the given limit is of the indeterminate form 00 . Let

y = xx
=⇒ log y = x log x
=⇒ lim log y = lim x log x (0 × ∞ form)
x→0 x→0
log x  ∞ 
= lim 1 form
x→0
x

1
x
= lim , by using the L’Hospital’s Rule
x→0 −1
x2
− lim x = 0.
x→0

Thus

log lim y = 0
x→0
=⇒ lim y = e0
x→0
=⇒ lim xx = lim y = e0 = 1.
x→0 x→0
162 LESSON - 7. INDETERMINATE FORMS

Example 7.13. Evaluate the following limit


 1−cos x
1
lim
x→0 x
Solution. Since, the given limit is of the form (∞)0 , therefore let
 1−cos x
1
y=
x
 
1
=⇒ log y = (1 − cos x) log
x
 
1
∴ lim log y = lim (1 − cos x) log
x→0 x→0 x
 x 2
= lim 2 sin (log 1 − log x)
x→0 2
 x 2
= lim 2 sin (− log x)
x→0 2
sin x2 2 x2
 
= lim 2 x · · (− log x)
x→0
2
4
 !2
(− log x · x2 ) sin x2
= lim · lim x
x→0 2 x→0
2
 x  
− log x sin 2 ∞ 
= lim 2 ∵ lim x = 1 form
x→0
x2
x→0
2

−1
x
= lim , by using the L’Hospital’s Rule
x→0 −4
3
x 2 
x
= lim =0
x→0 4
lim log y = log lim y = 0
x→0 x→0
0
=⇒ lim y = e = 1.
x→0

Thus
 1−cos x
1
lim = lim y = 1.
x→0 x x→0

Example 7.14. Evaluate


 1
sin x ( x2 )

lim
x→0 x
Solution. We have

sin x
lim f (x) = lim =1
x→0 x→0 x
 
1
lim g(x) = lim = ∞.
x→0 x→0 x2
7.8. INDETERMINATE FORMS 1∞ , ∞0 AND 00 163

 1
sin x ( x2 )

Therefore, lim is of the indeterminate form (1)∞ . Let
x→0 x
 1
sin x ( x2 )

y =
x
   
1 sin x
=⇒ log y = log
x2 x
   
1 sin x
lim log y = lim log (∞ × 0 form)
x→0 x→0 x2 x
log sinx x
  
0
lim log y = lim form
x→0 x→0 x2 0
x cos x−sin x
x2
sin x
x
= lim , by using the L’Hospital’s Rule.
2x
x→0
x cos x − sin x
= lim
x→0 2x2 sin x
x cos x − sin x  x 
= lim · lim
x→0
 2x3 x→0 sin x
   
−x sin x + cos x − cos x sin x
= lim ∵ lim =1 .
x→0 6x2 x→0 x
− sin x
= lim
x→0 6x  
−1 sin x
= lim · lim
x→0 6 x→0 x
−1
=
6
−1
lim log y = log lim y =
x→0 x→0 6
− 16
=⇒ lim y = e .
x→0

Hence,
 1
sin x ( x2 )

−1
lim log = lim y = e 6 .
x→0 x x→0

In-text Exercise 7.3. Evaluate the following limits:

1. limπ (tan x)tan 2x .


x→ 4

1
2. lim (csc x) log x .
x→0
 x
k
3. lim 1+ .
x→∞ x
164 LESSON - 7. INDETERMINATE FORMS

  x1
sin x
4. lim .
x→0 x
tan( πx )
2x2
 2a
5. lim 3 − 2 .
x→a a

7.9 Summary
We have discussed the following points in this lesson:
• Indeterminate forms of limits are of the following forms:
0 ∞
, , 0 × ∞, ∞ − ∞, 00 , 1∞ , and ∞0
0 ∞
0 ∞
• Indeterminate forms and can be easily evaluated by using L’ Hospital Rule
0 ∞
which state that
f (x) f ′ (x)
lim = lim ′ .
x→a g(x) x→a g (x)

• The limiting value of an indeterminate form is called the true value of the limit.
• Indeterminate forms 0 × ∞, ∞ − ∞, 00 , 1∞ , and ∞0 can be easily evaluated by
0 ∞
using or form.
0 ∞

7.10 Self-Assessment Exercises


1. Evaluate the determinate form:
x cos x − log(1 + x)
lim .
x→0 x2
2. Evaluate the following limits
(a)
π  x1
−1
lim − tan x .
x→∞ 2
(b)  
1 log(1 + x)
lim − .
x→0 x x2
(c)
1 − (sec x)2
lim .
x→0 3x2
(d)
ex sin x − x − x2
lim .
x→0 x2 + x log(1 − x)
7.10. SELF-ASSESSMENT EXERCISES 165

(e)
  x1
sinh x
lim .
x→0 x
3. Show that
ex + log 1−x

e 1
lim =− .
x→0 tan x − x 2
4. Find the value of p, q and r, if
rey − q cos y + pe−y
lim = 3.
y→0 y tan y

5. Find the values of a, b, c, so that


aex − b cos x + ce−x
lim = 2.
x→0 x sin x

6. Show that 1 ex
(1 + x) x − e + 2 11e
lim = .
x→0+ x2 24
7. Find the value of
2x4 + 4x3 − 100
lim .
x→∞ 4x4 + 9x2 + 2x + 100

8. Evaluate  
log sin x
lim logx sin x Hint. logx sin x = .
x→0+ log x

Solutions to In-text Exercises


Exercise 7.1

1. Considering a ∈ R to be finite, the given limit


sin 2x + a sin x
lim
x→0 x3
0
is of the form , therefore we evaluate it by applying L’Hôpital’s Rule to obtain
0
a = −2 and the value of limit as −1.
1
2. The value of the limit is .
3
3
4. The value of the limit is .
2
1
5. The value of the limit is .
3
166 LESSON - 7. INDETERMINATE FORMS

1
6. The value of the limit is .
3

Exercise 7.2

1. lim xm (log x)n ; m.n ∈ N is of the form (0 × ∞). Therefore, we can convert it
x→0+
0 ∞
either in the form of or and then use L’Hôpital’s Rule to evaluate the limit. The
0 ∞
value of the limit is 0.
 
1 1
2. lim 2 − is of the form ∞ − ∞. Therefore, we can convert it either in the
x→0 x sin2 x
0 ∞
form of or and then use L’Hôpital’s Rule to evaluate the limit. The value of the
0 ∞
1
limit is − .
3
 
1 csc x
3. lim 2 − is of the form ∞ − ∞. Therefore, we can convert it either in the
x→0 x x
0 ∞
form of or and then use L’Hôpital’s Rule to evaluate the limit. The value of the
0 ∞
1
limit is − .
6
 
1 1
4. lim − is of the form ∞ − ∞. Therefore, we can convert it either
x→4 log(x − 3) x−4
0 ∞
in the form of or and then use L’Hôpital’s Rule to evaluate the limit. The value
0 ∞
1
of the limit is .
2
5. The value of limit is 1.

Exercise 7.3

1. limπ (tan x)tan 2x is of the form (1)∞ which can be converted either in 00 form or

x→ 4



form. Then use L’hospital’s Rule to evaluate the limit. The value of limit is −1.
1
2. lim (csc x) log x is of the form (∞)0 . Therefore, by taking log on both side, we get
x→0

1
lim log y = lim log (csc x) (0 × ∞ form)
x→0 x→0log x
 
log csc x  ∞ 
= lim form
x→0 log x ∞
= −1

1 1
Thus, lim (csc x) log x = e−1 = .
x→0 e
7.11. SUGGESTED READINGS 167
 x
k
3. lim 1+ is of the form (1)∞ . Therefore, by taking log on both side, we get
x→∞ x
 
k
lim log y = lim x log 1 + (∞ × 0 form)
x→∞ x→∞ x
   
k 1 0
= lim log 1 + / form
x→∞ x x 0
=k
 x
k
Thus, lim 1+ = ek .
x→∞ x
  x1
sin x
4. lim is of the form (1)∞ and the value of limit is e.
x→0 x
tan( πx )
2x2
 2a
0
5. lim 3 − 2 is of the form (1)∞ which can be converted either in form
x→a a 0

or form. Then use L’hospital’s Rule to evaluate the limit. The value of limit is π 2 .

7.11 Suggested Readings


1. Narayan, Shanti (Revised by Mittal, P. K.). Differential Calculus. S. Chand, Delhi,
2019.

2. Prasad, Gorakh (2016). Differential Calculus (19th ed.) Pothishala Pvt. Ltd. Alla-
habad.

3. Thomas Jr., George B., Weir, Maurice D.,Hass, Joel (2014). Thomas Calculus (13th
ed.). Pearson Education, Delhi. Indian Reprint 2017.
Lesson - 8

Taylor’s Series and Maclaurin’s Series


Ms. Setu Rani
Satyawati College (E)
University of Delhi

Structure
8.1 Learning Objectives 168
8.2 Introduction 169
8.3 Cauchy’s Mean Value Theorem 169
8.4 Taylor’s Theorem 171
8.4.1 Taylor’s Infinite Series 175
8.4.2 Maclaurin’s Theorem 177
8.4.3 Maclaurin’s infinite series 178
8.5 Maclaurin’s Series Expansion of Some Standard Functions 179
8.5.1 Maclaurin’s Series Expansion of ex 179
8.5.2 Maclaurin’s Series Expansion of sin x 180
8.5.3 Maclaurin’s Series Expansion of cos x 181
8.5.4 Maclaurin’s Series Expansion of loge (1 + x) ≡ ln(1 + x) 181
8.5.5 Maclaurin’s Series Expansion of (1 + x)m , |x| < 1 183
8.6 Summary 187
8.7 Self-Assessment Exercises 188
8.8 Solutions to In-text Exercises 188
8.9 Suggested Readings 189

8.1 Learning Objectives


The learning objectives of this lesson are to:

• study the Cauchy’s Mean Value Theorem and its applications.

• explain the meaning and significance of Taylor’s theorem.

168
8.2. INTRODUCTION 169

• learn to obtain the Taylor series expansion of a function.

• obtain the Maclaurin’s series expansions of some standard functions.

8.2 Introduction
We have already learnt the applications of Rolle’s theorem and Lagrange’s Mean Value
Theorem in the previous lesson 5. Taylor’s theorem, which we will study in this lesson,
can be regarded as a general form of Lagrange’s Mean Value Theorem when the function
is differentiable successively n times, n > 1. In this lesson, we examine how functions
may be expressed in terms of power series. This is an extremely useful way of expressing
a function since we can replace complicated functions in terms of simple polynomials.

8.3 Cauchy’s Mean Value Theorem


Cauchy’s Mean Value Theorem is a generalized form of the Lagrange’s Mean Value The-
orem. This theorem is also called the “Second Mean Value Theorem”. It establishes the
relationship between the derivative of the two functions and the change in these functions
on a finite interval.

Theorem 8.1 (Cauchy’s Mean Value Theorem). Let f and g be two functions defined on
the closed interval [a, b], such that

(i) f (x) and g(x) both are continuous on [a, b].

(ii) f (x) and g(x) both are differentiable on (a, b).

(iii) g ′ (x) ̸= 0 ∀x ∈ (a, b).

Then, there exist a point c ∈ (a, b) such that

f (b) − f (a) f ′ (c)


= ′ .
g(b) − g(a) g (c)
Proof. We assume that g(a) ̸= g(b). If g(a) = g(b), then g(x) satisfies all the conditions
of Rolle’s theorem. Hence, there exist c ∈ (a, b) such that g ′ (c) = 0 which contradict
the condition (iii) of the theorem. Thus, g(a) ̸= g(b). We define a function ϕ(x) =
f (x) + Ag(x), where, A is constant to be determined. Assume ϕ(a) = ϕ(b), then

f (a) + Ag(a) = f (b) + Ag(b)


f (b) − f (a)
A= . (8.1)
g(b) − g(a)

Also, ϕ satisfies all the conditions of Rolle’s theorem, i.e.

(i) Since f and g both are continuous on [a, b], hence ϕ(x) is continuous.
170 LESSON - 8. TAYLOR’S SERIES AND MACLAURIN’S SERIES

(ii) f and g are derivable on (a, b), therefore ϕ(x) is derivable on (a, b).

(iii) ϕ(a) = ϕ(b).

Thus, there exist c ∈ (a, b) such that ϕ′ (c) = 0 i.e.

f ′ (c) + Ag ′ (c) = 0
f ′ (c)
=⇒ −A = ′ (8.2)
g (c)

Therefore from, 8.1 and 8.2, we get

f (b) − f (a) f ′ (c)


= ′ . (8.3)
g(b) − g(a) g (c)

Remark. In the Cauchy’s Mean Value Theorem, if we take g(x) = x, then g ′ (x) = 1.
Also, g(b) = b and g(a) = a. Thus, from 8.3, we get

f (b) − f (a)
f ′ (c) =
b−a
which is the result of the Lagrange’s Mean Value Theorem. Hence the Lagrange’s Mean
Value Theorem is a particular form of the Cauchy’s Mean Value Theorem.

Example 8.1. Verify the Cauchy’s Mean Value Theorem for f (x) = x2 , g(x) = x3 in
[1, 2].

Solution. Here f (x) = x2 and g(x) = x3

(i) f and g being polynomial functions, they are continuous on [1, 2].

(ii) f and g being polynomial functions, they are derivable on (1, 2).

(iii) g ′ (x) = 3x2 ̸= 0 for all x ∈ (1, 2).

Thus, the conditions of Cauchy’s Mean Value Theorem are satisfied. Therefore, there exists
some point c ∈ (1, 2) such that

f (2) − f (1) f ′ (c)


= ′
g(2) − g(1) g (c)
4−1 2c
=⇒ = 2
8−1 3c
3 2
=⇒ =
7 3c
14
=⇒ c = , which lies in (1, 2).
9
Hence, Cauchy’s Mean Value Theorem is verified.
8.4. TAYLOR’S THEOREM 171

Example 8.2. Show that


sin α − sin β π
= cot θ, 0 < α < β <
cos α − cos β 2
Solution. Consider two functions f (x) and g(x):
π
f (x) = sin x, g(x) = cos x for all x ∈ [α, β], 0 < α < β < .
2
We apply Cauchy’s Mean Value Theorem on f (x) and g(x) on the interval [α, β]. We have
(i) f (x) = sin x and g(x) = cos x are continuous functions in the closed interval [α, β].
(ii) Since, f ′ (x) = cos x and g ′ (x) = − sin x. Therefore, both the functions f and g are
derivable in (α, β).
(iii) g ′ (x) = − sin x ̸= 0 for all x ∈ (α, β).
Thus, all the conditions of Cauchy’s Mean Value Theorem are satisfied and so there exists
some point θ ∈ (α, β), such that
f (β) − f (α) f ′ (θ)
= ′
g(β) − g(α) g (θ)
sin β − sin α cos θ
⇒ =
cos β − cos α − sin θ
sin β − sin α
⇒ = − cot θ
cos β − cos α
sin α − sin β π
⇒ = cot θ, where 0 < α < β < .
cos β − cos α 2
In-text Exercise 8.1. Solve the following questions:
1. Verify Cauchy’s Mean Value Theorem for
(i) f (x) = sin x, g(x) = cos x in [−π/2, 0].
(ii) f (x) = ex , g(x) = e−x in [0, 1].
2. Let the function ϕ be continuous in [a, b] and derivable in (a, b). Show that there
exists a point c in (a, b) such that
2c (ϕ(a) − ϕ(b)) = ϕ′ (c)(a2 − b2 ).

8.4 Taylor’s Theorem


In the lesson 5, we have discussed Mean Value Theorems, which use the first order deriva-
tives of functions. We generalize the concept to the functions those are differentiable k
times(say), successively and obtain Taylor’s theorem. Taylor’s theorem gives an approxi-
mation of a k-times differentiable function around a given point by a polynomial of degree
k, called the kth-order Taylor polynomial.
172 LESSON - 8. TAYLOR’S SERIES AND MACLAURIN’S SERIES

Theorem 8.2 (Taylor’s Theorem with Lagrange’s form of Remainder). If a function f


is defined on [a, a + h], such that

(i) f and the derivatives f ′ , f ′′ ,..., f (n−1) are continuous on [a, a + h],

(ii) the nth derivative f (n) exist on (a, a + h),

then there exists at least one point θ, 0 < θ < 1, such that

h2 ′′ hn−1 (n−1) hn
f (a + h) = f (a) + hf ′ (a) + f (a) + . . . + f (a) + f (n) (a + θh).
2! (n − 1)! n!

Proof. Consider the function

(a + h − x)2 ′′
F (x) = f (x) + (a + h − x)f ′ (x) + f (x) + ...
2!
(a + h − x)n−1 (n−1) (a + h − x)n
... + f (x) + A, (8.4)
(n − 1)! (n)!

where A is a constant to be chosen so that F (a) = F (a + h).

Now from equation (8.4), we have

h2 ′′ hn−1 (n−1) hn
F (a) = f (a) + hf ′ (a) + f (a) + . . . + f (a) + A, (8.5)
2! (n − 1)! n!
and F (a + h) = f (a + h). (8.6)

Therefore F (a) = F (a + h) gives

h2 ′′ hn−1 (n−1) hn
f (a + h) = f (a) + hf ′ (a) + f (a) + . . . + f (a) + A, (8.7)
2! (n − 1)! n!

which gives the value of A. With the choice of A, the function F (x) satisfies all the condi-
tions of Rolle’s Theorem on [a, a + h], as

1. F (x) is continuous on [a, a + h]. As f, f ′ , f ′′ . . . , f (n−1) are continuous on [a, a + h].

2. F (x) is differentiable on (a, a + h). As f, f ′ , f ′′ . . . , f (n−1) are differentiable on


(a, a + h).

3. F (a) = F (a + h)

Hence, from Rolle’s Theorem there exist at-least one real number θ, 0 < θ < 1 such that

F ′ (a + θh) = 0. (8.8)
8.4. TAYLOR’S THEOREM 173

Differentiating equation (8.4), w.r.t x, we get


F ′ (x) = f ′ (x)
+ (a + h − x)f ′′ (x) − f ′ (x)
(a + h − x)2 ′′′ 2(a + h − x)
+ f (x) + (−1)f ′′ (x)
2! 2!
+ .........
+ .........
(a + h − x)n−1 (n) (n − 1)(a + h − x)n−2 (n−1)
+ f (x) − f (x)
(n − 1)! (n − 1)!
n(a + h − x)n−1 (−1)A
+
n!
n−1
(a + h − x) (a + h − x)n−1
or F ′ (x) = f (n) (x) − A
(n − 1)! (n − 1)!
(a + h − x)n−1 (n)
= [f (x) − A]
(n − 1)!
For x = a + θh, we get

′ [h(1 − θ)]n−1 (n)


F (a + θh) = [f (a + θh) − A]. (8.9)
(n − 1)!
From (8.8) and (8.9), we get
f ′′ (a + θh) − A = 0 =⇒ A = f n (a + θh).
Therefore, substituting the above value of A in (8.7), we have
h2 ′′ hn−1 (n−1) hn
f (a + h) = f (a) + hf ′ (a) + f (a) + · · · + f (a) + f (n) (a + θh),
2! (n − 1)! n!
which is the required result.

Remark. Following are some important remarks:


hn (n)
1. The (n + 1)th term i.e. f (a + θh) is called the Lagrange’s remainder after n
n!
terms and is denoted by Rn . Thus, the above theorem is called Taylor’s Theorem
with Lagrange’s Form of Remainder
2. Putting n = 1 in Taylor’s theorem, we get f (a + h) = f (a) + hf ′ (a + θh) where
0 < θ < 1, which is the conclusion of Lagrange’s Mean Value Theorem. Hence we
conclude that Mean Value Theorem is a particular case of Taylor’s Theorem.
3. If the remainder Rn is expressed as
hn
Rn = (1 − θ)n−1 f (n) (a + θh), (8.10)
(n − 1)!
then the above theorem is called Taylor’s Theorem with Cauchy’s Form of Re-
mainder and Rn is called Cauchy’s Remainder after n terms.
174 LESSON - 8. TAYLOR’S SERIES AND MACLAURIN’S SERIES

4. Alternate form of Taylor’s Theorem


If we choose b = a + h then, h = b − a and c = a + θh = a + θ(b − a) and c ∈ (a, b).
Therefore, alternately Taylor’s Theorem can be concluded as follows

(b − a)2 ′′
f (b) = f (a) + (b − a)f ′ (a) + f (a) + · · ·
2!
(b − a)n−1 (n−1) (b − a)n (n)
+ f (a) + f (c), [a < c < b].
(n − 1)! n!

Example 8.3. By using Taylor’s Theorem with Lagrange’s form of remainder, show that
Show that

h h2 hn
log(x + h) = log x + − 2 + . . . + (−1)n−1 , 0 < θ < 1, h > 0.
x 2x n(x + θh)n

Solution. Let f (x + h) = log(x + h). Therefore,

f (x) = log x, (log x is natural logarithm)


1
f ′ (x) =
x
′′ 1
f (x) = − 2
x
(−1)(−2) 2!
f ′′′ (x) = 3
= (−1)2 3
x x
...
...
(n − 1)!
f (n) (x) = (−1)n−1
xn
(n − 1)!
∴ f (n) (x + θh) = (−1)n−1 .
(x + θh)n

Then, by using Taylor’s theorem with Lagrange’s form of remainder, we have

′ h2 ′′ h3 ′′′ h2 n − 1 n−1 hn n
f (x + h) = f (x) + hf (x) + f (x) + f (x) + . . . f (x) + f (x + θh).
2! 3! (n − 1)! n!

h h2 h3 2! hn−1 n−2 (n − 2)! hn (n − 1)!


∴ log(x + h) = log x + − 2+ 3
+ . . . (−1) n−1
+ (−1)n−1
x 2x 3!x (n − 1)! x n! (x + θh)n
h h2 h3 hn−1 hn
= log x + − 2 + 3 + . . . (−1)n−2 + (−1)n−1
.
x 2x 3x (n − 1)xn−1 n(x + θh)n

Hence, the result.


8.4. TAYLOR’S THEOREM 175

8.4.1 Taylor’s Infinite Series


If a function f (x) possesses continuous derivatives of all orders in (a, a+h) , then for every
integer n, howsoever large, there corresponds a Taylor’s development with Lagrange’s form
of remainder, viz
h2 ′′ hn−1 (n−1)
f (a + h) = f (a) + hf ′ (a) + f (a) + . . . + f (a) + Rn
2! (n − 1)!
where,
hn n
Rn = f (a + θh), 0 < θ < 1. (8.11)
n!
It may be written as f (a + h) = Sn + Rn ,

where
h2 ′′ hn−1 (n−1)
Sn = f (a) + hf ′ (a) + f (a) + . . . + f (a). (8.12)
2! (n − 1)!
Now if Rn converges to 0 as n → ∞, then

h2 ′′
lim Sn = f (a) + hf ′ (a) + f (a) + . . .
n→∞ 2!
and therefore, we can write
h2 ′′
f (a + h) = f (a) + hf ′ (a) + f (a) + . . . .
2!
This is known as Taylor’s infinite series expansion of f (a + h). It is stated as follows:

Theorem 8.3 (Taylor’s infinite series expansion). If a function f (x) defined on [a, a + h]
is such that

(i) f (x) possesses continuous derivatives of all orders in (a, a + h).


hn (n)
(ii) For 0 < θ < 1, Taylor’s remainder Rn = f (a + θh) tends to 0 as n → ∞,
n!

then
h2 ′′ hn
f (a + h) = f (a) + hf ′ (a) + f (a) + . . . + f (n) (a) + . . . . (8.13)
2! n!

Other Forms of Taylor’s Infinite Series :


1. Replacing a by x , in equation (8.13), we have

h2 ′′ hn
f (x + h) = f (x) + hf ′ (x) + f (x) + . . . + f (n) (x) + . . . (8.14)
2! n!
176 LESSON - 8. TAYLOR’S SERIES AND MACLAURIN’S SERIES

2. Putting a + h = b or h = b − a , in (8.13), we have

(b − a)2 ′′ (b − a)n (n)


f (b) = f (a) + (b − a)f ′ (a) + f (a) + . . . + f (a) + . . .
2! n!
(8.15)

3. Putting a + h = x or h = x − a , in (8.13), we have

(x − a)2 ′′ (x − a)n (n)


f (x) = f (a) + (x − a)f ′ (a) + f (a) + . . . + f (a) + . . .
2! n!
(8.16)

which expands f (x) in ascending integral powers of (x − a).

Example 8.4. Assuming the validity of expansion by Taylor’s series, show that

θ2 θ3
π  
 1
sin +θ = √ 1+θ− − + ··· .
4 2 2! 3!

Solution. Given, f (x) = sin x. Let a = π4 , h = θ, then by assuming the validity of expan-
sion, we have
h2 ′′ h3
f (a + h) = f (a) + hf ′ (a) + f (a) + f ′′′ (a) + . . .
2! 3!
or
h2 h3
sin(a + h) = sin a + h cos a + (− sin a) + (− cos a) + . . .
2! 3!
π
Putting a = 4
and h = θ, we obtain

θ2 θ3
π    
1
 θ 1 1
sin +θ = √ + √ + −√ + −√ + ...
4 2 2 2! 2 3! 2
Hence,
θ2 θ3
π  
1
sin +θ = √ 1+θ− − + ... .
4 2 2! 3!
Example 8.5. Assuming the validity of Taylor’s series expansion, show that
2
x − π4 π x − π4

−1 −1 π
tan x = tan + 2 − 2 2
+ ···
4 1 + π16 4 1+ π 16

Solution. Given
f (x) = tan−1 x
Then,
1
f ′ (x) = ,
1 + x2
2x
f ′′ (x) = − etc.
(1 + x2 )2
8.4. TAYLOR’S THEOREM 177

Now, f (x) can be written as


π π
f (x) = tan−1 x = tan−1 +x− = tan−1 (a + h),
4 4
π π
where a = , and h = x − .
4 4
Assuming the validity of Taylor’s series expansion, we have

h2 ′′
f (a + h) = f (a) + hf ′ (a) + f (a) + . . .
2!
2
−1 −1 π  π 1 π x − π4
∴ tan x = tan + x− −  + · · ·
4 4 1 + π162 4 1 + π2 2
16

In-text Exercise 8.2. Solve the following questions:

1. Assuming the validity of Taylor’s series expansion, show that

(x − π/2)2 (x − π/2)4
sin x = 1 − + − ...
2! 4!

2. Apply Taylor’s series expansion to prove

h (2x2 − 1) h2
sec−1 (x + h) = sec−1 x + √ − 2 2 . + ...
x x2 − 1 x (x − 1)3/2 2!

3. Apply Taylor’s series expansion to prove

h2 h3
 
1+h
e =e 1+h+ + + ...
2! 3!

4. Expand tan x in power of (x − π/4) up-to first four terms.

8.4.2 Maclaurin’s Theorem


If in the statement of Taylor’s Theorem 8.2, we put a + h = x and a = 0, then we get the
Maclaurin’s Theorem as stated below:

Theorem 8.4. If a function f (x) is defined on [0, x], such that

(i) f and its derivatives f ′ , f ′′ , f ′′′ , . . . , f (n−1) are continuous on [0, x].

(ii) f (n) (x) exist on (0, x),


178 LESSON - 8. TAYLOR’S SERIES AND MACLAURIN’S SERIES

then there exists at least one point θ, 0 < θ < 1 such that

x2 ′′ xn−1
f (x) = f (0) + xf ′ (0) + f (0) + . . . + · f (n−1) (0) + Rn ,
2! (n − 1)!
xn (n)
where Rn = f (θx), (Lagrange’s Remainder after n terms)
n!
xn (1 − θ)n−1 (n)
and Rn = f (θx), (Cauchy’s Remainder after n terms).
(n − 1)!
Example 8.6. Show that for every value of x

x3 x5 x2n−1 x2n
sin x = x − + + . . . + (−1)n−1 + (−1)n sin θx, 0 < θ < 1.
3! 5! (2n − 1)! (2n)!

Solution. Let f (x) = sin x. Then


n
 nπ 
f (x) = sin x + n ∈ N.
2
Now for n = 2m (even)
  π 
(n) (2n)
f (x) = f (x) = sin x + 2n · = sin (x + nπ)
2
=⇒ f n (0) = 0, when n is even.

Similarly, when n = 2m − 1 (odd), then


n−1
f n (0) = (−1) 2
or f 2n−1 (0) = (−1)n−1
Also f 2n (θx) = sin[θx + nπ] = (−1)n sin(θx)

Therefore, by using Maclaurin’s Theorem for f (x) = sin x, we get

x2 ′′
′ x3 ′′′ x2n−1 (2n−1) x2n (2n)
∴ f (x) = f (0) + xf (0) + f (0) + f (0) + . . . + f (x) + f (θx)
2! 3! (2n − 1)! (2n)!
That is
x2 x3 x2n−1 x2n
sin x = 0 + x · 1 +· 0 + (−1)1 + . . . + (−1)n−1 + (−1)n sin θx
2! 3! (2n − 1)! (2n)!
x 3 x5 x2n−1 x2n
=x− + + ... + (−1)n−1 + (−1)n sin θx.
3! 5! (2n − 1)! (2n)!

8.4.3 Maclaurin’s infinite series


Theorem 8.5 (Maclaurin’s infinite series expansion). If a function f defined on [0, h] is
such that
(i) f (x) possesses continuous derivatives of all order in [0, h],
8.5. MACLAURIN’S SERIES EXPANSION OF SOME STANDARD FUNCTIONS 179

(ii) Taylor’s remainder Rn → 0 as n → ∞,

then for each x ∈ [0, h]

x2 ′′ xn
f (x) = f (0) + xf ′ (0) + f (0) + . . . + f (n) (0) + . . .
2! n!

This is called Maclaurin’s infinite series expansion of f (x).

8.5 Maclaurin’s Series Expansion of Some Standard Func-


tions
8.5.1 Maclaurin’s Series Expansion of ex
Let f (x) = ex x ∈ R, then

f ′ (x) = ex , f ′′ (x) = ex , . . . , f (n) (x) = ex ∀n ∈ N (8.17)

=⇒ f (n) (x) exist for all n ∈ N and they are continuous as ex is continuous.
Also,
f ′ (0) = e0 = 1, f ′′ (0) = e0 = 1, . . . , f (n) (0) = e0 = 1 ∀n ∈ N (8.18)

The Lagrange’s form of remainder Rn is given by

xn (n)
Rn = f (θx), 0 < θ < 1 (8.19)
n!
xn θx
= e 0 < θ < 1.
n!
xn
→ 0 as n → ∞ as = 0 (from example 6.18).
n!

Thus, all the conditions of Maclaurin’s Series Expansion are satisfied. Therefore, we have
the expansion
x2 ′′ x3
f (x) = f (0) + xf ′ (0) + f (0) + f ′′′ (0) + · · ·
2! 3!
Hence, substituting the value of the function and its derivatives from 8.18, we get

x2 x3
ex = 1 + x + + + ··· ∀x ∈ R
2! 3!

x
X xn
or e =
n=0
n!
180 LESSON - 8. TAYLOR’S SERIES AND MACLAURIN’S SERIES

8.5.2 Maclaurin’s Series Expansion of sin x


Let f (x) = sin x, then


 π
f (x) = cos x = sin x + ,
2
f ′′ (x) = − sin x = sin (x + π),
 
′′′ 3π
f (x) = − cos x = sin x + ,
2
..
.
 nπ 
f (n) (x) = sin x + , ∀n ∈ N. (8.20)
2

Therefore, f (x) and its derivatives f (n) (x), n ∈ N exist and they are continuous as sin x is
continuous for all x ∈ R. Also,

f (0) = sin 0 = 0, f ′ (0) = cos 0 = 1, f ′′ (0) = − sin 0 = 0, f ′′′ (0) = −1 · · · (8.21)

The Lagrange’s form of remainder after n terms for f (x) = sin x is

xn (n)
Rn = f (θx), 0 < θ < 1 (8.22)
n!
xn
 
 nπ 
=⇒ Rn = sin θx + 0 < θ < 1 (by using (8.20))
n! 2
n 
x nπ 
=⇒ |Rn − 0| = · sin θx +
n! 2

n
x

n!
→ 0 as n → ∞ (by using example 6.18) ,
=⇒ lim Rn = 0.
n→∞

Thus, all the conditions of Maclaurin’s Series Expansion are satisfied. Therefore, we have
the expansion
x2 ′′ x3
f (x) = f (0) + xf ′ (0) + f (0) + f ′′′ (0) + · · ·
2! 3!
Hence, substituting the value of the function and its derivatives, we get

x3 x5 x7
sin x = x − + − + ··· ∀x ∈ R
3! 5! 7!

X (−1)n x2n+1
or sin x = .
n=0
(2n + 1)!
8.5. MACLAURIN’S SERIES EXPANSION OF SOME STANDARD FUNCTIONS 181

8.5.3 Maclaurin’s Series Expansion of cos x


Let f (x) = cos x, then
 π
f ′ (x) = − sin x = cos x + ,
2
f ′′ (x) = − cos x = cos (x + π),
 
′′′ 3π
f (x) = sin x = cos x + ,
2
..
.
 nπ 
f (n) (x) = cos x + , ∀n ∈ N (8.23)
2
Therefore, f (x) and its derivatives f (n) (x), n ∈ N exist and they are continuous as cos x is
continuous for all x ∈ R. Also,
f (0) = cos 0 = 1, f ′ (0) = − sin 0 = 0, f ′′ (0) = − cos 0 = 1, f ′′′ (0) = 0 · · · (8.24)
The Lagrange’s form of remainder after n terms for f (x) = cos x is
xn (n)
Rn = f (θx), 0 < θ < 1. (8.25)
n!
xn  nπ 
=⇒ Rn = · cos θx + 0 < θ < 1 (by using (8.23))
n!n 2
x nπ
|Rn − 0| = · cos θx +

n! 2

n
x

n!
→ 0 as n → ∞. (from exmaple 6.18).
=⇒ lim Rn = 0. (8.26)
n→∞

Thus, all the conditions of Maclaurin’s Series Expansion are satisfied. Therefore, we have
the expansion
′ x2 ′′ x3 ′′′
f (x) = f (0) + xf (0) + f (0) + f (0) + · · ·
2! 3!
Hence, substituting the values of the function and its derivatives, we get
x2 x4 x6
cos x = 1 − + − + ··· ∀x ∈ R
2! 4! 6!

X (−1)n x2n
or cos x = .
n=0
(2n)!

8.5.4 Maclaurin’s Series Expansion of loge (1 + x) ≡ ln(1 + x)


Let f (x) = loge (1 + x), then

1 −1 (−1)n−1 (n − 1)!
f ′ (x) = , f ′′ (x) = , . . . , f (n)
(x) = ∀n ∈ N and x > −1.
(1 + x) (1 + x)2 (1 + x)n
182 LESSON - 8. TAYLOR’S SERIES AND MACLAURIN’S SERIES

Therefore, f (x) and its derivatives f (n) (x), n ∈ N are continuous for x > −1. Also,

f (0) = 0, f ′ (0) = 1, f ′′ (0) = −1, f ′′′ (0) = 2, . . . , f (n) (0) = (−1)n−1 (n − 1)! ∀n ∈ N
(8.27)
Now, we have two cases :

Case 1 : 0 ≤ x ≤ 1

The Lagrange form of remainder after n terms is

xn (n)
f (θx)
Rn = (8.28)
n!
xn (−1)n−1 (n − 1)! (n) (−1)n (n − 1)!
=⇒ Rn = · as f (θx) =
n! (1 + θx)n (1 + θx)n
n
(−1)n−1
 
x
= .
n 1 + θx
 
x
Since, 0 < θ < 1 and 0 ≤ x ≤ 1, therefore 0 ≤ (1+θx)
< 1. Therefore,
 n
x
→ 0 as n → ∞ (using lim (rn ) = 0, for 0 < r < 1.)
1 + θx n→∞

Also,
1
→ 0 as n → ∞.
n
Hence, n
(−1)n−1

x
Rn = → 0 as n → ∞.
n 1 + θx
Case 2 : −1 < x < 0

For this case, we will consider Cauchy’s form of remainder. That is

xn
Rn = (1 − θ)n−1 f n (θx), 0 < θ < 1
(n − 1)!
xn (−1)n−1 (n − 1)!
= (1 − θ)n−1 ·
(n − 1)! (1 + θx)n
 n−1
n−1 1−θ 1
= (−1) · xn · .
1 + θx 1 + θx

Since −1 < x < 0 and 0 < θ < 1, therefore

−θ < θx =⇒ 1 − θ < 1 + θx, Also 1 − θ > 0


8.5. MACLAURIN’S SERIES EXPANSION OF SOME STANDARD FUNCTIONS 183
 
1−θ
=⇒ 0 < <1 (8.29)
1 + θx
Therefore,
 n−1
1−θ
→ 0 as n → ∞ (using lim rn → 0, as n → ∞ for 0 < r < 1.)
1 + θx n→∞

Also, −1 < x < 0 ∴ xn → 0 as n → ∞. Therefore,


 n
n 1−θ
x → 0 as n → ∞.
1 + θx
Thus,  n−1
n−1 n 1−θ 1
Rn = (−1) x · → 0 as n → ∞.
1 + θx 1 + θx
Finally, combining Case 1 and Case 2, we get,

Rn → 0 as n → ∞.

Hence, both the condition of Maclaurin’s Series Expansion are satisfied. Therefore, we
have the expansion as
x2 ′′
f (x) = f (0) + xf ′ (0) + f (0) + . . .
2!
x2 x3 xn
=⇒ loge (1 + x) = 0 + x + (−1) + · 1 + . . . + (−1)n−1 (n − 1)! + . . .
2! 3! n!
2 3 4 n−1 n
x x x (−1) x
=x− + − + ... + + ...
2 3 4 n
or,

X (−1)n−1 xn
loge (1 + x) = .
n=1
n

8.5.5 Maclaurin’s Series Expansion of (1 + x)m , |x| < 1


Let f (x) = (1 + x)m . We have the following two cases:

Case 1 : ‘m’ is a positive integer

Since

f (x) = (1 + x)m ,
f ′ (x) = m(1 + x)m−1 ,
f ′′ (x) = m((m − 1)(1 + x)m−2 ,
..
.
f (m) (x) = m!,
184 LESSON - 8. TAYLOR’S SERIES AND MACLAURIN’S SERIES

=⇒ f (n) (x) = 0 ∀ n > m. (8.30)

The Lagrange’s form of remainder after n terms is


xn (n)
Rn = · f (θx), 0 < θ < 1.
n!
xn
For n → ∞ we have n > m. Therefore f (n) (θx) = 0 (using (8.30)). Also → 0 as
n!
n → ∞. (using 6.18). Thus, in this case we get

Rn → 0 as n → ∞.

Hence, both the condition of Maclaurin’s Series Expansion are satisfied in this case. There-
fore
x2 ′′ xm m
f (x) = f (0) + xf ′ (0) +
f (0) + . . . + f (0)
2! m!
x2 xm
= 1 + xm + m(m − 1) + . . . + m!
2! m!
m(m − 1)x2
= 1 + mx + + . . . + xm .
2!
Thus, if m, is a fixed positive integer, then we get a finite series expansion of (1 + x)m .

Case 2 : ‘m’ is not a positive integer.

We have,

f (x) = (1 + x)m
f ′ (x) = m(1 + x)m−1
f ′′ (x) = m((m − 1)(1 + x)m−2
..
.
f (n) (x) = m(m − 1)(m − 2) . . . (m − n + 1)(1 + x)m−n ∀n ∈ N

Also,

f (0) = 1, f ′ (0) = m, f ′′ (0) = m(m − 1), . . . , f (n) (0) = m(m − 1) . . . (m − n + 1)

Here, we will use Cauchy’s form of remainder. Therefore


xn
Rn = (1 − θ)n−1 · f n (θx)
(n − 1)!
n−1
n (1 − θ) m(m − 1) . . . (m − n + 1)
=x n−m
·
(1 + θx) (n − 1)!
 n−1
1−θ m(m − 1) . . . (m − n + 1)
= xn (1 + θx)m−1 (8.31)
1 + θx (n − 1)!
8.5. MACLAURIN’S SERIES EXPANSION OF SOME STANDARD FUNCTIONS 185

Since,
|x| < 1, ∴ xn → 0 as n → ∞. (8.32)
Also, from equation (8.29)
1−θ
0< <1
1 + θx
Therefore  n−1
1−θ
→ 0 as n → ∞ (8.33)
1 + θx
Also,

for m > 1, (1 + θx)m−1 < (1 + |x|)m−1 and


for m < 1, (1 + θx)m−1 < (1 − |x|)m−1 (8.34)

=⇒ (1 + θx)m−1 is a finite real number. Thus, using above equations 8.32, 8.33, 8.34 in
8.31, we get
Rn → 0 as n → ∞ for |x| < 1
Hence, both the condition of Maclaurin’s Series Expansion are satisfied. Therefore, we
have

′ x2 ′′
f (x) = f (0) + xf (0) + f (0) + . . .
2!
m(m − 1) 2 m(m − 1)(m − n + 1) n
(1 + x)m = 1 + mx + x + ... x + ...
2! n!
Thus, for non positive integral value of ‘m’, we get a infinite series expansion of (1 + x)m .
Example 8.7. Assuming the validity of expansion by Maclaurin’s series, prove that

2x3 22 x4 22 x5
en cos x = 1 + x − − − + ···
3! 4! 5!
Solution. Since Maclaurin’s expansion is valid for the function f (x) = ex cos x, therefore

′ x2 ′′ x3 ′′′
f (x) = f (0) + xf (0) + f (0) + f (0) + . . . (8.35)
2! 3!
We have

f (x) = ex cos x,
f ′ (x) = ex cos x − ex sin x
√ x 1
 
1
= 2 e √ cos x − √ sin x
2 2
√ x  π
= 2 e cos x +
4
Similarly, we get
n
 nπ 
f n (x) = 2 2 ex cos x + , n ∈ N.
4
186 LESSON - 8. TAYLOR’S SERIES AND MACLAURIN’S SERIES

Also,

f (0) = 1, f ′ (0) = 1, f ′′ (0) = 0, f ′′′ (0) = −2, f iv (0) = −4, f v (0) = −4 etc.

Substituting these values of f (0), f ′ (0), f ′′ (0), f ′′′ (0), f iv (0) and f v (0) in equation (8.35),
we get
2x3 22 x4 22 x5
ex cos x = 1 + x − − − + ···
3! 4! 5!
Example 8.8. Assuming the validity of expansion, expand ax and ex in powers of x by
Maclaurin’s theorem.

Solution. Let
f (x) = ax =⇒ f (0) = 1
f ′ (x) = ax log a =⇒ f ′ (0) = log a
f ′′ (x) = ax (log a)2 =⇒ f ′′ (0) = (log a)2
f ′′′ (x) = ax (log a)3 =⇒ f ′′′ (0) = (log a)3

and so on. Therefore, by Maclaurin’s series expansion, we have

x2 ′′ x3
f (x) = f (0) + xf ′ (0) + f (0) + f ′′′ (0) + . . .
2! 3!

x2 x3
Therefore, a = 1 + x(log a) + (log a) + (log a)3 + . . .
x 2
(8.36)
2! 3!
Putting a = e and log a = log e = 1 in equation (8.36), we get the Maclaurin’s series
expansion of ex as
x2 x3
ex = 1 + x + + + ...
2! 3!
In-text Exercise 8.3. Solve the following questions:

1. Assuming the validity of expansion by Maclaurin’s series, show that


x2 x4
(i) log sec x = 2
+ 12
+ ...
x2 x3 x4
(ii) log(1 + sin x) = x − 2
+ 6
− 12
+ ...
ex
2. Expand by Maclaurin’s theorem as far as the term containing x3 .
ex + 1
3. Show that by means of Maclaurin’s series expansion, that

x x2 x4
log(1 + ex ) = log 2 + + − + ...
2 8 192

4. Assuming the validity of Maclaurin’s series expansion, find the series expansion of
f (x) = e2x for all real values of x.
8.6. SUMMARY 187

8.6 Summary
In this lesson, we have discussed following topics:

1. Cauchy’s Mean Value Theorem: Let there be two functions,f (x) and g(x). These
two functions shall be continuous on the interval, [a, b], and these functions are dif-
ferentiable on the range (a, b), and g ′ (x) ̸= 0 for all x ∈ (a, b). Then there will be a
f (b) − f (a) f ′ (c)
point x = c in the given range or the interval such that, = ′
g(b) − g(a) g (c)
2. Taylor’s Theorem with Lagrange’s Form of Remainder: If a function f is defined on
[a, a + h], such that

(i) f and the derivatives f ′ , f ′′ ,..., f (n−1) are continuous on [a, a + h],
(ii) the nth derivative exist on (a, a + h),

then there exists at least one point θ, 0 < θ < 1 such that

h2 ′′ hn−1 (n−1) hn
f (a + h) = f (a) + hf ′ (a) + f (a) + . . . + f (a) + f (n) (a + θh).
2! (n − 1)! n!
hn (n)
The (n + 1)th term i.e. f (a + θh) is called the Lagrange’s remainder after n
n!
terms and is denoted by Rn .

3. Taylor’s Theorem with Cauchy’s Form of Remainder: If the remainder Rn is ex-


pressed as
hn
Rn = (1 − θ)n−1 f (n) (a + θh), (8.37)
(n − 1)!
then the above theorem is called Taylor’s Theorem with Cauchy’s Form of Re-
mainder and Rn is called Cauchy’s Remainder after n terms.

4. Taylor’s series Expansions of Functions: If a function f (x) is such that

(i) f (x) possesses continuous derivatives of all orders in [a, a + h].


hn (n)
(ii) For 0 < θ < 1, Taylor’s remainder Rn = n!
f (a + θh) tends to 0 as n → ∞,

then

′h2 ′′ hn (n)
f (a + h) = f (a) + hf (a) + f (a) + . . . + f (a) + . . . . (8.38)
2! n!
5. Maclaurin’s series Expansions of Functions: If a function f defined on [0, h] is such
that

(i) f (x) possesses continuous derivatives of all order in [0, h],


(ii) Taylor’s remainder Rn → 0 as n → ∞,
188 LESSON - 8. TAYLOR’S SERIES AND MACLAURIN’S SERIES

then for each x ∈ [0, h]

′ x2 ′′ xn (n)
f (x) = f (0) + xf (0) + f (0) + . . . + f (0) + . . .
2! n!
This is called Maclaurin’s infinite series expansion of f (x).

8.7 Self-Assessment Exercises


1. Verify Cauchy’s Mean Value Theorem for the following functions:
1
f (x) = x(x − 1)(x − 2), g(x) = x(x − 2)(x − 3), a = 0, b = .
2

2. Check the validity of Cauchy’s Mean Value Theorem for the following functions:

f (x) = x4 , g(x) = x2 , a = 1, b = 2.

3. Show that
(1 − x)−1 = 1 + x + x2 + x3 + ...

4. For −1 < x ≤ 1, show that


x2 x3 x4
log(1 − x) = −x − − − − ...
2! 3! 4!
9
5. If f (x) = x3 + 2x2 − 5x + 11, find the value of f ( 10 ) with the help of Taylor’s series
for f (x + h).

6. Apply Maclaurin’s theorem to prove

x4
cos2 x = 1 − x2 + + ...
3

7. prove eax cos bx is equal to

x2 3
xn 2 2 n/2 aθx
 
2 22 2 x −1 b
1+ax+(a −b ) +a(a −3b ) +...+ (a +b ) e cos bθx + n tan .
2! 3! n! a

8. Expand eax sin bx by Maclaurin’s theorem with Cauchy’s form of remainder.

8.8 Solutions to In-text Exercises


Exercise 8.1
1. (i) c = − π4 ∈ (−π/2, 0).
1
(ii) c = 2
∈ (0, 1).
8.9. SUGGESTED READINGS 189

2. Let g(x) = x2 , then using Cauchy’s Mean Value Theorem given result can be ob-
tained.

Exercise 8.2
π π
1. f (x) = sin x = sin +x− = sin(a + h), where a = π/2 and h = x − π/2.
2 2
2. Using Taylor expansion, we can obtained the required expansion.

3. e1+h = e(eh )
 π  π 2
4. tan x = 1 + 2 x − +2 x− + ...
4 4
Exercise 8.3

1. (i) By taking f (x) = log sec x and substituting in Maclaurin’s expansion, required
result can be obtained.
(ii) Use the expansion of log(1 + x) to prove it.
1 x x3
2. Expansion is + − + ...
2 4 48
3. Use the expansion of log(1 + x) to prove it.
(2x)2 (2x)3 (2x)4
4. Expansion is e2x = 1 + 2x + + + + ...
2! 3! 4!

8.9 Suggested Readings


1. Narayan, Shanti (Revised by Mittal, P. K.). Differential Calculus. S. Chand, Delhi,
2019.

2. Prasad, Gorakh (2016). Differential Calculus (19th ed.) Pothishala Pvt. Ltd. Alla-
habad.

3. Thomas Jr., George B., Weir, Maurice D.,Hass, Joel (2014). Thomas Calculus (13th
ed.). Pearson Education, Delhi. Indian Reprint 2017.
190 LESSON - 9. REDUCTION FORMULAE FOR INTEGRALS

Lesson - 9
Reduction Formulae for Integrals
Ms. Parvinder Kaur

Structure
9.1 Learning Objectives
9.2 Introduction
9.3 Basic Properties of Definite Integrals
n n
9.4 Reduction Formulae For  sin x dx and  cos x dx

n
9.5 Reduction Formulae For and  cot x dx
m
9.6 Reduction Formulae For  sin x cosn xdx

9.7 Applications of the Reduction Formulae


9.8 Summary
9.9 Self-Assesment Exercises
9.10 Solutions to In-Text Exercises
9.11 Suggested Readings

9.1 Learning Objectives


The learning objectives of the lesson are to:
 learn to evaluate the integrals of higher order, such as

 sinn x dx ,  cosn x dx,  cotn x dx ,  tann x dx,  secn x dx

 learn to evaluate the integrals of algebraic expressions of the type


a 
dx
 x 2 ax  x 2 dx ,  , a2  x 2 , x 2  a2 , x 2  a2
0 0
(a  x 2 )7
2

 learn to evaluate the integrals of product of trigonometric functions such as


m
 sin x cosn x dx

 to reduce complicated integrals into simpler integrals by using reduction


formulae.
191

9.2 Introduction
A reduction formula for an integral is a technique by means of which we can reduce the
given integral to some known integral. Successive application of a reduction formula
connects an integral with another integral of lower order which helps in evaluating the
given integral. The importance of this chapter lies in the fact that it plays a pivotal role in
Integral calculus, which has a wide range of applications in finding areas of irregular
plane regions, length of curves, volume of solid of revolution, surface area of solid of
revolution and many more miscellaneous problems of integrals. The basic steps involved
in obtaining a reduction formula is Integration by parts. We illustrate this with the help of
examples. We have also discussed some applications of the reduction formulae in solving
tedious integrals.

9.3 Basic Properties of Definite Integrals


Following are some basic properties of definite integrals;
b a
(i)  f (x )dx    f (x )dx
a b
b c b
(ii)  f (x )dx   f (x )dx   f (x )dx
a a c
b b b
(iii)  f (x )dx   f (t )dt   f (y )dy
a a a
a a
(iv) 
0
f ( x )dx   f (a – x )dx
0

Proof: It is known as Invariance property.


Put x  a  t such that dx  dt, for x  0, t  a and x  a, t  0 then L.H.S. become
a 0

 f (x )dx    f (a  t )dt
0 a
a
  f (a  t ) dt by property (i)
0
a
  f (a  x )dx by property (iii)
0
192 LESSON - 9. REDUCTION FORMULAE FOR INTEGRALS

a a
(v)  f (x )dx  2  f (x )dx if f (x ) is even i.e. f (x )  f (x )
a 0

=0 if f (x ) is odd i.e. f (x )   f (x )


a 0 a
Proof:  f (x )dx   f (x )dx   f (x )dx by property (ii)
a a 0

Putting x  t  dx  dt , for x  0, t  0 and x  a, t  a


0
in the integral  f (x )dx above, we get
a
0 9

 f (x )dx    f (t )dt


a a
a
  f (t )dt by property (i)
0
a
  f (x )dx by property (iii)
0
a a a
Thus  f (x )dx   f (x ) dx   f (x )dx becomes
a 0 0
a
  [ f (x )  f (x )]dx
0
a
 2  f (x )dx if f (x ) is even i.e. f (x )  f (x )
0

=0 if f (x ) is odd i.e. f (x )   f (x )


2a a
(vi)  f (x )dx  2  f (x )dx if f (2a  x )  f (x )
0 0

=0 if f (2a  x )   f (x )
Proof: We put x  2a  t , such that dx   dt .

For x  a, t  a and x  2a, t  0 in the second integral on the R.H.S. above, we get
2a a 0

 f (x )dx   f (x )dx   f (2a  t )dt


9 0 a
193

a a
  f (x )dx   f (2a  t )dt by property(i)
0 0
a a
  f (x )dx   f (2a  x )dx (t is a dummy variable).
0 0
2a a
Hence,  f (x )dx   [ f (x )  f (2a  x )]dx
0 0
a
 2  f (x )dx if f (2a  x )  f (x )
0

=0 if f (2a  x )   f (x )

n n
9.4 Reduction Formulae For  sin x dx and  cos x dx , nϵ  .

Consider the integrals and , n ϵ  , If n is odd, for example


n = 3, then substituting cos x = u, we get
–sin xdx = du
=
=
=–

= –u + + c , where c is a constant.
In general , if n = 2m + 1 (odd) , then the substitution cos x = u transforms the integral
to the integral , which can be easily evaluated by using the
binomial theorem for the expansion of .
If n is even and a larger number, then the process discussed above is not of much help.
Therefore, in such cases we resort to the reduction formulae.
n
Reduction Formula For  sin x dx

We have

 sinn xdx   sinn 1 x  sin x dx , n >1

Integrating by parts by taking sinn 1 x as first function and sin x as the second function,
we get
= (-cos x)-(n – 1) cos x (-cos x) dx
194 LESSON - 9. REDUCTION FORMULAE FOR INTEGRALS

=− cos x +(n – 1) dx

  cos x sinn 1 x  (n  1) sinn 2 x (1  sin2 x )dx

= – cos x +(n – 1) dx – (n – 1)
(1 + n – 1) = - cosx +(n – 1) dx
= - cosx +(n – 1) dx
This gives the reduction formula for in the form

 cos x sinn 1 x  n  1 
 sinn xdx   n 2
  sin xdx (9.1)
n  n 
Here, we note that the problem of evaluating is reduced to the problem of
evaluating dx, with reduced degree of sin x.
 /2
Example 9.1 Obtain a reduction formula for the integral  sinn x dx by using the
0
Reduction formula (9.1).
Solution. By using the Reduction formula (9.1)
We have
 /2  /2  /2
 cos x sinn 1 x n  1
In = 
n
sin dx      sinn  2 x dx
0
n 0
 n  0

That is
n 1
In  I (9.2)
n n 2
n 3
 I n 2  I (replacing n by n  2 in (9.2))
n  2 n 4
n 5
 In 4  I ... (replacing n by n  4 in (9.2))
n  4 n 6
Continuing in the same manner, we get
2
I3  I , when n is odd.
3 1
1
I2  I , when n is even
2 0
195

 /2
Now I1   sin x dx  (cos x )0 / 2  1
0

 /2  /2
0 
and I0   sin x dx   1dx 
0 0
2

Therefore, using all I 0 , I 1, I 2 , I 3 ,....I n  4 , I n  2 in (9.2), we get the desired reduction


formula as
 n  1  n  3  n  5 .... 2 , if n is odd
 /2
n  n n  2 n  4 3
 sin x dx   (9.3)
0  n  1  n  3  n  5 .... 1   , if n is even
 n n 2 n 4 2 2

Example 9.2 Evaluate


Solution. By taking n=7 (odd) in the formula (9.3) , we get
= . . =

n
Reduction Formula for  cos x dx .

We have

  cosn 1 x cos x dx , n>1

integrating by parts by taking cosn 1 x as the first function and cos x as the second
function, we get

 cosn 1 x  sin x   (n  1)cosn 2 x ( sin x )sin x dx

 cosn 1 x  sin x  (n  1) cosn 2 x sin2 x dx

 cosn 1 x sin x  (n  1) cosn 2 x (1  cos2 x )dx

 cosn 1 x sin x  (n  1) cosn 2 xdx  (n  1) cosn xdx

(1 + n – 1) = + (n – 1) dx
This gives the reduction formula for

+ dx (9.4)
196 LESSON - 9. REDUCTION FORMULAE FOR INTEGRALS

Here, we note that the problem of evaluating is reduced to the problem of


evaluating dx, with reduced degree of cos x.
 /2
Example 9.3: Obtain a reduction formula for  cosn x dx by using the reduction
0
formula (9.4).
Solution. By using the reduction formula (9.4), we have
 /2
In   cosn xdx = +
0

That is
n 1
In  I (9.5)
n n 2
n 3
I n 2  I (replacing n by n  2 in (9.5))
n  2 n 4
n 5
I n 4  I (replacing n by n  4 in (9.5))
n  4 n 6
Continuing like this, we get
2
I3  I, if n is odd
3 1
1
I2  I , if n is even
2 0
 /2
Now, I 1   cos xdx  (sin x )0 / 2  1
0

 /2  /2
0 
I0   cos xdx   1dx  .
0 0
2

Using all the above values of I 0 , I 1, I 2 , I 3 ,...I n  4 , I n  2 in (9.5), we get the desired
reduction formula as
 n  1  n  3 ... n 5 2
 , if n is odd
 /2

 cosn dx   n n 2 n 4 3
(9.6)
 n  1 n 3 n 5 1 
0  ...   , if n is even
 n n 2 n 4 2 2
197

Example 9.4 Evaluate .

Solution. By taking n=7 (odd) in the formula (9.6) , we get

= . . =
In-text Exercise 9.1 Solve the following questions:
 /2
1. Evaluate  sin6  d .
0

2. Evaluate .

3. Find the vaue of

4.

9.5 Reduction Formulae For  tann x dx and n


 cot x dx .
(i) To obtain the reduction formula for  tann x dx , we write the integral as

 tann x dx   tann 2 x tan2 x dx

  tann 2 x (sec2 x  1)dx

  tann  2 x sec2 x dx   tann 2 x dx (9.7)

Now, to evaluate the integral on the right side of the above equation, we substitute
tan x  t  sec2 x dx  dt

Therefore =

= (9.8)

Therefore, using (9.8) in (9.7) , we get the desired reduction formula as

 tann x dx = - ,n>2 (9.9)


198 LESSON - 9. REDUCTION FORMULAE FOR INTEGRALS

Example 9.5 Obtain a reduction formula for


 /4

 tann x dx , for n > 1.


0

By using the Reduction formula (9.9).

Solution. By using the reduction formula (9.9) , We


 /4  /4
1  /4
have  tann x dx  tann 1 x   tann  2 x dx
0
n 1 0
0

 /4  /4
n 1
or  tan xdx    tann  2 x dx (9.10)
0
n 1 0

 /4
taking I n   tann x dx in (9.10), we get
0

1
In  I , for n > 1. (9.11)
n  1 n 2

Which is the desired reduction formula.


n
(ii) To obtain the reduction formula for  cot x dx , we write

n n 2
 cot x dx   cot x  cot2 x dx

  cotn 2 x (cosec2x  1)dx (9.12)

=
Now to evaluate the integral on the right side of the above
equation, we substitute
cotx = t  –cosec2 xdx = dt
Therefore , =

= (9.13)

Therefore, using (9.13) in (9.12), we get the desired reduction formula.


199

n
 cot x dx = - , n >1. (9.14)

Example 9.6 Evaluate

Solution. Using the reduction formula (9.14) for , we have


2
2  cot 3 x 
 4  cot x  x 
4
cot xdx = –
 3  4

   1 
= 0  0   –  – 1  
 2 3
  4 
 2
= –
4 3
3 – 8 .
=
12
The process used to obtain the reduction formulae till now can be extended to obtain
the reduction formulae for  secn x dx and . Let us consider

 secn x dx   secn 2 x sec2 x dx

Integrating it by parts, taking sinn 2 x as first function and sec2 x as second


function, we get

 secn 2 x tan x   (n  2)secn  3 x sec x tan x  tan x dx

 secn 2 x tan x  (n  2) secn 2 x tan2 x dx

 secn 2 x tan x  (n  2) secn 2 x (sec2 x  1)dx

 secn 2 x tan x  (n  2) secn x dx  (n  2) secn 2 x dx

= tanx +(n – 2)
tanx +(n – 2) dx

= + (9.15)

Similarly, we can obtain the reduction formula

= + (9.16)
200 LESSON - 9. REDUCTION FORMULAE FOR INTEGRALS

Example 9.7 Evaluate the following integrals:


(i)
(ii)
Solution(i) Using the reduction formula for ,we have

= +

+ .

Solution(ii) ) Using the reduction formula for

= + dx

= - cotx .

In-text Exercise 9.2 Solve the following questions:


1. Evaluate the following integrals:

(ii)
(iii)
(iv)
 /4
2. Evaluate I   tan 5  d
0

3. Evaluate
m
9.6 Reduction Formula for  sin x cosn xdx , m,n 

We write

I m,n   sinm x cosn x dx

or I m,n   sinm 1 x (sin x cosn x )dx

Integrating by parts by taking, sinm 1 x as the first function and sinx cosn x as the
second function, we get
201

cosn 1 x m  1
I m,n   sinm x cosn x dx   sinm 1 x  cosn 1 x sinm 2 x  cos x dx
n 1 

n 1

cosn 1 x sinm 1 x m  1
   sinm 2 x cosn x (1  sin2 x )dx
n 1 n 1

cosn 1 x sinm 1 x m  1 m 1
   sinm 2 x cosn x   sinm x cosn x dx
n 1 n 1 n 1
cosn 1 x sinm 1 x m  1 m 1
I m,n     sinm 2 x cosn x  I m,n
n 1 n 1 n 1
m  1  cosn 1 x sinm 1 x m  1
  1   sinm
x cosn
x dx    sinm 2 x cosn x dx
 n 1 n 1 n 1
m n cosn 1 x sinm 1 x m  1
or  sinm x cosn xdx     sinm 2 x cosn x dx
n 1 n 1 n 1
 cosn 1 x sinm 1 x m 1
 I m ,n   I (9.17)
m n m  n m 2,n
That is,
 cosn 1 x sinm 1 x m  1
 sinm x cosn x dx    sinm 2 x cosn x dx (9.18)
m n m n

Which is the required reduction formula.


 /2
Example 9.8 Evaluate  sinm x cosn x dx , Where m, n N.
0

 /2
Solution. Let Im, n =  sinm x cosn x dx
0

By using the reduction formula (9.18), we get


 /2
 /2
m n
 cosn 1 x sinm 1 x m 1
 /2
I m ,n   sin x cos x dx    sinm  2 x cosn x dx
0
m n m n 0
0

 /2  /2
m m 1
n
=  sin x cos x dx   sinm  2 x cosn x dx
0
m n 0
202 LESSON - 9. REDUCTION FORMULAE FOR INTEGRALS

m 1
or I m ,n  I (9.19)
m  n m 2,n
m 3
Now I m 2,n  I (replacing m by m-2 in (9.19))
m  n  2 m  4,n
m 5
I m  4,n  I (replacing m by m-4 in (9.19)
m  n  4 m 6,n
Continuing like this ,we get

2
I 3,n  I if m is odd
3  n 1,n
1
I 2,n  I if m is even
2  n 0,n
Using the above values of I 2,n , I 3,n ,...., I m  4,n , I m  2,n in (9.19), we get

 m  1  m  3  m  5 ... 2 I If m is odd
 1,n
I m,n  m  n m  n  2 m  n  4 3  n
 m  1 . m  3  m  5 ... 1 I if m is even
 m  m m  n  2 m  n  4 2  n 0,n
 /2
1
Now, I 1,n   sin x cosn xdx 
0
n 1

put cos x  t   sin xdx  dt , for x = 0 , t = 1 as cos(0)  1

For x = π/2 , t = 0 as cos(π/2) = 0.


0 1
n n
I 1,n    t dt  t dt (Using the property (i) of definite integrals)
1 0
1
 t n 1  1 1
   0
 n  1 0 n  1 n 1

1
 I 1,n 
n 1
 n  1  n  3 ... 2 if n is odd
 /2

Now I 0,n   cosn xdx   n n 2 3
0  n - 1  n  3 ... 1   if n is even
 n n 2 2 2

(Using the reduction formula for  cosn xdx )


203

m 1 m3 2 1
Thus, I m,n   ...  (Using the value of I1,n)
m n m n 2 3n n 1
If m is odd and n may be even or odd.
Now If m is even
 m  1  m  3 ... 1  n  1  n  3 ... 2 n is odd

I m ,n   m n m n 2 2n n n 2 3
 m  1 m  3 1 n  1 n 3 1 
 ...   ...  n is even
m  n m  n  2 2  n n n 2 2 2
(Using the value of I0,n)

Example 9.9 Evaluate


Solution. Using the reduction formula (9.15) for n and m both even, we get
=

In-Text Exercise 9.3 Solve the following questions:

2
1. Evaluate 0
sin 4 x cos 5 x dx

 /2

 sin 3  cos 4  cos 2 d


2. Evaluate 0

 /2
3. If I m,n   sinm x cosn xdx , prove that
0

n 1 
I m,n   I where m and n are positive integers.
 m  n  m,n  2

7
4. Find the value of integral I   x sin x cos4 x dx
0

9.7 Applications of the Reduction Formulae


The reduction formulae discussed in the previous sections help us to evaluate many more
integrals, which are otherwise tedious to be evaluated. In this section, we will discuss the
evaluation of some of these integrals as follows:
204 LESSON - 9. REDUCTION FORMULAE FOR INTEGRALS

To evaluate the integral of a function that contains an algebraic expression of the type
a 2  x 2 , x 2  a 2 , x 2  a 2 we transform the integral to an integral of a trigonometric
function of the type discussed in the previous sections.For example, to evaluate the
integral of a function containing a 2  x 2 , we substitute x  a sin  , for the integral
that contains the radical a 2  x 2 , we substitute x  a tan  and for the integral
containing the radical x 2  a 2 , we substitute x  a sec  .
a x4
Example 9.10 Evaluate 0
a2 – x 2
dx

Solution. Substituting x = a sin , we get dx = a cos  d


Therefore when x = a, sin  = 1   = /2
And when x = 0, sin  = 0   = 0

Therefore, =

= , cos

= . (by using the reduction formula


for ).

dx
Example 9.11 Evaluate  (a  x 2 )7
2
0

Solution. Substituting x  a tan  , dx  a sec2  d



When x  0,   0, when x  ,  
2

dx
 /2 a sec2  d 1
 /2
Therefore,      cos12  d
0
(a  x 2 )7
2
0
a 14 14
sec  d a 13 0

 /2
Using the reduction formula for 0 cosn  d , n is even

dx 1 11 9 7 5 3 1 
 2 2 7
(a  x )
 13 
a
     
12 10 8 6 4 2 2
0

1 231
  .
a 13 2048
205

a
2
Example 9.12 Evaluate x ax  x 2 dx
0

Solution. Substituting x  a sin2  , we get



dx  2a sin  cos d, x  0,   0, x  a,  
2
a  /2
2 2
Thus, x ax  x dx   a 2 sin 4  a 2 sin 2   a 2 sin 4   2a sin  cos  d
0  0
 /2
 2a 4  sin6  cos2  d
0

5 3 1 1 
 2a 4      (using the reduction formula for
8 6 4 2 2
2

0
sin n x cosm x dx )

5 a 4
 .
128
In-Text Exercise 9.4 Solve the following questions:

dx
1. Evaluate  (16  x 2 )9 / 2
.
0

dx
2. Evaluate  (1  x 2 )5
0
a
5
3. Reduce the algebraic expression x (2a 2  x 2 )3dx to the known trigonometric
0
form.
4
3
4. Evaluate x 4x  x 2 dx
0

9.8 Summary
In this lesson we have discussed the following points:
1. In this chapter of Reduction formulae for integrals, integration of higher order
trigonometric functions are discussed.
2. We have derived the Reduction formula for higher order trigonometric functions
sinnx, cosnx, tannx, cotnx, secnx, sinnxcosmx etc.
206 LESSON - 9. REDUCTION FORMULAE FOR INTEGRALS

3. We also learned how to reduce different algebraic expressions of the form


a 
2 2 dx 2 2 2 2 2 2
 x ax  x dx ,  (a 2  x 2 )7 , a  x , x  a , x  a etc to known
0 0
trigonometric functions by the method of substitution.
4. In this chapter we discussed fundamental properties of Integrable functions and used
them in evaluating different integrals.
5. Various applications of Reduction formulae and miscellaneous problems of
integration are also being discussed in the chapter.
Reduction formula for integrals plays a key role in integral calculus which has a wide
range of applications in evaluating length of curves,their volumes, surface area of solid of
revolution and many more applications of integration.

9.9 Self-Assessment Exercises


 /2
1. Evaluate:  sin 7 x dx
0

 /4
2. Evaluate:  cos6 2x dx
0

 /2
3. Evaluate:  cos9 x dx
0
a
x 7dx
4. Evaluate :  a2  x 2
0

dx
5. Evaluate:  (a  x 2 )7 / 2
2
0

1 3 1

6. Evaluate:  x 2 (1  x )2 dx
0

 /4
7. Evaluate:  tan 4 x dx
0

8. Calculate
 /2

 sin 9 x  cos3 x dx
0
207

 /2
9.  sin 8 x  cos4 x dx
0

 /2
10.  sin 5 x  cos6 x dx
0

9.10 Solutions to In-Text Exercises


Exercise 9.1
Solution 1
 /2
(6  1)(6  3)(6  5) 
 sin 6  d  
0
6(6  2)(6  4) 2

5.3.1  5
   .
6.4.2 2 32

Solution 2. Using the reduction formula for n=5 odd we have,

Solution 3. Using the reduction formula for n=3 odd

We have, =

Solution 4. = .

Exercise 9.2
Solution1.(i) we have

=
208 LESSON - 9. REDUCTION FORMULAE FOR INTEGRALS

= .

(ii) We have ,

= –

= + +log (sinx)

(iii) We have ,

= +

= +

= + +

= + +

(iv) We have ,

= + dx

= + dx

= - cotx cosecx + log tan .

Solution 2. Using reduction formula for  tann  d , we have

 /4  /4  /4
5  tan 4  
 tan  d      tan 3   d
0  4 0 0
209

 /4
1  tan2  
 /4 
  
4  2  0
   tan   d 

 0

 /4
1 1 sin  
    d 
4  2 0
cos  
 /4
1 1 sin 
    d
4 2 0
cos 

1
   log cos  0
4
1 1 1
  log  log 2 
4 2 4
 /4
1
Hence  tan 5  d  log 2  .
0
4

Solution 3. We have,

= +

= { .1 + log(1+ )].

Exercise 9.3
Solution 1. We have = . . .

 /2
Solution2. We have  sin 3  cos4  (cos2   sin2  )d
0

 /2  /2
3 6
  sin  cos  d   sin 5  cos4  d
0 0

2 1 4 2 1
    
9 7 9 7 5
210 LESSON - 9. REDUCTION FORMULAE FOR INTEGRALS

2 8
 
63 63  5
10  8 2
  .
63  5 315
Solution 3. Let I m,n   sinm x cosn xdx , which can be written as

I m,n   sinm x cosn 1 x cos x dx

  cosn 1 x (sinm x cos x )dx

integrating it by parts, taking cosn 1 x as first function and (sinm x cos x ) as second
function, we get
sinm 1 x sinm 1 x
I m,n  cosn 1 x   (n  1)cosn 2 x ( sin x )  dx
m 1 m 1
(using the fact if sin x  t  cos xdx  dt when gives
t m 1 sinm 1 x
 sinm x cos xdx   t mdt  or )
m 1 m 1
cosn 1 x sinm 1 x n  1
I m ,n    cosn 2 x sinm 2 x dx
m 1 m 1
cosn 1 x sinm 1 x n  1
   cosn 2 x sinm x (1  cos2 x )dx
m 1 m 1
cosn 1 x sinm 1 x n  1 n 1 
   sinm x cosn  2 x   m n
  sin x cos x dx
m 1 m 1 m  1
cosn 1 x sinm 1 x n 1 n 1 
Thus I m,n   I m,n 2    I m,n
m 1 m 1 m  1

1  n  1  I cosn 1 x sinm 1 x n  1
or   m,n   I
 m  1 m 1 m  1 m,n 2

 m  n I cosn 1 x sinm 1 x  n  1 
  m,n   I
m 1 m 1  m  1  m,n 2
cosn 1 x sinm 1 x  n  1 
or I m,n   I
m n  m  n  m ,n  2
 /2
n 1 
Now  sinm x cosn x dx   I
0
 m  n  m ,n  2
211

n 1 
or I m,n   I .
 m  n  m,n  2
Hence proved.

7
Solution4. We have I   x sin x cos4 x dx (1)
0

7 4
I   (  x )sin (  x )cos (  x )dx (Using property (iv) sec 9.3)
0

  (  x ) sin 7 x cos4 x dx (2)
0

Adding (1) and (2)



7
2I    sin x cos 4 x dx
0

 2I    sin7 x cos4 x dx
0


 I   sin7 x cos 4 x dx
2 0

2I    sin7 x cos4 x dx
0


I  sin7 x cos 4 x dx
2

0

 /2
 2
 I   sin 7 x cos 4 x dx (Using property (vi) Sec 9.3)
2 0

( sin7 (  x )  sin7 x, cos4 (  x )  cos4 x )


 /2
 I   sin 7 x cos 4 x dx
0

 m  1  m  3  2 1 
     ...   m odd, n even.
  m  n   m  n  2  (n  3) (n  1) 
6 4 2 1
    
 11 9 7 5 
212 LESSON - 9. REDUCTION FORMULAE FOR INTEGRALS

6 4 2 1
    
11 9 7 5
16 16
  .
63  35 1155

Exercise 9.4

Solution 1. Put x  0,   0 for x  ,  
2
  /2  /2
dx 4 sec2  d 1
Thus,    9 9
 8  cos7  d
0
(16  x 2 )9 / 2 0
4 sec  d 4 0

Using reduction formula for  cosn  d , n is odd.



dx 1 6 4 2 1
 2 9/2
(16  x )
 8    6
4 7 5 3 4 75
0

1
 .
143360

Solution 2. Put x  tan   dx  sec2 d , x  0,   0, x  ,    /2


 /2
sec2  d
 /2 sec   d  /2 sec2  d
 I      
0
(1  tan2  )5 0
(sec2  )5 0
sec10 
 /2  /2
d
 I     cos8  d
0
sec 8  0

7 5 3 1 
    
8 6 4 2 2
35
 .
256

Solution 3. Put x  2a sin   dx  2a cos  d , for x  0,   0 and



x  a,  
4
a
5
Thus I  x (2a 2  x 2 )3 dx becomes
0
213

 /4
I   ( 2a )5 sin 5  (2a 2 )3 cos 6   2a  cos   d
0

 /4
6 6
 ( 2a ) ( 2a )  sin 5   cos 5  d
0

 /4
sin 5 
 ( 2a )6  6  d
0
cos5 
 /4
  tan 5  d
0

2 
Solution 4. Put x  4 sin , x  0,   0, x  4,  
2
 dx  8 sin  cos  d
 /2
  (4)3 sin 6   sin2   16 sin 4   8 sin  cos  d
0

 /2
  2  (4)4 sin 7  cos   4 sin2  (1  sin2  ) d
0

 /2
  2  (4)5 sin 8  cos2   d
0

 /2
5
 2  4  sin 8   cos2  d
0

7 5 3 1 1 
 2   4 5      
10 8 6 4 2 2
7 5 1 
  4 4    
10 4  2 2 2
7 5 
  4 3   
25 2 2
 4  7    28

9.11 SUGGESTED READINGS


(i) B.V. Raman, Higher Engineering Mathematics, Tata McGraw Hill,2008.
(ii) V.P. Mishra, Pratibha Mishra, Concepts of Engineering Mathematic, V.P. Mishra
Publications, 2010.
214 LESSON - 9. REDUCTION FORMULAE FOR INTEGRALS

(iii) Shanti Narayan, Integral Calculus, S. Chand, 2019.


(iv) H. Anton, Calculus (10th edition), John Wiley & Sons.
Lesson - 10

Asymptotes, Concavity and Point of


Inflexion
Ms. Mridu Sharma
Campus of Open Learning
UUniversity of Delhii

Structure
10.1 Learning Objectives 215
10.2 Introduction 216
10.3 Asymptote 216
10.3.1 Horizontal Asymptotes 216
10.3.2 Vertical Asymptotes 218
10.3.3 Oblique Asymptote 219
10.4 Concavity of a Curve 221
10.4.1 Criteria for Concavity 221
10.5 Point of Inflexion 223
10.5.1 Criteria for Point of Inflexion 223
10.6 Summary 225
10.7 Self Assessment Exercise 225
10.8 Solutions to In-text Exercises 226
10.9 Suggested Readings 228

10.1 Learning Objectives


The learning objectives of this lesson are to:
• understand the notion of asymptotic behaviour of curves
• learn the methods to find the different types of asymptotes
• understand the concept of concavity of a function
• learn to find a point of inflexion.

215
216 LESSON - 10. ASYMPTOTES, CONCAVITY AND POINT OF INFLEXION

10.2 Introduction
Informally, an asymptote of a curve is a straight line which touches the curve at infinity. An
asymptote indicates the behaviour of a curve at the points far off from the origin. A curve
that lies wholly in a bounded region has no asymptote. For example, the circle x2 +y 2 = a2
has no asymptote. In this lesson, we will learn about different kinds of asymptotes and the
procedures to find them. We will also discuss the important concepts of the concavity and
the points of inflexion of a curve. All these concepts are useful in sketching the graphs of
various functions. We begin with the formal definition of an asymptote of a curve.

10.3 Asymptote
Definition 10.1. (Asymptote): A straight line is called an asymptote of a given curve
y = f (x), if the perpendicular distance between the line and the point A(x, y) on the curve
approach to 0 as x or y or both approach to infinity.

In otherwords, An asymptote of a given curve is a straight line which touches the given
curve at infinity. Asymptotes plays an important role in sketching graphs of the function.
It gives information about the behaviour of the curve at infinity. There are three kinds of
asymptotes, as given below:

(i) Asymptotes parallel to x− axis or horizontal asymptotes.

(ii) Asymptotes parallel to y− axis or vertical asymptotes.

(iii) Oblique asymptotes.

In the following sub-sections, we will discuss them one by one.

10.3.1 Horizontal Asymptotes


Definition 10.2. A horizontal line y = c is a horizontal asymptote of the curve y = f (x),
if the perpendicular distance of the point on the curve from the line y = c tends to 0 as
x → ∞(or − ∞). That is, if

lim f (x) = c or lim f (x) = c,


x→∞ x→−∞

then y = c is a horizontal asymptote of the curve y = f (x).

Example 10.1. Find the asymptote parallel to x− axis of the curve

3x2 + 8x − 5
y=
5x2 + 2
10.3. ASYMPTOTE 217

Solution. We have

lim y = lim f (x)


x→∞ x→∞
3x2 + 8x − 5
= lim
x→∞ 5x2 + 2
3 + 8/x − 5/x2
= lim
x→∞ 5 + 2/x2
3
=
5

Therefore, the line y = 35 or 5y − 3 = 0 is a horizontal asymptote to the given curve


y = f (x) on the right.
Similarly, we can show that

lim y = lim f (x)


x→−∞ x→−∞
3x2 + 8x − 5
= lim
x→−∞ 5x2 + 2
3 + 8/x − 5/x2
= lim
x→−∞ 5 + 2/x2
3
=
5

Therefore, the line y = 3/5 is a horizontal asymptote to y = f (x) on the left. That is, the
line y = 5/3 to a horizontal asymptote or an asymptote parallel to x− axis.

x2 +2x+1
Example 10.2. Find a horizontal asymptote of the curve y = x3 +1
.

Solution. We have

x2 + 2x + 1
lim y = lim
x→∞ x→∞ x3 + 1
1/x + 2/x2 + 1/x3
= lim
x→∞ 1 + 1/x3
= 0

Similarly,we can deduce that limx→−∞ y = 0 Hence, y = 0 is the only horizontal asymptote
for the given function.

Note. A shortcut to obtain horizontal asymptote is by equating the coefficient of the highest
degree term in x(provided it is not constant) to zero.

For example, y = 0 is the asymptote parallel to the x− axis of the curve x3 y + x2 y +


2xy + 1 = 0.
218 LESSON - 10. ASYMPTOTES, CONCAVITY AND POINT OF INFLEXION

10.3.2 Vertical Asymptotes


Definition 10.3. A vertical line x = d is said to be a vertical asymptote (asymptote parallel
to y− axis) of the curve y = f (x), if the function becomes unbounded. In otherwords,
when the value of y = f (x) tends to ∞ or −∞ as x → d.

Example 10.3. Find the asymptote parallel to x− axis of the curve

x2 + 8x − 3
y= 2
x − 5x − 6
Solution. Consider the denominator

x2 − 5x − 6 = (x − 6)(x + 1),

which means that two zeros of the denominator are x = 6 and x = −1.

x2 + 8x − 3
lim =∞
x→6 x2 − 5x − 6

x2 + 8x − 3
lim =∞
x→−1 x2 − 5x − 6

Therefore, the line x = 6 and x = −1 are vertical asymptote to y = f (x).

Example 10.4. Find the asymptote parallel to x− axis of the curve

x2 + y 2 − a2 (x2 + y 2 ) = 0

Solution. The given equation can be written as

x 2 y 2 − a2 x 2 − a2 y 2 = 0

⇒ (y 2 − a2 )x2 − a2 y 2 = 0
⇒ (x2 − a2 )y 2 − a2 x2 = 0
Clearly, The Asymptotes parallel to x-axis are given by y 2 − a2 = 0 ⇒ y = ±a.

We give the following rule for determining the Asymptotes parallel to the coordinate
axes:
RULE 1. The Asymptotes parallel to x-axis can be easily obtained by equating to zero the
coefficient of the highest degree term in x. However if the coefficient of the highest degree
term in x is constant or has imaginary(no real) factors then curve has no Asymptote parallel
to x-axis.
RULE 2. The Asymptotes parallel to y-axis can be easily obtained by equating to zero the
coefficient of the highest degree term in y. However if the coefficient of the highest degree
term in y is constant or has imaginary(no real) factors then curve has no Asymptote parallel
to x-axis.
10.3. ASYMPTOTE 219

10.3.3 Oblique Asymptote


The asymptote which are not parallel to any of the coordinate axes are called as Oblique
Asymptote.

Definition 10.4. Homogenous function A function f(x,y) in x and y is called as homogenous


function , if f (λx, λy) = λf (x, y) for some real number λ i.e., if the degree of each term
in the expression are equal.

For example, the function f (x, y) = 3x2 + 2xy + y 2 is a homogenous function since
the sum of the powers of x and y in each term is the same.
Procedure to find Oblique Asymptote Consider a function y = f (x, y) which can be
rewritten as:

f (x, y) = ϕn (x, y) + ϕn−1 (x, y) + ....... + ϕ1 (x, y) + c = 0 (10.1)

where ϕr (x, y) is homogenous expression in x and y of degree r. Let y = mx + c is an


asymptote to the given curve. Our aim is to evaluate the values of m and c.
Step 1. Put x = 1 and y = m in 10.3.3 to get the polynomials ϕn (m), ϕn−1 (m), ϕn−2 (m),
etc.
Step 2. The slopes of the asymptotes can be evaluated by finding the roots of the polynomial
equation ϕn (m)=0.
Step 3. To evaluate c, then consider the following two cases

I. m is non-repeated root, then the corresponding value of c is given by

−ϕn−1 (m)
c=
ϕ′n (m)

II. m is repeated root(repeated twice) ⇒ ϕ′n (m) = 0, then

(i) ϕn−1 (m) ̸= 0, then their doesn’t exist any oblique asymptote corresponding to
slope m.
c2 ′′
(ii) ϕn−1 (m) = 0, then ϕ (m)
2 n
+ cϕ′n−1 (m) + ϕn−2 (m) = 0

If three roots of equation ϕn (m) = 0 are equal, then c can be obtained by

c3 ′′ c2 c
ϕn (m) + ϕ′′n−1 (m) + ϕn−2 (m) + ϕn−3 (m) = 0
3! 2! 1!
Example 10.5. Find the Asymptotes of the curve

x3 + 5x2 + 2x − 2y 3 + 7y 2 + 2y + 2x2 y − y 2 x = 0

Solution. Step 1: Clubbing the like degree terms,

(x3 − 2y 3 + 2x2 y − y 2 x) + (5x2 + 7y 2 ) + (2x + 2y) = 0


220 LESSON - 10. ASYMPTOTES, CONCAVITY AND POINT OF INFLEXION

The homogeneous expressions of degree 3,2 and 1 are as follows:

ϕ3 (x, y) = x3 − 2y 3 + 2x2 y − y 2 x
ϕ2 (x, y) = 5x2 + 7y 2
ϕ1 (x, y) = 2x + 2y

Step 2: Put x = 1 and y = m, get the following polynomials,

ϕ3 (m) = 1 − 2m3 + 2m − m2
ϕ2 (m) = 5 + 7m2
ϕ1 (m) = 2 + 2m

Step 3: Evaluating the roots of the polynomial ϕ3 (m), we get


1
m = ±1, −
2
2
As all the slopes are non-repetitive, c = − ϕϕ′2 (m)
(m)
= − −6m7m2 −2m+2
+5
. Solving for each value of
3
m, we get

m = 1, c = 2
m = −1, c = 6
m = −1/2, c = −9/2

Thus, the three asymptotes are y = x + 2, y = −x + 6 and y = − 12 (x + 9).

Note. The coefficients of the highest degree term of x and y are constant, therefore there is
no asymptote parallel to x or y axis.

Example 10.6. Find the asymptotes of the curve f (x, y) = 4xy 2 + y 3 + 2x2 + 4x2 y − 4y 2 −
1=0

Solution. Step 1. f (x, y) = ϕ3 (x, y) + ϕ2 (x, y) − 1 where

ϕ3 (x, y) = y 3 + 4xy 2 + 4x2 y


ϕ2 (x, y) = 2x2 − 4y 2

Step 2. Put x = 1 and y = m in the above expressions,

ϕ3 (m) = m3 + 4m2 + 4m
ϕ2 (m) = 2 − 4m2

Step 3. The slopes of the asymptotes are given by,

m3 + 4m2 + 4m = 0 ⇒ m = 0, −2, −2
10.4. CONCAVITY OF A CURVE 221

Step 4. As m = 1 is a non-repeated root, then c = − ϕϕ2′ (m)


(m)
.
3

1
∴c=−
2
The equation of asymptote is y = −1/2 i.e. parallel to x− axis.
For the repeated root m = −2 with multiplicity 2,
c2 ′′ c
c= ϕ3 (m) + ϕ′2 (m) + ϕ1 (m) = 0
2! 1!
c2
(6m + 8) + c(−8m) = 0
2
3c2 m + 4c2 − 8cm = 0

With m = −2 in the above quadratic equation, we get c = 0, c = 8


Thus, the other two asymptotes are y = −2x and y = −2x + 8
In-text Exercise 10.1. Find the asymptotes of the following curves :
1) y 3 − x2 y + 2y 2 + 4y + x = 0

2) x3 + 2x2 y − y 2 − 2y 3 − xy 2 + xy − 1 = 0
a2 b2
3) x2
+ y2
=1

4) x2 y 2 − a2 (x2 + y 2 ) = 0

5) x2 y 3 + x3 y 2 = x3 + y 3

6) y 3 − x2 y − 2xy 2 + 2x3 − 7xy + 3y 2 + 2x2 + 2x + 2y + 1 = 0

7) (x3 + a3 )y = bx3

10.4 Concavity of a Curve

Figure 10.1: Concave Up

Definition 10.5. (Concave upwards at a point) A portion of the curve on both sides of
a point lies above any tangent line drawn to it on the point, then the curve is said concave
upwards at P .
222 LESSON - 10. ASYMPTOTES, CONCAVITY AND POINT OF INFLEXION

Definition 10.6. Concave downwards at a point) A portion of the curve on both sides of
a point lies below any tangent line drawn to it on the point, then the curve is said concave
downwards at P .

Definition 10.7. (Concave upwards on an Interval) A curve y = f (x) is said to be


concave upwards in an interval, if it is concave upwards at every point of that interval.
That is, if the curve bends upwards on that interval. In otherwords, the portion of the
curve corresponding to the interval, lies above the tangent line at any point of the curve
corresponding to the interval as shown in figure 9.2.

Definition 10.8. (Concave downwards on an Interval) A curve y = f (x) is said to be


concave downwards in an interval, if it is concave downwards at every point of that interval.
That is, if the curve bends downwards on that interval. In otherwords, the portion of the
curve corresponding to the interval, lies below the tangent line at any point of the curve
corresponding to the interval as shown in figure 9.3.

Figure 10.2: Concave Down

10.4.1 Criteria for Concavity


Let y = f (x) be a function defined on an open interval I, such that f ′′ (x) exists for x ∈
I. We know that the sign of the first order derivative f ′ (x) indicates whether the curve
y = f (x) is increasing or decreasing on I. For checking the concavity (or convexity) of
y = f (x) on I using the second order derivative f ′′ (x) for x ∈ I. We have the following
criteria:

(i) The curve y = f (x) is concave upwards on I if f ′′ (x) > 0 ∀x ∈ I

(i) The curve y = f (x) is concave downwards on I if f ′′ (x) < 0 ∀x ∈ I

Example 10.7. For f (x) = x3 − 3x2 + 1, find the intervals on which f (x) is

(i) concave upwards

(i) concave upwards


10.5. POINT OF INFLEXION 223

Solution. We have,
f (x) = x3 − 3x2 + 1
⇒ f ′ (x) = 3x2 − 6x
⇒ f ′′ (x) = 6(x − 1)
For x < 1, f ′′ (x) < 0 and x > 1, f ′′ (x) > 0.
So, f (x) is concave downwards in (−∞, 1) and is concave inwards in (1, ∞).

Example 10.8. Show that f (x) = ex is concave upwards everywhere and g(x) = ln(x) is
concave downwards everywhere.
Solution. Consider f (x) = ex , then
f ′ (x) = ex
f ′′ (x) = ex > 0 (always)
Hence, f (x) = ex is concave upwards everywhere.
For g(x) = ln(x), then
g ′ (x) = x1
g ′′ (x) = − x12 < 0 as x2 > 0
Hence, f (x) = ln(x) is concave downwards everywhere.

10.5 Point of Inflexion


Definition 10.9. (Point of Inflexion of a curve) A point on the curve y = f (x) at which the
curve changes its concavity from upwards to downwards or from downwards to upwards is
called as a point of Inflexion.
For example, f (x) = x1/3 has (0, 0) as an inflexion point which is depicted in Fig.11.8.

Figure 10.3: Point of Inflexion

Note. • A curve changes its shape at a point of inflexion.


• A curve crosses the tangent line at a point of inflexion.
224 LESSON - 10. ASYMPTOTES, CONCAVITY AND POINT OF INFLEXION

10.5.1 Criteria for Point of Inflexion


If P (a, f (a)) is a point of inflexion of the curve y = f (x), then f ′′ (a) = 0 or f ′′ (a) does
not exist.

Example 10.9. Discuss the concavity of f (x) = x3 and find its point of inflexion.

Solution. We have f (x) = x3 , x ∈ R.

⇒ f ′ (x) = 3x2
⇒ f ′′ (x) = 6x.

Now, f ′′ (x) = 6x imply that,

f ′′ (x) < 0 for x ∈ (−∞, 0) and f ′′ (x) > 0 for x ∈ (0, ∞).

Therefore, the curve f (x) = x3 is concave downwards on (−∞, 0) and concave upwards
on (0, ∞) and it has a point of inflexion at x = 0, as f ′′ (x) changes its sign as x passes
through x = 0.

From the above mentioned criteria, we note that if f ′′ (a) = 0, then the curve y = f (x)
may not have point of inflexion at x = a.
For example, Consider the function f (x) = x4 . Then

f ′′ (x) = 12x2 ⇒ f ′′ (0) = 0

But x = 0 is not a point of inflexion for f (x) = x4 as shown in Figure ??.


Similarly, if f ′′ (x) does not exist at a point x = a, the the curve y = f (x) may have a point
of inflexion at x = a. For example, the function f (x) = x1/3 has a point of inflexion at
x = 0, but f ′′ (0) does not exist as given in Figure 11.8.

Example 10.10. Find the concavity of the curve f (x) = x3 − 15x2 + 20x + 20.

Solution. Differentiating f (x) = x3 − 15x2 + 20x + 20 twice with respect to x,

f ′ (x) = 3x2 − 30x + 20


f ′′ (x) = 6x − 30

For x < 5, f ′′ (x) = 6(x − 5) is negative, thus f is concave downwards in (−∞, 5).
For x > 5, f ′′ (x) = 6(x − 5) is positive thus f is concave upwards in (5, ∞).
For point of inflexion f ′′ (x) = 0,

⇒ 6(x − 5) = 0 ⇒ x = 5
Clearly, x = 5 is the point of inflexion as f (x) changes from concave downwards to con-
cave upwards.

In-text Exercise 10.2. 1) Find the open intervals where the given curve is concave up
and concave down
10.6. SUMMARY 225

(a) f (x) = 2x + 3 sin x


(b) f (x) = x4 + 1
(c) f (x) = e−x

2) Find the point of inflexion for the curve


2x
(a) f (x) = 1+x2

(b) f (x) = (x − 1) x − 2
a2 (x−y)
(c) f (x) = x2

10.6 Summary
Following points have been discussed in this lesson
• An asymptote of a curve y = f (x) is a straight line if the perpendicular distance
between the line and the point A(x, y) on the curve approach to 0 as x or y or both
approach to infinity.

• There are three kinds of asymptotes of a curve y = f (x), namely,

– Horizontal Asymptote
– Vertical Asymptote
– Oblique Asymptote

• Concavity of y = f (x) at a point (concave upwards and concave downwards)

• Concavity of y = f (x) in an interval (concave upwards and concave downwards)

• Criteria for checking the concavity of a curve y = f (x).

• Point of inflection and criteria for obtaining point of inflection for given curve.

10.7 Self Assessment Exercise


1. Find the asymptotes for the following curves:

(a) y 3 − 6xy 2 + 11x2 y − 6x3 + x + y = 0


(b) (x + y)2 (x + 2y + 2) = x + 9y − 2
a2 b2
(c) +
x2 y 2
=1
(d) y 3 − x2 y − 2xy 2 + 2x3 − 7xy + 3y 2 + 2x2 + 2x + 2y + 1 = 0
(e) x3 + 3xy 2 − 4y 3 − x + y + 3 = 0
(f) x3 − 2x2 y + xy 2 − x3 − xy + 2 = 0
(g) x2 (x − y)2 + a2 (x2 − y 2 ) = a2 xy
226 LESSON - 10. ASYMPTOTES, CONCAVITY AND POINT OF INFLEXION

(h) y 2 (x − 2a) = x3 − a3
(i) x2 y + xy 2 + xy + y 2 + 3x = 0
(j) (y − a)2 (x2 − a2 ) = x4 + a4
(k) x3 + y 3 = 3axy.
2. Find the intervals in which f (x) = x5 is concave up and concave down. Also, find
the points of inflexion.

10.8 Solutions to In-text Exercises


Exercise 10.1

1) y = 0, y = x − 1, y = −x − 1
1−x
2) y = x, y = −x − 1, y = 2

3) x = ±b, y = ±a
4) y = ±a, x = ±a
5) y = ±1, x = ±1
6) y = −x − 2, y = x − 1, y = 2x
7) y = b, x = −a
Exercise 10.2

1) (a)f is concave downwards on [0, π] and concave upwards [π, 2π]


(−∞, 0)
(b)f is concave downwards on √ √ and concave upwards (0, ∞)
(c)concave downwards in [−1/ 2, 1/ 2] and concave upwards outside this interval.
√ √
2) (a)(0, 0) and [± 3, ± 23 ]
(b)[ 73 , 3√4 3 ]

(c)x = 0, x = ±a 3

10.9 Suggested Readings


1. Anton, Howard, Bivens, Irl and Davis, Stephen (2013). Calculus (10th edition).
Wiley India Pvt. Ltd.
2. B.V Ramana, Higher Engineering Mathematics,TATA MCGRAW HILL, 2008.
3. Shanti Narayan , Differential calculus, S. Chand,(1962).
4. Shanti Narayan, Integral Calculus, S. Chand, 2019.
5. V.P. Mishra, Concepts of Engineering Mathematics, V.P MISHRA Publications, 2010.
Lesson - 11

Tracing of Curves
Ms. Mridu Sharma
Campus of Open Learning
UUniversity of Delhi

Structure
11.1 Learning Objectives 227
11.2 Introduction 227
11.3 Procedure to trace the curve y = f (x) 228
11.4 Tracing of a polynomial function y = f (x) 228
11.5 Tracing of a Rational function y = f (x) 230
11.6 Tracing of a function of the form y 2 = f (x) 232
11.7 Summary 238
11.8 Self Assessment Exercise 239
11.9 Suggested Readings 240

11.1 Learning Objectives


The learning objectives of this lesson are to:

• identify the points required for the tracing of curves

• trace various types of curves in Cartesian coordinates

• learn the properties of curves through tracing

11.2 Introduction
In the previous lessons, we have discussed many important concepts to know the behaviour
of a curve y = f (x). These concepts, like tangents and normals, monotonicity, maxima
and minima, critical points, concavity(or convexity), asymptotes and points of inflexion,
are useful in determining the slope of a curve. In this lesson, we will use all these concepts
to trace some curves in x − y coordinates.

227
228 LESSON - 11. TRACING OF CURVES

11.3 Procedure to trace the curve y = f (x)


The procedure to trace a curve y = f (x) consists of the following steps:

1. To find the domain and symmetries of the function y = f (x).

• A curve is y = f (x) is symmetrical about x−axis if y occurs in even degree or


if f (x, −y) = f (x, y). For example, y 2 = 4ax.
• A curve y = f (x) is symmetrical about y−axis if x occurs in even degree or if
f (−x, y) = f (x, y). For example, x2 = 4ay.
• A curve is symmetrical about the line y = x if the equation of the given curve
remains unchanged on interchanging x and y i.e. f (x, y) = f (y, x). For exam-
ple, x3 + y 3 = 3axy.

2. To find the points of intersection of the curve with the axes x = 0 and y = 0.

3. To find the intervals where f (x) is increasing or decreasing.

4. To find the extreme values of y = f (x).

5. To find the intervals in which the curve is concave upwards or concave downwards.

6. To find the points of inflexion, if any.

7. To find asymptotes of the curve, if they exist.

11.4 Tracing of a polynomial function y = f (x)


Let us consider the following examples and trace the curves by using the steps specified in
the above section. It may be noted that such type of curves have no asymptote.

Example 11.1. Trace the curve y = f (x) = x2 − 1.

Solution. We have

1. f (−x) = x2 − 1 = f (x)
Therefore, the curve is symmetric about y−axis(i.e.x = 0).

2. Substituting y = 0 in y = x2 − 1, we get

x2 − 1 = 0 ⇒ x = ±1

Therefore, the curve intersects the x− axis(i.e. y = 0) at the points (−1, 0) and
(1, 0).
Again, substituting x = 0 in y = x2 − 1, we get y = −1. Therefore, the curve
intersects the y−axis at the point (0, −1).
11.4. TRACING OF A POLYNOMIAL FUNCTION y = f (x) 229

3. Sign of y: The points (−1, 0) and (1, 0) divide the x− axis in three intervals to check
the sign of y.

Interval Sign of y
(−∞, −1) +ve
(−1, 1) −ve
(1, ∞) +ve

4. We have f (x) = x2 − 1,

⇒ f ′ (x) = 2x and f ′′ (x) = 2

⇒ f ′ (x) = 0 for x = −1, 1


Therefore, f (x) is increasing or decreasing as shown in the following intervals:

Interval Sign of f ′ (x) Increasing/Decreasing


(−∞, −1) −ve Decreasing
(−1, 0) −ve Decreasing
(0, 1) +ve Increasing
(1, ∞) +ve Increasing

5. The critical point is given by f ′ (x) = 0, that is x = 0. Since f ′′ (0) = 2 > 0, the
curve has the minimum at x = 0.
That is, the point (0, −1) is the lowest point on the curve.
6. Asymptotes: (i) We have limx→∞ y = limx→∞ (x2 − 1) = ∞.
Also, limx→−∞ y = limx→−∞ (x2 − 1) = ∞.
Therefore, the curve has no horizontal asymptote.
(ii) Since, limx→a y = (a2 − 1) ̸= ∞(or − ∞), for any finite real number and
therefore, the curve has no vertical asymptote.
(iii) The curve has no oblique asymptote as xy does not tend to a finite number as
x → ∞.
So, the curve has no asymptote.
7. Concavity: Since f ′′ (x) = 2 > 0, ∀x ∈ R
Therefore, the curve is concave upwards for x ∈ R.
Hence, by using the above steps, we trace the curve as following:
Example 11.2. Trace the polynomial function f (x) = x3 − x2 − 6x.
Solution. The function f (x) = x3 − x2 − 6x is not symmetric about any axis. The roots
of the function are:
f (x) = x3 − x2 − 6x
⇒ f (x) = x(x − 3)(x + 2)
So, the points of intersection with x−axis are (0, 0), (3, 0) and (−2, 0). These three points
divide the x−axis in four intervals, the sign of y in these four intervals:
230 LESSON - 11. TRACING OF CURVES

Interval Sign of y
(−∞, −2) −ve
(−2, 0) +ve
(0, 3) −ve
(3, ∞) +ve

Now, to check the increasing/ decreasing behaviour of the function, we first evaluate the
critical points of the function f (x), i.e. f ′ (x) = 0,

⇒ 3x2 − 2x − 6 = 0

⇒ x ≈ 1.8, −1.1

Interval Sign of f ′ (x) Increasing/Decreasing


(−∞, −1.1) +ve Increasing
(−1.1, 0) −ve Decreasing
(0, 1.8) −ve Decreasing
(1.8, ∞) +ve Increasing

The given curve has no asymptote. Let us now move to concavity, f ′′ (x) = 6x − 2. The
root of second derivative divides the interval in two parts, (−∞, 3) and (3, ∞).
f ′′ (x) < 0, concave downwards in the interval (−∞, 3) and f ′′ (x) > 0, concave upwards
in the interval (3, ∞). Based on the above information, we can trace the curve of f (x) as
follows:

11.5 Tracing of a Rational function y = f (x)


Let us consider an example to discuss the tracing of a rational function
3x+6
Example 11.3. Trace the rational function y = x−1
.

Solution. We have

1. The curve is not symmetric about any axis.

2. The curve intersects the x− axis(i.e. y = 0) at the point (−2, 0) and intersects the
y−axis at the point (0, −6).

3. Sign of y : The point (−2, 0) divides the x− axis in two intervals to check the sign
of y.

Interval Sign of y
(−∞, −2) +ve
(−2, ∞) −ve
11.5. TRACING OF A RATIONAL FUNCTION y = f (x) 231

3x+6
4. We have f (x) = x−1
,

9 18
⇒ f ′ (x) = − 2
and f ′′ (x) =
(x − 1) (x − 1)3

Therefore, f (x) is decreasing throughout the interval.

5. The curve has no critical point.


That is, the point (0, −1) is the lowest point on the curve.

6. Asymptotes: (i) We have limx→∞ y = limx→∞ 3x+6 x−1


= 3.
3x+6
Also, limx→−∞ y = limx→−∞ x−1 ) = 3.
Therefore, the curve has y = 3 as horizontal asymptote.
(ii) Since, limx→1 y = ∞, and therefore, the curve has vertical asymptote x = 1.
(iii) The curve has no oblique asymptote. So, the curve has no asymptote.

7. Concavity: Since f ′′ (x) > 0, ∀x > 1 and f ′′ (x) < 0, ∀x < 1.


Therefore, the curve is concave upwards for x > 1 and concave downwards for
x < 1.
Also, x = 1 is the point of inflexion. Hence, by using the above steps, we trace the
curve as following:

Example 11.4. Trace the curve y(a2 + x2 ) = a3


a3
Solution. Consider the curve f (x) = a2 +x2
. We can follow the steps mentioned in the
section 11.3.

1. Symmetric: The given curve is symmetrical about y−axis as the function is of even
degree.

2. Intersection with coordinate axes: The curve doesn’t intersect the x−axis whereas it
intersects y−axis at (0, a). Also, it does not passes through origin.

3. Asymptote: Asymptote parallel to x−axis is given by ya2 = 0 or y = 0 and asymp-


tote parallel to y−axis is given by (x2 + a2 ) = 0 or x = ±ia (doesn’t exist).
 
a3 dy
4. For tangent at (0, a), we have y = x2 +a2 . ⇒ dx = 0 ⇒ y = a is the tangent
(0,a)
at (0, a).
a3 −ya2 a2 (a−y)
5. Region: x2 = y
or x2 = y

(a) For y > a, x2 is −ve (no portion of the curve lies in the region y > a)
(b) For 0 < y < a, x2 is +ve (curve lies)
(c) For y < 0 say y = −2a, x2 is −ve (no portion of the curve lies in the region
y < 0)
232 LESSON - 11. TRACING OF CURVES

a3
6. Monotonicity: As f (x) = a2 +x2
, then

2a3 x
⇒ f ′ (x) = −
(a2 + x2 )2
So, the only critical point is x = 0 which divides the x− axis into two parts,

Interval Sign
(−∞, 0) +ve
(0, ∞) +ve

7. Asymptotes: y = 0 is the horizontal asymptote and is the only asymptote for the
given curve.

8. Further, we give some values to x as x = a2 , a, 2a, ... to trace the curve more precisely.

Figure 11.1: Curve of the equation y(x2 + a2 ) = a3

11.6 Tracing of a function of the form y 2 = f (x)


Example 11.5. Trace the curve y 2 (a2 − x2 ) = a3 x
Solution. We can proceed in similar pattern as the above examples. Consider the function
3x
y 2 = a2a−x 2:

1. Symmetry: The given equation is of even degree in y,so the curve is symmetrical
about x−axis.

2. Origin: The curve passes through the origin (0, 0).

3. The tangent at the origin is given by a3 x = 0 or x = 0 i.e. y−axis.

4. Asymptotes: Asymptote parallel to x-axis is given by ya2 = 0 or y = 0. Asymptotes


parallel to y-axis are given by (x2 + a2 ) = 0 or x = ±a.
11.6. TRACING OF A FUNCTION OF THE FORM y 2 = f (x) 233

5. Intersection with coordinate axes: For intersection with x-axis, we put y = 0 in the
given equation and for intersection with y-axis, we put x = 0 in the given equation.
Clearly, (0, 0) is the only point of intersection with the axes.
a3 x
6. Region of absence of curve: We write the given curve as y 2 = a2 −x2
. Now,
(a) For 0 < x < a, y 2 is +ve (curve lies)
(b) For −a < x < 0, y 2 is -ve (the curve doesn’t lie in the region −a < x < 0)
(c) For x > a, y 2 is -ve (the curve doesn’t lie in the region −a < x < 0)
(d) For x < −a, y 2 is +ve (curve lies)
7. Monotonicity:
8. Asymptote:
9. Concavity:
10. To trace it more accurately, take some points such as x = a2 , a3 , − a2 , − a3 , .... and trace
the curve more precisely.

Figure 11.2: Curve of equation y 2 (a2 − x2 ) = a3 x


234 LESSON - 11. TRACING OF CURVES

Example 11.6. Trace the curve y 2 (a + x) = x2 (3a − x).


Solution. By following the steps mentioned below, we can trace the curve,
1. Symmetry: the given equation is of even degree in y,so the curve is symmetrical
about x−axis.
2. The given equation satisfies x = 0 and y = 0 so curve passes through origin.
3. Further, tangent at the origin is given by putting lowest degree term equal to zero i.e.
a(y 2 − 3x2 ) = 0
⇒ y 2 = 3x2

⇒ y = ± 3x
Thus, two real and distinct tangents exist at origin.
4. Asymptotes:
Parallel to x-axis : doesn’t exists.
Parallel to y-axis : x + a = 0 ⇒ x = −a.
5. For point of intersection with x-axis put x = 0 we get y = 0. For point of intersection
with y-axis put y = 0 we get x = 0,x = 3a. Therefore, (0, 0) and (3a, 0) are the
points where the curve touches the coordinate axes.
6. Tangent at (3a, 0): Let us change the origin from (0, 0) to (3a, 0). Replace x by
x + 3a in the equation and then check for tangent at (3a, 0).
Then y 2 (x + 4a) = (x + 3a)2 (−x)
or, xy 2 + 4ay 2 = (x2 + 9a2 + 6ax)(−x)
or, xy 2 + 4ay 2 = −x3 − 9a2 x − 6ax2
Now, put lowest degree term equal to zero 9ax2 = 0 ⇒ x = 0 (y-axis). Thus, y-axis
is the tangent at point (3a, 0).

x2 (3a−x)
7. Region: We have y 2 = x+a

(a) y 2 < 0 when x < −a i.e. for x = −2a, −3a , ....


(b) y 2 < 0 when x > 3a
Thus, no portion of the curve lies in the region x < −a and x > 3a.
8. Find some more points on the curve by giving values to x such as x = a2 , 2a,.... to
trace the curve more precisely.

REGION OF ABSENCE OF CURVE


If possible find the region of the plane where no part of the curve lies. such a region is
obtained on solving the given equation for y 2 in terms of x (or x2 in terms of y). Suppose
y 2 < 0 for x > a .Similarly if x2 < 0 for y > b , then the curve does not lies in the region
y > b.
11.6. TRACING OF A FUNCTION OF THE FORM y 2 = f (x) 235

Figure 11.3: Curve of the equation y 2 (a + x) = x2 (3a − x)

Example 11.7. Trace the curve x2 (x2 + y 2 ) = a2 (x2 − y 2 ).

Solution. We need to follow the steps mentioned below.

Step 1. Curve is symmetrical about about both the axes. Since powers of x and y are
both even in the equation.

Step 2. x = 0, y = 0 satisfies the given equation thus curve passes through origin.

Step 3. Tangents at the origin is given by putting lowest degree term equal to zero,

i.e. x2 − y 2 = 0.

⇒ y = ±x. We get two real and distinct tangent at the origin. Thus, origin is a Node.
236 LESSON - 11. TRACING OF CURVES

Step 4. Asymptotes:

Parallel to x-axis doesn’t exist.

Parallel to y-axis is given by x2 + y 2 = 0 ⇒ x = ±ia (does not exist).

Step 5. Point of intersection:

With x-axis put y=0 which gives x = 0, x = ±a

With y-axis put x=0 which gives y=0.

Thus, (0,0),(a,0),(-a,0) are the point of intersection with coordinate axes.

Step 6. Tangent at (a,0) and (-a,0)

Let us shift the origin to the point (a,0) by putting x = x+a, in the given equation. Also,
shift the origin to the point (-a,0) by putting x = x-a, then find the tangents.

We see that y-axis is the only tangent.

Step 7. Region:
x2 (a2 −x2 )
We have y 2 = x2 +a2

y 2 < 0 when x2 > a2 or x > a and x < −a.

Thus, no portion of the curve lies where x > a and x < −a.

Step 8. We can further trace the curve by taking more points like x = a2 , a3 , −a
2
, −a
3
,...

Example 11.8. Trace the curve


ay 2 = x(a2 − x2 ), a>0
Solution. We need to follow the steps mentioned below.

Step 1. Symmetry: the given equation of curve is even in y, so curve is symmetrical


about x-axis.

Step 2. x = 0, y = 0 satisfies the equation thus the curve passes through the origin.

Step 3. Tangent at the origin can be found by putting lowest degree term equal to zero
in the equation, i.e.
11.6. TRACING OF A FUNCTION OF THE FORM y 2 = f (x) 237

Figure 11.4: Curve of equation x2 (x2 + y 2 ) = a2 (x2 − y 2 )

xa2 = 0
⇒ x = 0 (y-axis)

Thus, y-asis is the tangent at the origin.

Step 4. Asymptote:

Parallel to x-axis does not exists.

Parallel to y-axis does not exists.

Step 5. Point of intersection:

For intersection with x-axis put y = 0 in the equation, we get

x(a2 − x2 ) = 0 ⇒ x = 0, ±a

Step 6. Tangent at (a, 0) and (−a, 0).


238 LESSON - 11. TRACING OF CURVES

Shift the origin from (0, 0) to (a, 0) by substituting x = x + a in the given equation, we
get

ay 2 = (x + a)[a2 − (x + a)2 ]

= (x + a)(a2 − x2 − a2 − 2ax)

⇒ ay 2 = (x3 + 2ax2 + ax2 + 2a2 x)

Tangent at (a, 0) is given by putting lowest degree term equal to zero in above equation,

i.e. 2a2 x = 0 ⇒ x = 0.

Thus, x=0 (y-axis) is the tangent at (a, 0)

Similarly, at (−a, 0) y-axis is the tangent.

Step 7. Region:
x(a2 −x2 )
We can write the given equation as y 2 = a

(a) For 0 < x < a , y 2 is +ve (curve lies)

(b) For −a < x < 0 , y 2 is -ve (curve does not lie)

(c) For x > a , y 2 is -ve (curve does not lie)

(d) For x < −a , y 2 is +ve (curve lies)

Step 8. Further to trace the curve more accurately give values to x like x = a2 , −2a, −3a,......and
find the corresponding values of y. Plot these points.

11.7 Summary
Curve tracing is a technique of drawing rough sketches of the algebraic curves by follow-
ing some standard steps like symmetry of the curve about axes, origin of the curve, shifting
origin to the point of intersection of the curves with x-axis and y-axis.

Finding tangents at the origin and at the point of intersection of the curve with the co-
ordinate axes. Finding multiple or double points of a curve such as Node, cusp, isolated
point. Finding asymptotes to a curve, region of absence and existence of a curve.

By following these very basic steps students can trace various algebraic curves and learn
their properties.which enhances their knowledge of drawing curves and solving various
problems of mathematics.
11.8. SELF ASSESSMENT EXERCISE 239

Figure 11.5: Curve of equation ay 2 = x(a2 − x2 ), a>0

11.8 Self Assessment Exercise


Trace the following curves:
1. a2 x2 = y 3 (2a − y)
2. 4ay 2 = x(x − 2a)2
3. 3ay 2 = x(x − a)2
4. xy 2 + (x + a)2 (x + 2a) = 0
2 2 2
5. (x) 3 + (y) 3 = a 3
6. y(x2 + 4a2 ) = 8a3
7. y 2 x = a2 9a − x)
8. y 2 (2a − x) = x3
9. y 2 (a2 + x2 ) = x4
10. y 2 x2 = x2 + a2
240 LESSON - 11. TRACING OF CURVES

11.9 Suggested Readings


1. B.V. Ramana , Higher Engineering Mathematics, TATA MCGRAW HILL.

2. V.P. Mishra, Conceots of Engineering Mathematics, V.P.Mishra Publications,2010.

3. Shanti Narayan, Integral Calculus, S. Chand, 2019.

4. Shanti Narayan, Differential Calculus, S. Chand, 1962.


978-93-95774-36-9

9 789395 774369

You might also like